You are on page 1of 62

www.toprankers.

com

Monthly Edition
www.toprankers.com

MONTHLY EDITION
SEP, 2021 VOL - 01 ISSUE - 20 ` 100/-

CURRENT
AFFAIRS
PRACTICE
SHEETS (CAPS)
T he geographical influence
of Maoists has contracted to
only [1] districts in the country,
Chief Ministers of Jharkhand,
Madhya Pradesh, Bihar,
Telengana, Maharashtra and

Naxalbari movement start?
a. 1961
reduced this year the
number of deaths have
increased.
b. 1963
a sharp reduction from 96 such Odisha were present while b. 8 districts have been
districts in 10 States in 2010, Chattisgarh, West Bengal and c. 1965
categorized as ‘District
according to data provided by the Kerala were represented by d. 1967 of Concern’.
Ministry of Home Affairs (MHA) Directors General of Police
to Chief Ministers and other and Chief Secretaries. Andhra 3. In which year did the Union c. Both a and b
officials at a meeting held on Pradesh was represented by its Home Ministry first create d. Neither a nor b
Sunday .“These miscreants have Home Minister. the LWE desk?
been pushed to a few pockets 5. When was the Aspirational
a. 2002
with only 25 districts accounting 1. Which of the following has Districts Programme
been redacted by [1]? b. 2006 launched by the
for 85% of the LWE violence in
the country,” the Ministry said a. 35 c. 2009 Government of India?
in a statement. Union Home b. 41 d. 2014 a. 2016
Minister Amit Shah chaired the b. 2017
meeting to review the security c. 47 4. Which of the following
d. 59 statements is NOT correct? c. 2018
and other developmental aspects
in the Left Wing Extremism a. Though the number d. 2019
(LWE) affected States. 2. In which year did the of LWE incidents have

1. b. 41 against local landlords who LWE incidents have reduced them to bounce back in the
The geographical influence of bashed a peasant over a this year the number of areas recently taken away from
Maoists has contracted to only land dispute. The rebellion deaths have increased. LWE influence, 8 districts have
41 districts in the country, was initiated in 1967, with The number of deaths reduced been categorized as ‘District of
which is a sharp reduction an objective of rightful from 908 to 110 during the Concern’.
from 96 such districts in 10 redistribution of the land to same period. The Union Home 5. c. 2018
States in 2010. LWE incidents working peasants under the Ministry first created the LWE Launched in 2018, the
have also reduced from 2,258 leadership of Kanu Sanyal and desk, a separate department Aspirational Districts
in 2009 to 349 incidents till Jagan Santhal. to address the Maoist issue Programme aims to rapidly
August 2021. 3. b. 2006 in 2006.“Few new areas have transform the districts that
2. d. 1967 The Union Home Ministry been identified as potential have shown relatively lesser
The term Naxalism derives first created the LWE desk, a locations for emergence of progress in key social areas.
its name from the village separate department to address this phenomenon. To arrest
Naxalbari of West Bengal. the Maoist issue in 2006. the expansion plan of the CPI
It originated as a rebellion 4. a. Though the number of (Maoist) and also to restrict
Bhopal +91-7676564400 Indore +91-9589613810, +91-731-4987379 Prayagraj +91-8114000926, +91-8114000927 Kanpur +91-8576066660, +91-8707605589 E: support@toprankers.com
Gurugram +91-8448444207 Delhi +91-9810610466 Ranchi +91-9334969993 Lucknow +91-6390576666 Jabalpur +91-7004386936 W: www.toprankers.com
2 The CLAT Post • September 2021

Passage 2

T he [1] UN Food Systems


Summit saw nearly 300
commitments from hundreds
of thousands of people from
around the world and across
all constituencies to accelerate
action and to transform food
systems.
The Summit process gave rise
to several multi-stakeholders’
initiatives led by civil society,
farmers, women, youth and
indigenous groups that Member
States commit to in order to
deliver on the priorities, needs,
and gaps identified in national
pathways.
“Indigenous Peoples have been
supporting the Summit. We
have organized dialogues in the
seven socio-cultural regions, with
almost 300 Indigenous Peoples
has been very inclusive,” said 2. In 2021, UN Food Systems system’ mean?
organizations participating,
President of the Pan-African Summit took place in: 1. It refers to the
said Indigenous Peoples rights
Farmers Organizations (PAFO), a. California constellation of
activist and Summit Advisory
Elizabeth, Nsimadala, who activities involved in
Committee member Myrna b. New York
represents 80 million farmers producing, processing,
Cunningham, who launched c. Rome
across 50 African countries, and transporting and
the Indigenous Peoples’ Food
is a member of the Summit’s d. Toronto consuming food.
Systems at a UN press briefing on
Advisory Committee. “As
Thursday. 3. _______ action areas to 2. It refers to the chain
producers we held several
The 148 commitments that independent dialogues at all help inform the transitions of producers that are
have been registered so far levels and these dialogues needed to realise the vision farmers, around the
are collective or institutional resulted into a global common of the 2030 agenda have world
commitments to action that are position.” emerged from the Summit 3. It is another name of
aligned to the Summit’s Action process. ecosystem
Areas, and come out as a result 1. Which of the following can
a. three a. 1 only
of an 18-month inclusive and be redacted by [1]?
engaging process with diverse b. four b. 3 only
a. First
stakeholders. c. five c. 1 and 2 only
b. Second
The Summit process was also d. six d. 1 and 3 only
applauded by farmer leaders c. Third
for its inclusivity. “The Summit d. Fourth 4. What does the term ‘food

1. a. First for global food systems Advance equitable livelihoods, transporting and consuming
The first-ever UN Food transformation to achieve decent work and empowered food.
Systems Summit saw nearly the Sustainable Development communities. Food systems touch every
300 commitments from Goals by 2030. Build resilience to aspect of human existence.
hundreds of thousands of 3. c. five vulnerabilities, shocks and The health of our food systems
people from around the world Five action areas to help stresses. profoundly affects the health
and across all constituencies inform the transitions needed Support means of of our bodies, as well as the
to accelerate action and to to realise the vision of the 2030 implementation. health of our environment, our
transform food systems. agenda have emerged from 4. a. 1 only economies and our cultures.
2. b. New York the Summit process. These The term “food system” When they function well, food
The UN Food Systems Summit include: refers to the constellation systems have the power to
is taking place in New York. Nourish all people. of activities involved in bring us together as families,
It will seek to set the stage Boost nature-based solutions. producing, processing, communities and nations.
Bhopal +91-7676564400 Indore +91-9589613810, +91-731-4987379 Prayagraj +91-8114000926, +91-8114000927 Kanpur +91-8576066660, +91-8707605589 E: support@toprankers.com
Gurugram +91-8448444207 Delhi +91-9810610466 Ranchi +91-9334969993 Lucknow +91-6390576666 Jabalpur +91-7004386936 W: www.toprankers.com
3 The CLAT Post • September 2021

Passage 3 impactful than a large-scale d. periodically wet and dry mangrove cover in India.

B iological diversity is deforestation, these patches form due to waves and tides a. West Bengal
important for the functioning the habitat of several different
2. The Sunderbans are b. Gujarat
of the ecosystem and for humans flora and fauna species that are
rare and threatened. affected due to the polluted c. Andaman & Nicobar
to sustain. Moreover, local
livelihoods of humans also With these ecosystems being discharges from shrimp d. Goa
depend on a thriving biologically lost continuously, rare species ponds. In which of the
diverse ecosystem. However, are now left with fragmented following states have 5. India has lost ____% of
according to expert Krishna Ray, and fragile habitats among the maximum area under the area under its four
despite the efforts made across mangrove, and their movement shrimp cultivation? biodiversity hotspots,
the country to motivate people and dispersal are now full of a. West Bengal with the Indo-Burma
to support the conservation of obstacles. A major problem hotspot taking the worst
b. Arunachal Pradesh
biodiversity, Indian Sundarbans is that this loss of biodiversity hit, a report released by
c. Goa Centre for Science and
are still witnessing a loss of is also being neglected, and
biodiversity across the settlement for these species, cutting the d. Maharashtra Environment.
zones’ shorelines. This is existing mangrove and planting a. 50
3. Sunderban is a home of:
unfortunate because several rare a new one will not work, which b. 60
species of flora and fauna are means that this loss is essentially 1. critically endangered
northern river terrapin c. 80
important in the maintenance of irreversible.
the integrity and complexity of 2. endangered Irrawaddy d. 90
the ecosystem. 1. ‘Shoreline’ is mentioned in dolphin
the paragraph. It is divided 6. How many biodiversity
Ray said that the Sundarbans are 3. vulnerable fishing cat hotspots are present in
into 5 primary zones. What
gradually losing small patches a. 1 and 2 only India?
of mangroves due to activities does offshore zone mean?
related to either short-term a. below water b. 2 and 3 only a. 3
gains or for coastal development. b. area of shore that is c. 1 and 3 only b. 4
While the loss of small mangrove affected by waves d. 1,2 and 3 c. 6
patches might seem less c. where waves break d. 8
4. _______ has maximum

1. a. below water hummocky cross-stratified wise potential and present the Indo-Burma region and
Shorelines are divided into sand. The surf zone is where level of development. the Sundaland, that hosts
five primary zones—offshore, the waves break. 3. d. 1,2 and 3 numerous endemic species.
nearshore, surf, foreshore, The foreshore zone Sunderban is also home Around 24.46% of the
and backshore. The offshore overlaps the surf zone and to a large number of “rare country’s geographical area
zone is below water, but it is is periodically wet and dry and globally threatened covered by forests and trees.
still geologically active due to due to waves and tides. The species, such as the critically 6. b. 4
flows of turbidity currents that foreshore zone is where endangered northern river Officially, four out of the 36
cascade over the continental planer-laminated, well-sorted terrapin (Batagur baska), the Biodiversity Hotspots in the
slope and accumulate in the sand accumulates. The beach endangered Irrawaddy dolphin world are present in India: the
continental rise. The nearshore face is the part of the foreshore (Orcaella brevirostris), and Himalayas, the Western Ghats,
zone is the area of the shore zone where the breaking waves the vulnerable fishing cat the Indo-Burma region and the
affected by the waves where swash up and the backwash (Prionailurus viverrinus).” Sundaland.
water depth is one-half flows back down. 4. a. West Bengal 4. Dark energy, the mysterious
wavelength or less. The width 2. a. West Bengal West Bengal (46.39%) has the form of energy that makes up
of this zone depends on the The estimated brackish water maximum mangrove cover about 68% of the universe,
maximum wavelength of the area suitable for undertaking in the country, followed has intrigued physicists and
approaching wave train and shrimp cultivation in India is by Gujarat (22.55%) and astronomers for decades. Dark
the slope of the seafloor. The around 11.91 lakhs ha spread Andaman & Nicobar Islands energy has been noted as “the
nearshore zone includes the over 10 states and union (13.26%) of the country’s total most profound mystery in all
shoreface, which is where sand territories viz; West Bengal, mangrove cover. of science”. With advanced
is disturbed and deposited. Orissa, Andhra Pradesh, 5. d. 90 technologies and newer
The shoreface is broken into Tamil Nadu, Pondicherry, India has lost 90% of the area experiments, scientists have
two segments: upper and lower Kerala, Karnataka, Goa, under its four biodiversity found certain clues about it
shoreface. Upper shoreface Maharashtra and Gujarat. Of hotspots, with the Indo-Burma and, last week, an international
is affected by everyday wave this only around 1.2 lakhs ha hotspot taking the worst hit, team of researchers made the
action and consists of finely- are under shrimp farming now a report released by Centre first putative direct detection
laminated and cross-bedded and hence lot of scope exists for Science and Environment. of dark energy.
sand. The lower shoreface for entrepreneurs to venture India has four biodiversity
is the only area moved by into this field of activity. The hotspots, including the
storm waves and consists of following table gives the state- Himalayas, the Western Ghats,
Bhopal +91-7676564400 Indore +91-9589613810, +91-731-4987379 Prayagraj +91-8114000926, +91-8114000927 Kanpur +91-8576066660, +91-8707605589 E: support@toprankers.com
Gurugram +91-8448444207 Delhi +91-9810610466 Ranchi +91-9334969993 Lucknow +91-6390576666 Jabalpur +91-7004386936 W: www.toprankers.com
4 The CLAT Post • September 2021

Passage 4 1. Which of the following has xT is the name of another form of energy

T hey noticed certain been redacted by [1]? experiment for dark matter d. None of the above
unexpected results in a. XENON1T and energy. Where is it
an underground experiment located? 5. The dark energy is known
b. XERON1T
and write that dark energy a. USA as the -
c. ALTUS2S
may be responsible for it. The b. UK a. Third fundamental force
[1] experiment is the world’s d. ALTUS3D of nature
c. China
most sensitive dark matter 2. Which of the following has b. Fourth fundamental
experiment and was operated d. Japan
been redacted by [2]? force of nature
deep underground at the INFN 4. What does the word ‘dark’ c. Fifth fundamental force
Laboratori Nazionali del Gran a. France
in ‘dark matter’ denote? of nature
Sasso in [2]. The finding also b. Germany
suggests that experiments like a. The colour of the matter d. Sixth fundamental force
c. Italy
[1], which are designed to b. The unknown of nature
d. Belgium
detect dark matter, could also c. The fact that light is
be used to detect dark energy. 3. Other than [1], PandaX- fully absorbed in this

1. a. XENON1T underground research center Underground Laboratory – something that can’t


The XENON1T experiment in the world. Situated below (CJPL) in Sichuan, China. be explained by the four
at the Laboratori Nazionali Gran Sasso mountain in The experiment occupies forces. To hide or screen
del Gran Sasso (LNGS) is Italy, it is well known for the deepest underground this fifth force, many
the first WIMP dark matter particle physics research laboratory in the world, and is models for dark energy use
detector operating with a by the INFN. In addition among the largest of its kind. special mechanisms. Some
liquid xenon target mass above to a surface portion of the 4. b. The unknown theorists have named this
the ton-scale. Out of its 3.2t laboratory, there are extensive The term ‘dark’ is used to “quintessence,” after the
liquid xenon inventory, 2.0t underground facilities beneath denote the unknown. fifth element of the Greek
constitute the active target of the mountain. 5. c. Fifth fundamental force philosophers. However, none
the dual-phase time projection 3. c. China of nature of the theories have been
chamber. The Particle and Astrophysical There are four fundamental proved. Due to this, Dark
2. c. Italy Xenon Detector, or PandaX, forces in the universe, energy has been noted as “the
Laboratori Nazionali del is a dark matter detection and speculative theories most profound mystery in all
Gran Sasso is the largest experiment at China Jinping have proposed a fifth force of science”.
Bhopal +91-7676564400 Indore +91-9589613810, +91-731-4987379 Prayagraj +91-8114000926, +91-8114000927 Kanpur +91-8576066660, +91-8707605589 E: support@toprankers.com
Gurugram +91-8448444207 Delhi +91-9810610466 Ranchi +91-9334969993 Lucknow +91-6390576666 Jabalpur +91-7004386936 W: www.toprankers.com
5 The CLAT Post • September 2021

Passage 5

I taly on Friday recognised


Covishield, the vaccine
against coronavirus jointly
developed by Oxford University
and pharma giant [1] and
manufactured by the Serum
Institute of India. This would
mean Indian cardholders
inoculated by the vaccine
would now be eligible for an
Italian green pass, the Indian
embassy in Italy said on Sept
24. The embassy credited Union
health minister [2]’s meeting
with his Italian counterpart
Roberto Speranza and the
ministry of external affairs
(MEA) continuous efforts for
the recognition.
“As an outcome of a meeting
between Health Minister [2]
and his Italian counterpart
Roberto Speranza coupled with
MEA’s persistent efforts, Italy
recognised India’s Covishield.
Indian vaccine cardholders are
now eligible for Green Pass,”
the embassy said in a statement.
[2], who was in Rome, Italy, 2. Which of the following authorized to approve the d. Dharmendra Pradhan
for the G20 health ministers’ statements is/are true usage of vaccine for public 5. Which of the following is a
meeting early September, regarding Covishield vaccination in the country? major diplomatic effort to
held talks with Speranza on Vaccine? a. Ministry of Health gift and supply made-in-
September 6 and discussed a. Covishield is technically b. Aayush Ministry India vaccines to low-
prioritising travel for vaccinated referred to as AZD1222 income and developing
c. Central Drugs Standard
Indian students among other or ChAdOx 1 nCoV-19. countries globally?
Control Organization
issues. b. It is a Non-Replicating a. Vaccine Maitri
d. Indian Medical
Viral Vector type of b. Vaccine Sahayta
1. Which of the following Association
Vaccine
replaces [1] in the passage? c. Vaccine Help
c. The current gap 4. Which of the following
a. Johnson & Johnson d. Vaccine – Bridging the
between the two dosage replaces [2] in the passage?
b. AstraZeneca gap
of the vaccine is 84 days. a. Mansukh Mandaviya
c. Sinopharm d. All of the above b. Anurag Thakur
d. Zydus Cadila
3. Which of the following is c. Jyotiraditya Scindia

1. b. AstraZeneca ChAdOx 1 nCoV-19. It is a responsible for approval of Family Welfare and Chemical
Serum Institute of India’s Non-Replicating Viral Vector licences of specified categories and Fertilizers of India.
COVID-19 vaccine, called type of Vaccine. The current of drugs such as blood and 5. a. Vaccine Maitri
Covishield, is a version of the gap between the two dosage of blood products, IV fluids, In January 2021, India
Oxford-AstraZeneca vaccine the vaccine is 84 days. vaccines, and sera in India. launched the Vaccine Maitri
that manufacturers in India 3. c. Central Drugs Standard 4. a. Mansukh Mandaviya (Vaccine Friendship) initiative
produce locally. Control Organization Mansukh Laxmanbhai – a major diplomatic effort to
2. d. All of the above Central Drugs Standard Mandaviya is an Indian gift and supply made-in-India
Covishield is technically Control Organization of politician currently serving vaccines to low-income and
referred to as AZD1222 or the Government of India is as the Minister of Health & developing countries globally.
Bhopal +91-7676564400 Indore +91-9589613810, +91-731-4987379 Prayagraj +91-8114000926, +91-8114000927 Kanpur +91-8576066660, +91-8707605589 E: support@toprankers.com
Gurugram +91-8448444207 Delhi +91-9810610466 Ranchi +91-9334969993 Lucknow +91-6390576666 Jabalpur +91-7004386936 W: www.toprankers.com
6 The CLAT Post • September 2021

Passage 6 states to take up the idea of with public cooperation... All month is observed as

U nion Minister for Women Nutrition Month. aspirational districts of the ‘Poshan Maah’ in India?
and Child Development [1] Addressing the press conference, country should be committed a. August
on Tuesday urged all aspirational [1] said, “According to a 2010 to establishing a [2]’ during the
b. September
districts of the country to make World Bank report, India nutrition month.
a commitment to establish a [2] suffered an economic loss of Rs c. October
1. Which of the following d. March
during the nutrition month for 24,000 crore due to lack of toilets.
replaces [1] in the passage?
the protection of Severe Actual According to the study of the
Malnourished (SAM) children. year 2018, the GDP suffered a a. Menaka Gandhi 3. Which of the following
decline of four per cent due to replaces [2] in the passage?
While inaugurating Poshan 2.0, b. Smriti Irani
National Conference, [1] said malnutrition, but since 2018, a. Poshan Vatika
c. Uma Bharti
that Prime Minister Narendra even in the midst of the Covid-19 b. Poshan Kendra
epidemic, a mass movement is d. Sadhvi Pragya
Modi wants the empowerment c. Tatva Poshan
of women in all fields and urged going on against malnutrition. 2. Which of the following
Carrying out 16 crore activities d. Nutrition Nurseries

4. Indian government
launched ‘National
Nutrition Mission’ on the
occasion of which of the
following?
a. World Food Day
b. World Nutrition Day
c. International Women’s
Day
d. Universal Children’s
Day

5. Which of the following


condition is/are considered
as ‘Severe Acute
Malnutrition’ as per the
definition by WHO?
a. Low weight-for-height
b. Mid-upper arm
circumference less than
115 mm
c. By the presence of
nutritional oedema
d. All of the above

1. b. Smriti Irani mothers. (SAM) children. (WHO) defines ‘severe acute


Smriti Irani is the Minister 3. a. Poshan Vatika malnutrition’ (SAM) by very
of Women and Child Union Minister for Women 4. c. International Women’s low weight-for-height or a
Development in the Union and Child Development Day mid-upper arm circumference
Cabinet of India since May Smriti Irani on Tuesday POSHAN Abhiyaan, also less than 115 mm, or by
2019. urged all aspirational districts called National Nutrition the presence of nutritional
2. b. September of the country to make a Mission, was launched by the oedema.
Month of September is commitment to establish government on the occasion of Nutritional oedema: Abnormal
celebrated as POSHAN a Poshan Vatika (nutrition the International Women’s Day fluid retention in the tissues
Maah since 2018 to improve garden) during the nutrition on 8th March, 2018. (oedema) resulting especially
nutritional outcomes for month starting from 5. d. All of the above from lack of protein in states
children, adolescent girls, September 1 for the protection WHO’s Definition: The of starvation or malnutrition.
pregnant women, and lactating of Severe Actual Malnourished World Health Organisation
Bhopal +91-7676564400 Indore +91-9589613810, +91-731-4987379 Prayagraj +91-8114000926, +91-8114000927 Kanpur +91-8576066660, +91-8707605589 E: support@toprankers.com
Gurugram +91-8448444207 Delhi +91-9810610466 Ranchi +91-9334969993 Lucknow +91-6390576666 Jabalpur +91-7004386936 W: www.toprankers.com
7 The CLAT Post • September 2021

Passage 7

T he UAE and India


commenced
their discussions on a
Comprehensive Economic
Partnership Agreement with
Dr. Thani Al Zeyoudi, Minister
of State for Foreign Trade,
meeting Piyush Goyal, India’s
Minister of Commerce and
Industry. The meeting focused
on how the two nations can
accelerate bilateral trade,
investment and economic
opportunity. hoped will broaden economic statements is not true b. 1971
and investment opportunities regarding India-UAE c. 1978
Dr. Al Zeyoudi also held talks
and push bilateral trade beyond relations?
with Jyotiraditya Scindia, d. 1980
$100 billion in five years. a. The UAE is currently
Minister of Civil Aviation,
[1], Minister of Electronics India’s third-largest 5. Which of the following
1. Which of the following
and Information Technology, trading partner with statements is/are true?
is replaced by [1] in the
and Nirmala Sitharaman, passage? bilateral trade in a. Originally the UAE was
Minister of Finance, where the 2019/2020. conceived as a 7-state
a. Ashwini Vaishnaw
potential for deepening ties in b. The UAE is also India’s federation including
investment, technology and b. Narayan Tatu Rane second-largest export Bahrain and Qatar.
aviation were discussed. c. Pralhad Joshi destination after the b. In 1971, only six
The UAE delegation arrived d. Narendra Singh Tomar China. emirates namely Abu
in India on September 21 for c. The UAE is the eighth- Dhabi, Dubai, Kuwait,
a four-day visit to the Indian 2. India has not signed CEPA Ajman, Umm Al
largest investor in India.
capital, New Delhi, as the two agreement with which of Quwain, and Fujairah
the following countries? d. Investment by Indian
sides seek ways to strengthen had joined the union.
companies in the UAE
their economic ties through a. South Korea
is estimated to be over c. Before the UAE was
a Comprehensive Economic b. Canada USD 85 billion. formed in 1971, it was
Partnership Agreement
c. Mauritius known as the Trucial
(CEPA). The talks mark the 4. In which of the following
beginning of a new era of d. Seychelles States.
years was UAE formed?
strategic cooperation between d. All of the above
the two countries, which it is 3. Which of the following a. 1960

1. a. Ashwini Vaishnaw India-UAE Economic 4. b. 1971 by the Sheikhs with the British
Ashwini Vaishnaw: Minister of Relations: Originally the UAE was Government. The origin of
Electronics and Information • The UAE is currently conceived as a 9-state the name Trucial is from
Technology India’s third-largest trading federation including Bahrain these truces. These truces
Narayan Tatu Rane: Minister partner with bilateral trade in and Qatar. However, in 1971 remained in effect until the
of Micro, Small and Medium 2019/2020 valued at USD 59 only six emirates namely British withdraw permanently
Enterprises billion. Abu Dhabi, Dubai, Sharjah, in 1971. Originally the UAE
Pralhad Joshi: Minister • The UAE is also India’s Ajman, Umm Al Quwain, and was conceived as a 9-state
of Parliamentary Affairs; second-largest export Fujairah had joined the union. federation including Bahrain
Minister of Coal; and Minister destination after the US, Ras al Khaimah joined them and Qatar. However, in 1971
of Mines with exports valued at on the 10th Feb 1972 making only six emirates namely
Narendra Singh Tomar: approximately USD 29 billion it a 7-state federation and the Abu Dhabi, Dubai, Sharjah,
Minister of Agriculture and in 2019-2020. modern UAE was born. Ajman, Umm Al Quwain, and
Farmers Welfare • The UAE is the eighth- 5. c. Before the UAE was Fujairah had joined the union.
2. d. Seychelles largest investor in India, formed in 1971, it was known Ras al Khaimah joined them
India has signed CEPA deal having invested USD 11 billion as the Trucial States. on the 10th Feb 1972 making
with South Korea, Japan and between April 2000 and March Before the UAE was formed it a 7-state federation and the
Canada. 2021, while investment by in 1971, it was known as the modern UAE was born.
3. b. The UAE is also India’s Indian companies in the UAE Trucial States. Between 1820
second-largest export is estimated to be over USD 85 and 1892 several protective
destination after the China. billion. treaties or truces were signed
Bhopal +91-7676564400 Indore +91-9589613810, +91-731-4987379 Prayagraj +91-8114000926, +91-8114000927 Kanpur +91-8576066660, +91-8707605589 E: support@toprankers.com
Gurugram +91-8448444207 Delhi +91-9810610466 Ranchi +91-9334969993 Lucknow +91-6390576666 Jabalpur +91-7004386936 W: www.toprankers.com
8 The CLAT Post • September 2021

Passage 8

I ndia has stood among the


top-20 generous countries,
according to the CAF World
Giving Index 2021. India has
bagged the [1] spot and is up
from the 10-year global rank
of 82. “India is one of the
fastest climbers on the Index, it
ranks [1] this year, in the 10th
Anniversary report, India ranked
82,” the [2] latest survey said. As
per the report, 61% more Indians
have helped a stranger during the
2020 Covid-19 pandemic year.
Last year, 36% of Indians donated
money, 34% volunteered for
social causes in the country.
Communities around the world
mobilised to help fellow citizens
during the pandemic, resulting
in the highest ‘helped a stranger’
figures since 2009. More than
half (55%) of the world’s adults
– or 3 billion people - reported
a. Charities Aid regarding CAF? Kenya, Nigeria, and
helping someone they didn’t
know in 2020, according to the Foundation a. It was established in Ghana found place
survey. Besides, more people b. Charities Assistance 1974. among top-10 generous
donated money in 2020 than Foundation countries in CAF 2021
b. It was founded by
had done so in the last five years index.
c. Charities Aid Fund National Council of
(31%). Levels of volunteering in Social Service. b. Developed countries
d. Charities Assistance
2020 are broadly unchanged at such as the USA, the
Fund c. CAF are responsible for
the global level, it added. UK, Canada, Ireland,
the UK arm of Giving
3. Which of the following and the Netherlands
1. Which of the following Tuesday, the global day
countries topped the World were out of the highest
replaces [1] in the passage? of giving.
Giving Index 2021 list? ranking.
a. 10th d. CAF is a subsidiary of
a. Kenya c. 55% of the world’s
British Royal Family
b. 12th adults (that is 3 billion
b. Ghana charities.
c. 14th people) were reported
c. Indonesia 5. Which of the following helping someone they
d. 18th statements is/are true
d. Australia didn’t know in 2020.
regarding the World Giving
2. Which of the following d. All of the above.
4. Which of the following Index 2021?
replaces [2] from the
statements is not true a. Poor countries like
passage?

1. c. 14th Indonesia has been ranked Chartered Aid Foundation index. Developed countries
India has bagged the 14th spot at first position in the CAF was established in 1974. It was such as the USA, the UK,
and is up from the 10-year Index this year. Indonesia is founded by National Council Canada, Ireland, and the
global rank of 82. followed by Kenya, Nigeria, of Social Service. CAF are Netherlands were out of the
2. a. Charities Aid Foundation Myanmar, Australia, Ghana, responsible for the UK arm of highest ranking. 55% of the
The Charities Aid Foundation New Zealand, Uganda, Kosovo Giving Tuesday, the global day world’s adults (that is 3 billion
(CAF) is a registered UK and Thailand in that order. In of giving. people) were reported helping
charity. CAF released World the year 2019, US was ranked 5. d. All of the above. someone they didn’t know in
Giving Index annually. at top position. Poor countries like Kenya, 2020.
3. c. Indonesia 4. d. CAF is a subsidiary Nigeria, and Ghana found
The most generous country of British Royal Family place among top-10 generous
across the world is Indonesia. charities. countries in CAF 2021
Bhopal +91-7676564400 Indore +91-9589613810, +91-731-4987379 Prayagraj +91-8114000926, +91-8114000927 Kanpur +91-8576066660, +91-8707605589 E: support@toprankers.com
Gurugram +91-8448444207 Delhi +91-9810610466 Ranchi +91-9334969993 Lucknow +91-6390576666 Jabalpur +91-7004386936 W: www.toprankers.com
9 The CLAT Post • September 2021

is replaced by [2] in the


passage?
a. Australia
b. Argentina
c. Germany
d. Italy

4. Which of the following


statements regarding G20
is/are true?
a. It is an informal group
of 19 countries and the
European Union (EU).
b. It also includes
representatives of the
International Monetary
Fund and the World
Bank.
c. It is headquartered
in London, United
Kingdom.
d. Both a & b.
Passage 9 transforming India from a net c. 2040

I ndia on September 18 stressed food importer to exporter. d. 2025 5. Which of the following
at G20 agriculture meeting “Today with an annual statements is/are true?
that there is a need to increase production of 308 million tonnes 2. Which of the following a. The United Nations
investment in agricultural of food grains, India is not only statements was signed in (UN) has accepted
research and development amid in the realm of food security the meet to strengthen India’s proposal and
the challenges of climate change but is also catering to needs of cooperation between G20 declared 2023 as the
and expected increase in food other countries,” he said and members and developing International Year of
demand by [1]. added India has experienced countries on food and Millet.
The G20 Agriculture Ministers’ a revolution in the field of agriculture?
b. Sustainable
Meeting is one of the ministerial agricultural produce due to the a. Canberra Sustainability
efficient research of scientists. Development Goal 6
meetings organised as part of Charter aims to achieve “zero
the G20 Leaders Summit 2021 1. Which of the following is b. Rosario Sustainability hunger”.
to be hosted by [2] in October. replaced by [1] in the passage? Charter
Addressing a session on the c. Sustainable
1. Which of the following c. Berlin Sustainability Development Goals are
topic ‘Research as a driving force
is replaced by [1] in the Charter legally binding.
behind sustainability’, Agriculture
Minister Narendra Singh Tomar passage? d. Florence Sustainability d. All of the above
shared that agricultural research a. 2030 Charter
has played a major role in b. 2035 3. Which of the following

1. a. 2030 was signed in the meet to 2021 to be hosted by Italy in has accepted India’s proposal
India on September 18 stressed strengthen cooperation October 2021. and declared 2023 as the
at G20 agriculture meeting between G20 members and 4. d. Both a & b. International Year of Millet.
that there is a need to increase developing countries on food G20 is an informal group The United Nations (UN)
investment in agricultural and agriculture. of 19 countries and the has accepted India’s proposal
research and development 3. d. Italy European Union (EU), and declared 2023 as the
amid the challenges of climate Recently, India’s Agriculture with representatives of the International Year of Millet.
change and expected increase Minister addressed the G20 International Monetary Sustainable Development
in food demand by 2030. Agriculture meeting virtually. Fund and the World Bank. It Goal 2 aims to achieve “zero
2. d. Florence Sustainability It is one of the ministerial does not have a permanent hunger”.
Charter meetings organised as part secretariat or Headquarters. Sustainable Development
Florence Sustainability Charter of the G20 Leaders Summit 5. a. The United Nations (UN) Goals are not legally binding.
Bhopal +91-7676564400 Indore +91-9589613810, +91-731-4987379 Prayagraj +91-8114000926, +91-8114000927 Kanpur +91-8576066660, +91-8707605589 E: support@toprankers.com
Gurugram +91-8448444207 Delhi +91-9810610466 Ranchi +91-9334969993 Lucknow +91-6390576666 Jabalpur +91-7004386936 W: www.toprankers.com
10 The CLAT Post • September 2021

Passage 10 Total workforce

G overnment think tank estimated to be 0.64


Niti Aayog on Thursday billion by 2030, of
launched a report on steps which 0.26 billion to
required for reforming urban be employed in urban
planning capacity in the country. areas.
It was released by vice chairman c. Infrastructure Targets:
Rajiv Kumar and CEO [1]. Creation of 11 large
The report titled “Reforms in industrial corridors as
Urban Planning Capacity in part of the National
India”, noted that though urban Industrial Corridor
planning is the key for integrated Programme, several
development of cities, ironically multi-modal logistic
it has not received due attention. parks, etc.
Also the existing urban planning local bodies. a. India’s urban population
and governance framework is d. National Infrastructure
is 11% of that of the
also quite complex that often 1. Which of the following Pipeline (NIP): The
world.
leads to ambiguity and lack of replaces [1] in the passage? urban sector has a
b. Goa, Tamil Nadu, significant share of 17%
accountability. a. Amitabh Kant Kerala, Maharashtra, in the NIP.
Most significantly, the b. Shaktikant Das and Gujarat have
Coronavirus pandemic has attained over 40% 5. Which of the following
c. Ajit Doval
exposed the urgent need for urbanisation. Schemes/Programmes aims
better planning and management d. Vinod Kumar Paul
c. NCT of Delhi, Daman to bring together urban
of Indian cities. The report has planning, economic growth
2. According to Census and Diu, Chandigarh,
made several recommendations and heritage conservation
of India 2011, India’s and Lakshadweep, show
to decongest value chain of in an inclusive manner
population stood at 1210 above 75% urbanisation.
urban planning capacity in the & with the objective of
country. Need for all cities in million in 2011, with an d. All of the above
urbanisation level of which preserving the heritage
the country to become “healthy character of the City?
cities” is an aim which needs to of the following? 4. Which of the following
be achieved by 2030, the report a. 29% statements is not correct a. HRIDAY
elaborated. For this purpose, the regarding India’s National b. AMRUT Mission
b. 31.1%
report suggests a central sector Growth Targets?
c. 42.3% c. Swachh Bharat Mission-
scheme titled “500 Healthy Cities a. Economic Growth Urban
Programme” for a period of five d. 22.5% Target: USD 5 trillion
d. Pradhan Mantri Awas
years where priority cities will be economy by 2030.
3. Which of the following Yojana-Urban
selected jointly by the states and b. Employment Target:
statements is/are true?

1. a. Amitabh Kant Uttar Pradesh continue to be corridors as part of the towns in the country.
Amitabh Kant is the second at a lower level of urbanisation National Industrial Corridor HRIDAY: The National
and current chief executive than the national average of Programme, several multi- Heritage City Development
officer of NITI Aayog. 31.1%. modal logistic parks, etc. and Augmentation Yojana
2. b. 31.1% NCT of Delhi, Daman National Infrastructure (HRIDAY), aims to bring
India’s population stood at and Diu, Chandigarh, and Pipeline (NIP): The urban together urban planning,
1210 million in 2011, with an Lakshadweep, show above 75% sector has a significant share economic growth and heritage
urbanisation level of 31.1% urbanisation. of 17% in the NIP. conservation in an inclusive
(Census of India 2011). 4. a. Economic Growth Target: 5. a. HRIDAY manner & with the objective
Urbanisation is an increase in USD 5 trillion economy by AMRUT Mission: To ensure of preserving the heritage
the number of people living in 2030. that every household has character of the City.
towns and cities. Economic Growth Target: USD access to a tap with the assured Pradhan Mantri Awas Yojana-
3. d. All of the above 5 trillion economy by 2024. supply of water and a sewerage Urban: Addresses Urban
India’s urban population is Employment Target: Total connection. housing shortage among the
11% of that of the world. workforce estimated to be Swachh Bharat Mission-Urban: Urban Poor including the Slum
Goa, Tamil Nadu, Kerala, 0.64 billion by 2030, of which Aims at making urban India Dwellers by ensuring a pucca
Maharashtra, and Gujarat 0.26 billion to be employed in free from open defecation house to eligible urban poor.
have attained over 40% urban areas. and achieving 100% scientific
urbanisation. Infrastructure Targets: management of municipal
Bihar, Odisha, Assam, and Creation of 11 large industrial solid waste in 4,041 statutory
Bhopal +91-7676564400 Indore +91-9589613810, +91-731-4987379 Prayagraj +91-8114000926, +91-8114000927 Kanpur +91-8576066660, +91-8707605589 E: support@toprankers.com
Gurugram +91-8448444207 Delhi +91-9810610466 Ranchi +91-9334969993 Lucknow +91-6390576666 Jabalpur +91-7004386936 W: www.toprankers.com
11 The CLAT Post • September 2021

Passage 11

I ndia has jumped two spots


and has been ranked [1] by
the World Intellectual Property
Organisation in the Global
Innovation Index 2021 rankings.
The country has been on an
upward trajectory over the
past few years in the Global
Innovation Index (GII). India has
shot up from a rank of 81 in 2015
to [1] in 2021.
“Innovation has been at the
forefront of our battle against the
unprecedented crisis created by
the pandemic, and will be pivotal
in driving the country’s resilience b. 46th among the 34 lower 4. India Innovation Index
and self-reliance, as enshrined in c. 44th middle-income group is developed by the lines
the Prime Ministers’ clarion call economies. of GII, by which of the
d. 42nd
on Atma Nirbhar Bharat,” noted b. India ranks 1st among following?
the NITI Aayog in an official 2. WIPO is an agency working the 10 economies in a. Ministry of Finance
statement. under which of the Central and Southern b. Niti Aagoy
The public policy think tank following? Asia.
c. Ministry of Earth
further added that the consistent a. United Nations c. The government
improvement in India’s GII Sciences
b. World Bank attributed the country’s
standing can be attributed to d. Department of Science
improved performance
immense knowledge capital, c. International Monetary and Technology
to the departments
the vibrant start-up ecosystem, Fund 5. Which of the following
of atomic energy,
and the amazing work done by d. BRICS Countries is not in the top
science and technology,
the public and private research 5 of the Index?
3. Consider the following biotechnology and
organisations.
space. a. Switzerland
about India’s performance
1. Which of the following will in the Global Innovation d. India has been on an b. USA
replace [1] in the above Index and choose the upward trajectory over c. China
passage? incorrect? the past few years in the d. South Korea
a. 48th a. India ranks 2nd GII except 2019.

1. b. 46th India has been on an upward The government attributed It also adds parameters that are
Recently, India has climbed trajectory over the past few the country’s improved specific to the Indian economy
two spots and has been ranked years in the GII. performance to the (eg. Demographic dividend),
46th in the Global Innovation India has shot up from a rank departments of atomic energy, to give it a more holistic
Index (GII) 2021 rankings. of 81 in 2015 to 46 in 2021. science and technology, coverage.
2. a. United Nations India performs better in biotechnology and space. 5. c. China
The GII is launched by the innovation outputs than 4. b. Niti Aagoy Top Five: Switzerland, Sweden,
World Intellectual Property innovation inputs in 2021. The India Innovation Index U.S., and U.K. continue to lead
Organization (WIPO), a This year India ranks 57th in has been developed by NITI the innovation ranking, and
specialized agency of the innovation inputs, the same as (National Institution for have all ranked in the top 5 in
United Nations. last year but higher than 2019. Transforming India) Aayog on the past three years.
The GII aims to capture the As for innovation outputs, the lines of the GII. The Republic of Korea joins
multi-dimensional facets of India ranks 45th. This position The index goes beyond the top 5 of the GII for the first
innovation ranking and rich is the same as last year but traditional approaches time in 2021.
analysis referencing around higher than 2019. by considering the best Asian Countries: Four Asian
132 economies. India ranks 2nd among the 34 parameters in measuring economies feature in the top
3. d. India has been on an lower middle-income group innovation such as patents 15: Singapore (8), China (12),
upward trajectory over the economies. per million of population, Japan (13) and Hong Kong,
past few years in the GII India ranks 1st among the publication in scientific China (14).
except 2019. 10 economies in Central and journals, percentage of GDP
India’s Performance: Southern Asia. spending on research.
Bhopal +91-7676564400 Indore +91-9589613810, +91-731-4987379 Prayagraj +91-8114000926, +91-8114000927 Kanpur +91-8576066660, +91-8707605589 E: support@toprankers.com
Gurugram +91-8448444207 Delhi +91-9810610466 Ranchi +91-9334969993 Lucknow +91-6390576666 Jabalpur +91-7004386936 W: www.toprankers.com
12 The CLAT Post • September 2021

the regulatory issues.


d. India doesn’t has any
CEPAs any country till
now.

3. Which of the following is


TRUE?
a. The UAE is India’s
third-largest trading
partner.
b. Its second-largest export
destination after the US.
c. The UAE is also the
eighth-largest investor
in India, having invested
Passage 12 talks aimed at improving bilateral 2. Which of the following is around $11 billion

I ndia and the United Arab economic relations, including not true? between April 2000 and
Emirates (UAE) have officially the expanding of existing trade a. CEPA is free trade March 2021.
launched negotiations for a free and investment relationships. pact which covers d. All of the above
trade agreement, the first round According to an official negotiation on the
of talks for which are set to statement, both the ministers 4. The UAE comprises of how
trade in services and
begin on Thursday. According to expressed a desire to reach a many emirates?
investment, and other
Union minister of commerce and mutually beneficial economic areas of economic a. 5
industry [1], the two countries deal, building upon the progress
partnership. b. 6
are aiming to conclude the trade made by both India and the
UAE under the Comprehensive b. It may even consider c. 7
talks by the end of the year
Strategic Partnership signed in negotiation on d. 8
and sign a mutually beneficial
[2]. areas such as trade
Comprehensive Economic
facilitation and 5. Which of the following will
Partnership Agreement
1. Which of the following will customs cooperation, replace [2] in the above
(CEPA) by March 2022, after
replace [1] in the above competition, and passage?
the completion of the legal
passage? Intellectual Property a. 2015
paperwork and ratification.
a. Piyush Goyal Rights.
Union minister [1] met Thani b. 2016
bin Ahmed Al Zeyoudi, the UAE b. Arjun Munda c. CEPA also looks into
c. 2017
minister for foreign trade, in this c. Dharmendra Pradhan the regulatory aspect of
trade and encompasses d. 2018
regard on Wednesday and held d. Sarbananda Sonowal an agreement covering

1. a. Piyush Goyal competition, and Intellectual year. India’s major exports to The UAE is a constitutional
Shri Piyush Goyal is the Property Rights. the UAE include petroleum federation. On 2 December
Minister of Commerce Partnership agreements or products, precious metals, 1971, the United Arab
and Industry; Minister of cooperation agreements are stones, gems and jewellery, Emirates (UAE) was declared
Consumer Affairs, Food and more comprehensive than Free minerals, food items such as an independent, sovereign
Public Trade Agreements. as cereals, sugar, fruits and federal state. The UAE
Distribution; and Minister of CEPA also looks into the and vegetables, tea, meat, comprises seven emirates: Abu
Textiles of India. regulatory aspect of trade and seafood, textiles, engineering, Dhabi, Dubai, Sharjah, Ras
2. d. India doesn’t has any encompasses an agreement machinery products, and Al Khaimah, Ajman, Umm Al
CEPAs any country till now. covering the regulatory issues. chemicals. Quwain and Fujairah.
Comprehensive Economic India has signed CEPAs with The UAE is also the eighth- 5. c. 2017
Partnership Agreement: South Korea and Japan. largest investor in India, Looking to build on the
It is a kind of free trade pact 3. d. All of the above having invested around $11 progress made by both
which covers negotiation Notably, the UAE is India’s billion between April 2000 countries under the
on the trade in services and third-largest trading partner and March 2021, while the Comprehensive Strategic
investment, and other areas of and also its second-largest investment made by Indian Partnership signed in 2017,
economic partnership. It may export destination after the companies in the UAE is both countries expressed a
even consider negotiation on US, with exports valued at estimated to be over $85 desire to reach a mutually
areas such as trade facilitation approximately $29 billion billion. beneficial economic deal.
and customs cooperation, in the 2019-2020 financial 4. c. 7
Bhopal +91-7676564400 Indore +91-9589613810, +91-731-4987379 Prayagraj +91-8114000926, +91-8114000927 Kanpur +91-8576066660, +91-8707605589 E: support@toprankers.com
Gurugram +91-8448444207 Delhi +91-9810610466 Ranchi +91-9334969993 Lucknow +91-6390576666 Jabalpur +91-7004386936 W: www.toprankers.com
13 The CLAT Post • September 2021

Passage 13 entity dedicated to

T here was a [1] per cent rise gender equality and


in complaints of crimes the empowerment of
against women in the first women.
eight months of 2021 over the b. The European Union
corresponding period of last and the UN have
year, the National Commission collectively taken an
for Women (NCW) said. initiative called ‘The
NCW chief Rekha Sharma Spotlight Initiative’.
said there has been a rise in
c. National Commission
the complaints because the
for Women’s mission
commission has been regularly
is to strive towards
conducting awareness programs
enabling women to
due to which the public is now
achieve equality and
more aware about its work. The
NCW received a total of 19,953 equal participation in all
complaints of crimes against spheres of life.
women from January to August received regarding the offence b. Haryana d. National Commission
this year, up from 13,618 in the of outraging modesty of women c. Uttar Pradesh for Women (NCW) was
corresponding period of 2020, or molestation, followed by set up as a statutory
d. Rajasthan
it said. 1,022 complaints of rape body in January 2013
and attempt to rape and 585 3. On which of the following under The Protection of
In July, as many as 3,248 complaints of cyber-crimes.
complaints were received days is the International Children from Sexual
by the NCW which is the Source: Extracted with edits Day for the Elimination of Offences (POCSO),
highest in a month since and revisions from news article Violence against Women 2012.
June 2015, it added. Of the of The Indian Express. observed?
19,953 complaints, the highest 5. Which of the following is/
1. Which of the following a. 11th November
number of 7,036 were recorded are functions of National
replaces [1] in the passage? b. 21st November Commission for Women?
under the right to live with
a. 46 c. 25th November a. Review the
dignity clause, followed by
4,289 complaints of domestic b. 62 d. 7th December constitutional and legal
violence and 2,923 complaints c. 56 safeguards for women.
of harassment of married 4. Which of the following
d. 48 statements is not true b. Recommend remedial
women or dowry harassment, legislative measures.
the NCW said. The right to regarding the initiatives
2. Which of the following c. Facilitate redressal of
live with dignity clause takes taken against the Violence
states has topped the list in grievances.
into account the emotional against Women?
highest number of crimes
abuse of women. As many as a. UN Women is the d. All of the above.
in India?
1,116 complaints have been United Nations
a. Delhi

1. a. 46 Elimination of Violence to this issue, moving it into National Commission for


There was a 46 per cent against Women-25th the spotlight and placing it at Women was set up as a
rise in complaints of crimes November. the centre of efforts to achieve statutory body in January
against women in the first 4. d. National Commission for gender equality and women’s 1992 under the National
eight months of 2021 over the Women (NCW) was set up as empowerment. UN Women Commission for Women Act,
corresponding period of last a statutory body in January is the United Nations entity 1990. Its functions are to:
year. 2013 under The Protection dedicated to gender equality • Review the constitutional and
of Children from Sexual and the empowerment of legal safeguards for women.
2. c. Uttar Pradesh Offences (POCSO), 2012. women. • Recommend remedial
Among states and union The Spotlight Initiative: The NCW was set up as a statutory legislative measures.
territories, the highest European Union (EU) and body in January 1992 under • Facilitate redressal of
number of complaints was the United Nations (UN) have the National Commission for grievances.
received from Uttar Pradesh embarked on a new, global, Women Act, 1990. Its mission • Advise the Government on
(10,084), followed by Delhi multi-year initiative focused is to strive towards enabling all policy matters affecting
(2,147), Haryana (995) and on eliminating all forms of women to achieve equality women
Maharashtra (974). Violence Against Women and and equal participation in all
3. c. 25th November Girls (VAWG). It is so named spheres of life.
International Day for the as it brings focused attention 5. d. All of the above.
Bhopal +91-7676564400 Indore +91-9589613810, +91-731-4987379 Prayagraj +91-8114000926, +91-8114000927 Kanpur +91-8576066660, +91-8707605589 E: support@toprankers.com
Gurugram +91-8448444207 Delhi +91-9810610466 Ranchi +91-9334969993 Lucknow +91-6390576666 Jabalpur +91-7004386936 W: www.toprankers.com
14 The CLAT Post • September 2021

Passage 14

I n order to constantly
monitor implementation
of city-specific action plans
for improving air quality in
132 non-attainment cities, the
Centre on Tuesday launched
a dedicated portal that will
support tracking of physical as
well as financial status of the
plan under the National Clean
Air Programme (NCAP).
The programme was launched
in January, 2019 to improve air
quality through a city specific
action plan in all such cities
which do not meet the National
Ambient Air Quality Standards
(NAAQS). Under the NCAP,
the Centre and states together
would work to reduce the level
of particulate matter (PM10
and PM2.5) by 20-30% from
the 2017 (base year) level by
[1].
“Eighty-six cities showed
better air quality in 2019 in c. 2025 Air Programme (NCAP) c. 112
comparison to 2018 and this d. 2028 in Non-Attainment Cities d. 118
number increased to [2] cities (NAC)?
in 2020,” said environment 2. When is the International a. Prerna Portal 5. Who among the following
minister Bhupender Yadav at Day of Clean Air for Blue is the Minister responsible
b. Breath Portal
an event to mark the second Skies observed? for the Ministry of
c. Prana Portal Environment, Forest and
International Day of Clean Air a. 7th September
for Blue Skies. d. Safar Portal Climate Change?
b. 8th September
a. Bhupendra Yadav
1. Which of the following will c. 9th September 4. Which of the following will
replace [1] in the above replace [2] in the above b. Ashwini Kumar Chobey
d. 12th September
passage? passage? c. Prakash Javadekar
3. Which of the following a. 104 d. Gajendra Singh
a. 2022
portals has been launched Shekhawat
b. 2024 b. 110
under the National Clean

1. b. 2024 developmental challenges and It will support tracking of PM 2.5 levels: 11 cities showed
It is targeted to achieve 20- raise public awareness related physical as well as financial a decreasing trend, 79 cities a
30% reduction in particulate to air quality at all levels. status of city air action fluctuating trend and 9 cities
matter (PM10 and PM2.5) 3. c. Prana Portal plan implementation, and an increasing trend.
concentrations across the Prana Portal: It was launched disseminate information on air 5. a. Bhupendra Yadav
country by 2024. under the National Clean quality to the public. The Ministry of Environment,
2. a. 7th September Air Programme (NCAP) in 4. a. 104 Forest and Climate Change
In December 2019, the United Non-Attainment Cities (NAC), Cities with improved air is an Indian government
Nations General Assembly cities which didn’t meet quality had increased to 104 in ministry. The ministry
adopted a resolution to Ambient Air Quality Standards 2020 from 86 in 2018. portfolio is currently held
observe this day on 7th under NCAP. From 2015-2019: by Bhupender Yadav, Union
September every year, starting It is targeted to achieve 20- Particulate Matter (PM) 10 Minister of Environment,
from 2020. 30% reduction in particulate levels: 23 cities marking a Forest and Climate Change.
The Day aims to demonstrate matter (PM10 and PM2.5) “decreasing trend”, 239 cities a
the close link of air quality concentrations across the “fluctuating trend” & 38 cities
to other environmental/ country by 2024. an “increasing trend”.
Bhopal +91-7676564400 Indore +91-9589613810, +91-731-4987379 Prayagraj +91-8114000926, +91-8114000927 Kanpur +91-8576066660, +91-8707605589 E: support@toprankers.com
Gurugram +91-8448444207 Delhi +91-9810610466 Ranchi +91-9334969993 Lucknow +91-6390576666 Jabalpur +91-7004386936 W: www.toprankers.com
15 The CLAT Post • September 2021

Passage 15 touted as the site for a

C hina is unleashing its buying Chinese base suitable


power to lure Myanmar, for People’s Liberation
Singapore and the rest of the Army Navy (PLAN)
Association of Southeast Asian operations.
Nations (Asean) to its side within c. Gwadar helps anchor
two weeks of inaugurating rail or stabilise western
and road linkages with Myanmar. China, a region where
“China will import more China feels vulnerable
connects China to Myanmar’s Yunnan (China) directly
distinctive products from Asean, to Islamic agitation.
[1] and gives it easy access to the to the port.
expand mutual investment, d. Gwadar is just 400 km
market in Southeast Asia. The b. China plans to develop
deepen the industrial supply from the important
first cargo train brought goods this region in Myanmar
chain and promote Lancang- Malacca Strait.
from Singapore to [1]. as a ‘border economic
Mekong cooperation,” Chinese
cooperation zone’ under 4. This trade route is also
vice-president Wang Qishan 1. Which of the following will
said, referring to the economic the Belt and Road China’s alternative to
replace [1] in the above the “Malacca Dilemma”.
corridor with Southeast Asia. He passage? Initiative.
was speaking at the China-Asean Malacca Dilemma is a word
a. Yangon port c. This trade corridor is
Business and Investment Summit coined by who among the
another direct Chinese
in Nanning city. b. Dalian port following?
outlet to the Indian
The first cargo train using the c. Yingkou port ocean. The first one a. Hu Jintao
new rail link carried goods from d. Tianjin port being at the Gwadar b. Jiang Zemin
Myanmar that reached Chengdu port in Pakistan. c. Wen Jiabao
city in China on August 27. This 2. Which of the following is
d. All of the above. d. Xi Jinping
is the first time that China has true about the new trade
established a direct link to the corridor talked about in the 3. Which of the following is 5. Belt and Road initiative was
Indian Ocean through Myanmar. passage? not true about Gwadar started by China in which
For India, the rail and road a. China also has plans to port? of the following years?
linkage through Myanmar is. develop another port a. Gwadar is being a. 2009
both a strategic and a trade in Kyaukphyu in the developed as part of the
challenge. Trains make it easy Rakhine state, Myanmar b. 2011
CPEC to the far western
to transport extremely heavy including a proposed Xinjiang region. c. 2013
military equipment. The new link railway line from d. 2015
b. Gwadar has long been

1. a. Yangon port Kyaukphyu in the Rakhine Gwadar is just 400 km from of a maritime blockade at
Recently, a new sea-road- state, Myanmar including a the important Hormuz Strait the Straits of Malacca. Since
rail link providing Chengdu proposed railway line from (linking Persian Gulf with the most of China’s oil imports
(China) with access to the Yunnan (China) directly to the Gulf of Oman and the Arabian pass through the Straits of
Indian Ocean via Yangon port, but the progress there sea), through which 40% of Malacca, a maritime blockade
(Myanmar) was opened. The has been stalled by unrest in Chinese imported oil flows. here could paralyze China’s
trade corridor is China’s first Myanmar. The Strait of Malacca or Straits economy.
to link western China with China plans to develop this of Malacca is a narrow stretch 5. c. 2013
the Indian Ocean. The Port region in Myanmar as a of water, 580 mi in length, The Belt and Road Initiative,
of Yangon is the main port ‘border economic cooperation between the Malay Peninsula reminiscent of the Silk Road,
of Myanmar that handles zone’ under the Belt and Road and the Indonesian island of is a massive infrastructure
approximately 90% of import Initiative. Sumatra. As the main shipping project that would stretch from
and export activities in the It is expected to become the channel between the Indian East Asia to Europe. It was
country. lifeblood of international trade Ocean and the Pacific Ocean, launched in 2013.
2. d. All of the above. for China, while providing a it is one of the most important The plan is two-pronged:
The new trade corridor source of income for Myanmar. shipping lanes in the world. the overland Silk Road
passage connects the logistics This trade corridor is another 4. a. Hu Jintao Economic Belt and the
lines of Singapore, Myanmar direct Chinese outlet to the This trade route is also China’s Maritime Silk Road-The two
and China, and is currently Indian ocean. alternative to the “Malacca were collectively referred to
the most convenient land and The first one being at the Dilemma”. first as the One Belt, One
sea channel linking the Indian Gwadar port in Pakistan. Malacca Dilemma is a word Road initiative but eventually
Ocean with southwest China. 3. d. Gwadar is just 400 km coined in 2003, by the then became the Belt and Road
China also has plans to from the important Malacca Chinese President Hu Jintao. Initiative.
develop another port in Strait. This refers to China’s fear
Bhopal +91-7676564400 Indore +91-9589613810, +91-731-4987379 Prayagraj +91-8114000926, +91-8114000927 Kanpur +91-8576066660, +91-8707605589 E: support@toprankers.com
Gurugram +91-8448444207 Delhi +91-9810610466 Ranchi +91-9334969993 Lucknow +91-6390576666 Jabalpur +91-7004386936 W: www.toprankers.com
16 The CLAT Post • September 2021

Security Council has


the power to make
decisions that member
states are then obligated
to implement under the
Charter.
d. The council is
headquartered at
Washington DC.

4. India is currently serving


which term at the UNSC?
a. 6th term
b. 7th term
c. 8th term
d. 9th term

5. Consider the following


about Taliban, and choose
Passage 16 shelter or train terrorists or to a. USA
the incorrect answer?
W ith 17 out of 33 ministers plan or to finance terrorist acts. b. France
While reiterating the importance a. The Taliban,
in the Taliban cabinet c. Russia
on the sanctions list of the UN of combating terrorism in emerged in the early
Afghanistan, the resolution d. UK 1990s in northern
Security Council’s (UNSC) 1267
Committee, the Sunni Islamist also calls to take action against Pakistan following
3. Which of the following the withdrawal of
government will have to seek those individuals and entities
designated pursuant to resolution is True about the United Soviet troops from
delisting of these individuals Nations Security Council?
based on the latest [1] UNSC 1267 (1999). Afghanistan.
resolution on Afghanistan. The a. Article 23 of the UN b. It is an Islamic
1. Which of the following will Charter concerns the
1267 Committee is headed by fundamentalist
replace [1] in the above composition of the
India till December 2022. political and military
passage? UNSC.
Resolution [1] was passed by the organisation operating
UNSC under the presidency of a. 2593 b. The UNSC has in Afghanistan.
India on August 30, 2021 with b. 2596 been given primary c. They seek to reimpose
Russia and China abstaining c. 2598 responsibility their strict version of
from voting. for maintaining Islam in Afghanistan.
d. 2599
Paragraph two of resolution [1], international peace
and security and may d. Afghanistan was
which has become a mantra for 2. Which of the following admitted into SAARC
the democratic world, clearly Countries have not voted meet whenever peace is
threatened. as the eighth member in
demands that Afghan territory in the favour of the 2012.
should not be used to threaten Resolution? c. Among all the organs
or attack any country or to of the UN, only the

1. a. 2593 including India, voting in All the options are correct as a non-permanent member
Recently, the India-led United favour, none against it. except option d) as the council of the UNSC in January 2021.
Nations Security Council Two permanent and veto- is headquartered at New York. This is India’s eighth term on
(UNSC) adopted a Resolution wielding members Russia and 4. c. 8th term the UNSC.
2593 on Taliban. China abstained. India assumed the presidency 5. d. Afghanistan was
The resolution, sponsored by The adoption of the resolution of the United Nations Security admitted into SAARC as the
France, UK and the US, was is a strong signal from the Council (UNSC) for the month eighth member in 2012.
adopted with 13 members, Security Council and the of August 2021. All the options are correct
including India, voting in international community on This will be the country’s first except option d), Afghanistan
favour, none against it. its expectations in respect of presidency during its 2021-22 was admitted into the SAARC
2. c. Russia Afghanistan. tenure as a non-permanent as the eighth member in 2007.
The resolution, sponsored by 3. d. The council is member of the Security Therefore, as the statement
France, UK and the US, was headquartered at Washington Council. given in option d. is incorrect,
adopted with 13 members, DC. India began its two-year tenure it is the correct answer.
Bhopal +91-7676564400 Indore +91-9589613810, +91-731-4987379 Prayagraj +91-8114000926, +91-8114000927 Kanpur +91-8576066660, +91-8707605589 E: support@toprankers.com
Gurugram +91-8448444207 Delhi +91-9810610466 Ranchi +91-9334969993 Lucknow +91-6390576666 Jabalpur +91-7004386936 W: www.toprankers.com
17 The CLAT Post • September 2021

Passage 17 ULFA, the smaller insurgencies Dimasa, Bodo, Kuki, an Autonomous District

T he tripartite agreement have been no less disruptive of Hmar, Tiwa, Garo, Man Council under which of the
signed by the Centre, five the state-building process in the (Tai speakers), Rengma following Schedule?
insurgent groups active in region. Naga. a. Fifth Schedule
Karbi Anglong, and the [1], 1. Which of the following will b. Its diversity also b. Sixth Schedule
marks the culmination of an replace [1] in the above generated different
extended process of negotiation c. Eighth Schedule
passage? outfits and fuelled an
to end insurgency in the region. insurgency that did d. Ninth Schedule
a. Nagaland Government
According to the Memorandum not allow the region to
of Settlement, greater autonomy b. Odisha Government 4. Which of the following was
develop. not one of the demand of
will be devolved to the Karbi c. Assam Government
c. The history of the Karbi the insurgent groups?
Anglong Autonomous d. Manipur Government group has been marked a. Inclusion of some areas
Council; the identity, language,
culture of Karbi people will be into Karbi Anglong
protected; and more focused Autonomous Council
development carried out in the (KAAC).
Council area. The government b. Reservation of seats for
will also consider notifying Scheduled Tribes.
Karbi language as the official
language of the Council. Over a c. Inclusion of Karbi
thousand armed insurgents have language in the Eighth
surrendered their arms under the Schedule.
peace deal. d. Financial package of Rs
The Northeast’s map is 9,000 crore.
dotted with big and small
5. Which of the following is
insurgent groups that have
not one of the four North-
made demands ranging from
eastern State mentioned
a separate nation-state to
under 6th Schedule for
statehood within the Indian
administration of their
Constitution and autonomy
tribal areas?
under the state government. 2. Which of the following
The Naga insurgency has is TRUE about the Karbi by killings, ethnic a. Assam
been an inspiration for these Crisis? violence, abductions b. Meghalaya
separatist movements, which and taxation since the
a. Karbi Anglong is the c. Mizoram
exploit alienation caused by late 1980s.
state’s largest district d. Nagaland
an insensitive and exploitative d. All of the above
and a melting pot of
state, and engage in extortion.
ethnicities and tribal 3. Karbi Anglong
While the focus has been on big
groups — Karbi, Autonomous Council is
groups such the NSCN-IM and

1. c. Assam Government pot of ethnicities and tribal Anglong district like People’s Inclusion of some areas into
Recently, a tripartite groups — Karbi, Dimasa, Democratic Council of Karbi Karbi Anglong Autonomous
agreement among five Bodo, Kuki, Hmar, Tiwa, Longri (PDCK), Karbi Longri Council (KAAC),
insurgent groups of Assam, Garo, Man (Tai speakers), NC Hills Liberation Front Reservation of seats for
the Centre and the state Rengma Naga. Its diversity (KLNLF), etc. originated from Scheduled Tribes,
government was signed. also generated different outfits the core demand of forming a More powers to the council,
This agreement is in synergy and fuelled an insurgency that separate state. Inclusion of Karbi language in
with the vision of Insurgency did not allow the region to 3. b. Sixth Schedule the Eighth Schedule,
free prosperous North East, develop. The Karbi Anglong Financial package of Rs 1,500
that envisages all-round • The Karbis are a major ethnic Autonomous Council (KAAC) crore.
development of northeast, group of Assam, dotted by is an autonomous district 5. d. Nagaland
peace and prosperity. several factions and splinters. council, protected under the The Sixth Schedule of the
2. d. All of the above The history of the Karbi group Sixth Schedule of the Indian Constitution deals with the
About the Karbi Anglong has been marked by killings, Constitution. administration of the tribal
Crisis ethnic violence, abductions 4. d. Financial package of Rs areas in the four northeastern
• Located in central Assam, and taxation since the late 9,000 crore. states of Assam, Meghalaya,
Karbi Anglong is the state’s 1980s. Some of the demands of the Tripura and Mizoram as per
largest district and a melting • Insurgent groups of Karbi militant groups are: Article 244.
Bhopal +91-7676564400 Indore +91-9589613810, +91-731-4987379 Prayagraj +91-8114000926, +91-8114000927 Kanpur +91-8576066660, +91-8707605589 E: support@toprankers.com
Gurugram +91-8448444207 Delhi +91-9810610466 Ranchi +91-9334969993 Lucknow +91-6390576666 Jabalpur +91-7004386936 W: www.toprankers.com
18 The CLAT Post • September 2021

Passage 18 time but also reduces

S ecurities and Exchange and frees up the capital


Board of India’s (Sebi’s) required to collateralise
decision to move to a that risk.
shortened trade settlement b. It also reduces the
cycle and implement tighter number of outstanding
margin norms will make unsettled trades at
India’s stock market safer any instant, and thus
and are in the interest of all decreases the unsettled
market participants, said exposure to Clearing
chairman Ajay Tyagi. Corporation by 50%.
“The transition from T+3 c. A shortened settlement
to T+2 took place in [1]. cycle will help in
There is a need to reduce reducing systemic risk.
it further now as there has
been significant reforms in d. All of the above
the payments and banking 4. If the stock exchange opts
system. Investors have for the T+1 settlement
the right to receive what cycle for a scrip, it will have
they purchase as quickly to mandatorily continue
as possible,” Tyagi said
with it for a minimum of
during a media interaction
how many months?
after addressing the CII
passage? b. It became autonomous a. 3 months
Financial Markets Summit on
Thursday. a. 2008 and given statutory
b. 6 months
powers by SEBI Act
Foreign Portfolio Investors b. 2005 c. 9 months
1992.
have opposed the move c. 2003 d. 12 months
saying it would cause c. The headquarters of
d. 2001 SEBI is situated in
operational challenges. 5. The regional offices of SEBI
Mumbai. is not located at which of
Tyagi said the regulator 2. Which of the following is
has allowed the shift of the true about SEBI? d. All of the above the following places?
settlement cycle to T+1 in a a. In April, 1988 the SEBI a. Ahmedabad
3. Which of the following is
phased manner keeping in was constituted as the b. Delhi
benefit of T+1 settlement
mind the concerns of FPIs. regulator of capital system? c. Kolkata
markets in India under
1. Which of the following will a. A shortened cycle not d. Bangalore
a resolution of the
replace [1] in the above only reduces settlement
Government of India.

1. c. 2003 (Control) Act, 1947. A shortened cycle not only trades will get proportionately
“The transition from T+3 In April, 1988 the SEBI was reduces settlement time but reduced with the number of
to T+2 took place in 2003. constituted as the regulator also reduces and frees up the outstanding unsettled trades at
There is a need to reduce it of capital markets in India capital required to collateralise any point of time.
further now as there has been under a resolution of the that risk. Reduction in Systemic Risks: A
significant reforms in the Government of India. Reduction in Unsettled Trade: shortened settlement cycle will
payments and banking system. Initially SEBI was a non- It also reduces the number of help in reducing systemic risk.
Investors have the right to statutory body without any outstanding unsettled trades at 4. b. 6 months
receive what they purchase as statutory power. any instant, and thus decreases If the stock exchange opts
quickly as possible,” Tyagi said It became autonomous and the unsettled exposure to for the T+1 settlement cycle
during a media interaction given statutory powers by SEBI Clearing Corporation by 50%. for a scrip, it will have to
after addressing the CII Act 1992. The narrower the settlement mandatorily continue with it
Financial Markets Summit on The headquarters of SEBI cycle, the narrower the time for a minimum 6 months.
Thursday. is situated in Mumbai. The window for a counterparty 5. d. Bangalore
2. d. All of the above regional offices of SEBI insolvency/bankruptcy to The headquarters of SEBI
Before SEBI came into are located in Ahmedabad, impact the settlement of a is situated in Mumbai. The
existence, Controller of Capital Kolkata, Chennai and Delhi. trade. regional offices of SEBI
Issues was the regulatory 3. d. All of the above Reduction in Blocked Capital: are located in Ahmedabad,
authority; it derived authority Benefits of T+1 Settlement: Further, the capital blocked in Kolkata, Chennai and Delhi.
from the Capital Issues Reduced Settlement Time: the system to cover the risk of
Bhopal +91-7676564400 Indore +91-9589613810, +91-731-4987379 Prayagraj +91-8114000926, +91-8114000927 Kanpur +91-8576066660, +91-8707605589 E: support@toprankers.com
Gurugram +91-8448444207 Delhi +91-9810610466 Ranchi +91-9334969993 Lucknow +91-6390576666 Jabalpur +91-7004386936 W: www.toprankers.com
19 The CLAT Post • September 2021

started yielding results, said b. SDG 10


Shankar. c. SDG 11
1. The Word Resource d. SDG 15
Institute India was
5. Which of the following
established in which of the
about ‘Connect Karo 2021’?
following years?
a. It is held every year
a. 1982
and it sees presenters
b. 1998 from various fields —
c. 2008 air pollution, electric
d. 2011 mobility, urban
planning, urban water
2. WRI is registered in India resilience, climate
as which of the following? mitigation, and public
a. India Resource Trust transit among others
b. Indian Charity sharing their insights
Organisation and research findings.
Passage 19 Shankar said that in the b. The event also saw
c. Indian National Trust
I n the next 30 years, Indian
cities will have improved
urban infrastructure and
coming years, the size of cities
in India will grow, and so will
opportunities for development
d. India Trust Resource
the launch of a book,
titled ‘Guidance Note
for Environmentally
and challenges attached to 3. Which of the following is
will be better equipped to Sensitive, Climate
it. “The Covid-19 pandemic the theme of Connect Karo
accommodate the population Adaptive and Socially
has slowed down the speed of 2021?
migrating from rural pockets, Inclusive Urban
said Durga Shankar Mishra, economic development globally, a. Clean, Green and Just
Riverfront Planning and
secretary of Union ministry and it has also hit India. We are b. Green is must Development’, by WRI-
of housing and urban affairs channelling all our resources to
c. Clean and Green India
at the India-Latin America meet the targets we have set for
our developmental projects,” d. Climate Justice c. It will be on addressing
urbanisation dialogue on
said Shankar. critical challenges faced
Wednesday, which was part of 4. Which of the following
the World Resources Institute’s by Indian cities with an
He added India has Sustainable Development
(WRI) annual flagship event emphasis on deep dive
been working on several Goals include investment
‘Connect Karo’. developmental programmes to cities of Mumbai, Delhi,
in public transport, creating Kochi, Hyderabad and
Discussing the growth tackle various aspects of urban green public spaces, and
growth, such as sanitation, Bengaluru.
and challenges of urban improving urban planning
development that both India infrastructure development d. All of the above
and management?
and Latin America are facing, and management. These
programmes have already a. SDG 8

1. d. 2011 3. a. Clean, Green and Just being organized by the World critical challenges faced by
WRI India—established WRI India’s flagship event Resources Institute (WRI) Indian cities with an emphasis
in 2011—works with local CONNECT KARO with through video conferencing. on deep dive cities of Mumbai,
and national governments, its 2021 theme of CLEAN, It brings together Indian, Delhi, Kochi, Hyderabad and
businesses, and civil society to GREEN and JUST explores global leaders, policymakers Bengaluru.
address India’s development pressing urban challenges and and other stakeholders The event also saw the launch
challenges. how best to address them. committed to designing of a book, titled ‘Guidance
2. a. India Resource Trust 4. c. SDG 11 inclusive, sustainable and Note for Environmentally
WRI India is an independent Sustainable Development climate forward Indian cities. Sensitive, Climate Adaptive
charity, legally registered as Goals (SDG) 11 includes It is held every year and it and Socially Inclusive Urban
the India Resources Trust. investment in public transport, sees presenters from various Riverfront Planning and
Connect Karo sees presenters creating green public spaces, fields — air pollution, electric Development’, by WRI-India
from various fields — air and improving urban mobility, urban planning, for conserving the river Ganga
pollution, electric mobility, planning and management in urban water resilience, climate under the Union government’s
urban planning, urban water participatory and inclusive mitigation, and public transit ‘Namami Gange’ project.
resilience, climate mitigation, ways. among others sharing their Launching the book, Rajiv
and public transit among 5. d. All of the above insights and research findings. Ranjan Mishra, director-
others — sharing their insights About Connect Karo 2021: The Its theme was ‘Clean, Green & general, National Mission for
and research findings. Connect Karo 2021 event is Just’. It will be on addressing Clean Ganga.
Bhopal +91-7676564400 Indore +91-9589613810, +91-731-4987379 Prayagraj +91-8114000926, +91-8114000927 Kanpur +91-8576066660, +91-8707605589 E: support@toprankers.com
Gurugram +91-8448444207 Delhi +91-9810610466 Ranchi +91-9334969993 Lucknow +91-6390576666 Jabalpur +91-7004386936 W: www.toprankers.com
20 The CLAT Post • September 2021

Passage 20 reference to World Bank?

C limate change could push


more than 200 million
people to leave their homes
a. MIGA is a member
of the World Bank
Group and its mandate
by 2050 unless urgent action is to promote cross-
is taken, the World Bank has border investment in
warned. In a landmark report developing countries by
published on Monday, the providing guarantees to
international financial institution investors and lenders.
called on states to reduce
b. International
global emissions and bridge the within national borders in the a. Hotspots of internal
Development
development gap to avoid the worst-case scenario. climate migration can
Association (IDA) is the
effects of slow-onset climate emerge as early as 2030
1. Which of the following part of the World Bank
change such as water scarcity, and continue to spread
replaces [1] in the passage? that helps the world’s
decreasing crop productivity, and and intensify by 2050.
rising sea levels. poorest countries.
a. South Asia b. The two groundswell
“All these issues are c. World Bank
b. Sub Saharan Africa reports’ combined
fundamentally connected, which Group consists of
c. East Asia and the findings provide, for
is why our support to countries is five development
Pacific the first time, a global
positioned to deliver on climate institutions.
d. North Africa picture of the potential
and development objectives scale of internal climate d. The International
together while building a more 2. In which of the following migration across the six Trade Centre is one of
sustainable, safe and resilient World Bank regions. the five development
years was the first
future,” Juergen Voegele, institutions.
Groundswell Report c. As many as 3% of total
vice president of Sustainable
published? population across the 5. Which of the following
Development at the World Bank,
said in a statement. a. 2001 six World Bank regions Sustainable Development
b. 2010 could be internal Goals covers migration?
[1] has been identified as the
climate migrants. a. SDG 7
most vulnerable region due to c. 2013
desertification, fragile coastlines d. All of the above. b. SDG 8
d. 2018
and the population’s dependence c. SDG 9
4. Which of the following
on agriculture. Up to 86 million 3. Which of the following is
statements is not true with d. SDG 10
people are projected to move true?

1. b. Sub Saharan Africa and used a robust and Centre is one of the five investment-disputes between
As many as 216 million people novel modeling approach development institutions. investors and countries.
could be internal climate to understand the scale, The World Bank Group While, The International Trade
migrants across the six World trajectory, and spatial patterns consists of five development Centre (ITC) is a multilateral
Bank regions (by 2050). This of future climate migration institutions. agency which has a joint
represents almost 3% of within countries, with a focus • International Bank for mandate with the World Trade
these regions’ total projected on three regions: Sub-Saharan Reconstruction and Organization (WTO) and the
population. Africa, South Asia, and Latin Development (IBRD) provides United Nations (UN) through
Sub-Saharan Africa: 85.7 America. loans, credits, and grants. the United Nations Conference
million internal climate 3. d. All of the above. • International Development on Trade and Development
migrants (4.2% of the total Hotspots of internal climate Association (IDA) provides (UNCTAD).
population); migration can emerge as early low- or no-interest loans to 5. d. SDG 10
East Asia and the Pacific: 48.4 as 2030 and continue to spread low-income countries. The SDGs’ central reference
million (2.5%); and intensify by 2050. The two • The International Finance to migration is made in Target
South Asia: 40.5 million groundswell reports’ combined Corporation (IFC) provides 10.7 to facilitate orderly,
(1.8%); findings provide, for the first investment, advice, and asset safe, regular and responsible
North Africa: 19.3 million (9 time, a global picture of the management to companies and migration and mobility of
%); potential scale of internal governments. people, including through the
Latin America: 17.1 million climate migration across the • The Multilateral Guarantee implementation of planned
(2.6%); and six World Bank regions. As Agency (MIGA) insures and well-managed migration
Eastern Europe and Central many as 3% of total population lenders and investors against policies, which appears under
Asia: 5.1 million (2.3%). across the six World Bank political risk such as war. Goal 10 to reduce inequality
2. d. 2018 regions could be internal • The International Centre for within and among countries.
First Groundswell Report: climate migrants. the Settlement of Investment
It was published in 2018 4. d. The International Trade Disputes (ICSID) settles
Bhopal +91-7676564400 Indore +91-9589613810, +91-731-4987379 Prayagraj +91-8114000926, +91-8114000927 Kanpur +91-8576066660, +91-8707605589 E: support@toprankers.com
Gurugram +91-8448444207 Delhi +91-9810610466 Ranchi +91-9334969993 Lucknow +91-6390576666 Jabalpur +91-7004386936 W: www.toprankers.com
21 The CLAT Post • September 2021

and then US energy secretary 4. Which of the following


Rick Perry. is NOT true regarding
Strategic Clean Energy
1. Which of the following Partnership?
replaces [1] in the passage?
a. A new India-U.S. Task
a. Jyotiraditya Scindia Force on Biofuels was
b. Kiren Rijeju announced.
c. Bhupendra Yadav b. It will support India
d. Hardeep Singh Puri in achieving its goal
2. The SCEP was launched in of installing 550GW
accordance with which of of renewable energy
the following? capacity by 2030.
a. US - India Climate and c. With this, the SCEP
Clean Energy Agenda inter-governmental
2030 Partnership engagement now
spreads across five
b. US - India Climate and pillars of cooperation.
Clean Energy Agenda
2035 Partnership d. The fifth Pillar added
is on Emerging Fuels
c. US - India Climate and (cleaner energy fuels).
Passage 21 “The meeting also reviewed the Clean Energy Agenda

I ndia and the US have added a progress on the India-US civil 2040 Partnership 5. Which of the following does
fifth track on ‘emerging fuels’ nuclear energy cooperation,” d. US - India Climate and not include India and USA’s
as part of US-India Strategic petroleum and natural gas Clean Energy Agenda as partners?
Clean Energy Partnership ministry said in a late Thursday 2045 Partnership a. Project Agreement
(SCEP), said India’s petroleum night statement. Energy (PA) on Air-launched
and natural gas ministry. This security is at the core of India- 3. The Indo-US nuclear Unmanned Aerial
came against the backdrop of US strategic energy partnership. cooperation agreement Vehicle (ALUAV)
India’s petroleum and natural India elevated the India-US was signed in which of the
energy dialogue to a strategic following years? b. NISAR
gas minister [1] on Thursday
co-chairing a virtual ministerial energy partnership in February a. 2005 c. Exercise Shakti
meeting with US energy 2018 after the first ministerial d. Malabar Exercise
meeting held in New Delhi on b. 2007
secretary Jennifer Granholm to
launch the revamped US-India 17 April 2018 between then oil c. 2008
SCEP. minister Dharmendra Pradhan d. 2010

1. d. Hardeep Singh Puri at the Leaders’ Summit on energy fuels). 25th edition of the exercise.
Jyotiraditya Scindia is the Climate held earlier this year • With this, the SCEP inter- India-US Agreement on
minister of Civil Aviation. (2021). governmental engagement ALUAV: India and the US have
Kiren Rijeju is the minister of 3. c. 2008 now spreads across five pillars signed a Project Agreement
Law and Justice. Bhupendra The Indo-US nuclear of cooperation - Power and (PA) to jointly develop an Air-
Yadav is the minister of cooperation agreement was Energy Efficiency, Responsible launched Unmanned Aerial
Environment, Forest and signed in 2008. A major Oil and Gas, Renewable Vehicle (ALUAV) or drones
Climate Change and also aspect of the Deal was that Energy, Sustainable Growth that can be launched from an
for the ministry of Labour the Nuclear Suppliers Group and Emerging Fuels. aircraft.
and Employment. Hardeep (NSG) gave a special waiver • Support India in achieving its NISAR: NASA and ISRO are
Singh Puri is the minister to India that enabled it to sign goal of installing 450GW of collaborating on developing
of Petroleum and Natural cooperation agreements with a renewable energy capacity by an SUV-sized satellite called
Gas and also the minister of dozen countries. 2030. NISAR, which will detect
Housing and Urban Affairs. 4. b. It will support India • A new India-U.S. Task movements of the planet’s
2. a. US - India Climate and in achieving its goal of Force on Biofuels was also surface as small as 0.4 inches
Clean Energy Agenda 2030 installing 550GW of announced. over areas about half the size
Partnership renewable energy capacity by 5. c. Exercise Shakti of a tennis court.
The SCEP was launched in 2030. Malabar Exercise: The Navies ‘Exercise SHAKTI’ is a biennial
accordance with the US - India Revamped Strategic Clean of the Quad (Quadrilateral exercise which is conducted
Climate and Clean Energy Energy Partnership (SCEP): Framework) Nations (India, alternately in India and France.
Agenda 2030 Partnership • Addition of a fifth Pillar the United States, Japan, and
announced by both countries on Emerging Fuels (cleaner Australia) participated in the
Bhopal +91-7676564400 Indore +91-9589613810, +91-731-4987379 Prayagraj +91-8114000926, +91-8114000927 Kanpur +91-8576066660, +91-8707605589 E: support@toprankers.com
Gurugram +91-8448444207 Delhi +91-9810610466 Ranchi +91-9334969993 Lucknow +91-6390576666 Jabalpur +91-7004386936 W: www.toprankers.com
22 The CLAT Post • September 2021

Passage 22 released to the public at the students of classes c. Ministry of Education

T he government’s think- event to engage the wisdom 6 to 12 are given an d. Planning Commission
tank’s [1]’s Atal Innovation of the scientific community to open platform where
Mission and Atal Tinkering analyse the data. they can innovate and 4. Which of the following is
Labs (ATL) have joined hands enable themselves to an initiative under Atal
1. Which of the following solve digital age space Innovation Mission for
with the Indian Space Research
is NOT TRUE about the technology problems. Creating problem solving
Organisation (ISRO) and the
World Space Week? mindset across schools in
Central Board of Secondary b. Under the ATL
Education (CBSE) to launch the a. It is coordinated by the initiative, schools India?
ATL Space Challenge 2021 for United Nations with the receive a grant for a. Atal Incubation Centers
young students during World support of the World setting up such labs b. Atal Tinkering Labs
Space Week from October 4 to Space Week Association to encourage students
10. (WSWA), a non-profit c. Atal Research and
to give shape to their Innovation for Small
Dr. Sudheer Kumar, Director, organisation. ideas and come up with Enterprises (ARISE)
Capacity Building Programme b. The United Nations innovative experiments
Office, Isro, Dr Chintan General Assembly on their own in these d. Mentor India Campaign
Vaishnav, Mission Director, (UNGA) declared the laboratories. 5. Which of the following is
Atal Innovation Mission, [1], World Space Week in c. This comes against the India’s rank in the Global
and CBSE Chairman Manoj 1999. backdrop of India’s Innovation Index 2021?
Ahuja were present at the c. World Space Week is push to move up in a. 52
virtual launch of the space held each year from 4th- the Global Innovation
challenge on Monday. 10th September. b. 50
Index
The launch of this challenge c. 48
d. In 2021, World Space d. All of the above.
for young students was part d. 46
Week celebrates
of Isro’s two-day online lunar 3. Which of the following
“Women in Space.”
science workshop on September replaces [1] in the passage?
6 and 7 which was be broadcast 2. Which of the following is
live on Isro’s official Facebook a. Niti Aayog
true about the ‘ATL Space
page and the Isro website. Challenge 2021’? b. Ministry of Science and
Chandrayaan-2 data was also Technology
a. It is to ensure that

1. c. World Space Week is Governing the Activities of Week 2021 which is observed problem solving mindset
held each year from 4th-10th States in the Exploration and from 4th -10th October across schools in India.
September. Peaceful Uses of Outer Space each year, to celebrate the Atal Incubation Centers:
World Space Week is an including the Moon and other contributions of space science Fostering world class startups
international celebration of celestial bodies). and technology. and adding a new dimension
science and technology and 2. d. All of the above. This comes against the to the incubator model.
their contribution to the This is to ensure that students backdrop of India’s push Mentor India Campaign: A
betterment of the human of classes 6 to 12 are given to move up in the Global national mentor network
condition. an open platform where they Innovation Index (GII, in collaboration with the
It is coordinated by the United can innovate and enable released by the World public sector, corporates and
Nations with the support themselves to solve digital age Intellectual Property institutions, to support all the
of the World Space Week space technology problems. Organisation) rankings. initiatives of the mission.
Association (WSWA), a non- Students from both ATL (Atal 3. a. Niti Aayog Atal Research and Innovation
profit organisation. Tinkering Labs) and non- The government’s think-tank’s for Small Enterprises (ARISE):
The United Nations General ATL schools can submit their Niti Aayog’s Atal Innovation To stimulate innovation
Assembly (UNGA) declared in entries in the space challenge. Mission and Atal Tinkering and research in the MSME
1999 that World Space Week School teachers, ATL In- Labs (ATL) have joined hands industry.
would be held each year from charges and mentors may with the Indian Space Research 5. d. 46
4th-10th October. These dates support the student teams. Organisation (ISRO) and the India has improved its position
commemorate two events: Under the ATL initiative, Central Board of Secondary on the Global Innovation
4th October 1957: Launch of schools receive a grant Education (CBSE) to launch Index (GII) 2021 prepared by
the first human-made Earth for setting up such labs to the ATL Space Challenge 2021 the World Intellectual Property
satellite, Sputnik 1, thus encourage students to give for young students during Organisation (WIPO). India
opening the way for space shape to their ideas and World Space Week from has been ranked 46th this year,
exploration. come up with innovative October 4 to 10. marking an improvement from
10th October 1967: The experiments on their own in 4. b. Atal Tinkering Labs last year when it occupied the
signing of the Outer Space these laboratories. Major Initiatives: 48th position.
Treaty (Treaty on Principles It aligns with the World Space Atal Tinkering Labs: Creating
Bhopal +91-7676564400 Indore +91-9589613810, +91-731-4987379 Prayagraj +91-8114000926, +91-8114000927 Kanpur +91-8576066660, +91-8707605589 E: support@toprankers.com
Gurugram +91-8448444207 Delhi +91-9810610466 Ranchi +91-9334969993 Lucknow +91-6390576666 Jabalpur +91-7004386936 W: www.toprankers.com
23 The CLAT Post • September 2021

Passage 23 Ministry of Statistics

I ndia’s industrial production and Programme


rose 11.5 per cent in July as low Implementation.
base effect waned, according to c. Base Year for Index of
official data released on Friday. Industrial Production is
As per the Index of Industrial 2011-2012.
Production (IIP) data released d. It is used by government
by the[ [1], the manufacturing agencies including the
sector’s output surged 10.5 per Ministry of Finance, the
cent in July 2021. Reserve Bank of India,
In July, the mining output etc, for policy-making
climbed 19.5 per cent and power purposes.
generation increased 11.1 per
cent. 4. Which of the following is
correct about GDP?
On use-based classification,
primary goods saw 12.4 per cent a. It is a measure of
growth in output, while capital economic activity in a
goods’ production increased 29.5 country.
per cent. Intermediate goods passage? b. 9.4% b. It is the total value of a
recorded 14.1 per cent increase in country’s annual output
production, while infrastructure a. National Statistical c. 11.2%
of goods and services.
or construction goods saw 11.6 Office d. 11.5%
c. It gives the economic
per cent rise in production. b. Niti Aayog
3. Which of the following is output from the
Production of consumer durables c. United Nations Statistics consumers’ side.
incorrect about Index of
increased 20.2 per cent during Division
Industrial Production? d. All of the above
July, and consumer non-durables d. National Sample Survey
declined 1.8 per cent. The IIP had a. IIP is an indicator that
Organisation 5. Who among the following is
contracted 10.5 per cent in July measures the changes
the head of the Ministry of
2020. During April-July this year, 2. The country’s output of in the volume of
Commerce and Industry?
the IIP grew 34.1 per cent. It was the eight core sectors production of industrial
a contraction of 29.3 per cent in products during a given a. Prakash Javadekar
- also known as the
the same period last year. infrastructural output, grew period. b. Dr. Harshvardhan
at what percentage in July b. It is compiled and c. Piyush Goyal
1. Which of the following will 2021? published monthly
replace [1] in the above d. Smiriti Irani
a. 10.8% by Niti Aayog,

1. a. National Statistical Implementation. Significance of IIP: + Gross investment +


Office Index of Industrial Production: It is used by government Government investment +
India’s industrial output grew IIP is an indicator that agencies including Government spending +
by 11.5% in July compared to a measures the changes in the the Ministry of Finance, the (exports-imports)
10.5% contraction a year ago, volume of production of Reserve Bank of India, etc, for 5. c. Piyush Goyal
as per quick estimates from industrial products during a policy-making purposes. Piyush Goyal (57) is the
the National Statistical Office given period. IIP remains extremely relevant Minister of Commerce &
(NSO). It is compiled and published for the calculation of the Industry, Consumer Affairs
2. b. 9.4% monthly by the National quarterly and advance GDP & Food & Public Distribution
Mining, manufacturing and Statistical Office (NSO), (Gross Domestic Product) and Textiles, Government
electricity recorded growth Ministry of Statistics and estimates. of India. Leader of House in
of 19.5%, 10.5% and 11.1%, Programme Implementation. 4. d. All of the above Rajya Sabha.
respectively, albeit driven by It is a composite indicator Gross Domestic Product He is a Member of the Upper
favourable base effect. that measures the growth rate (GDP): House (Rajya Sabha) of the
The country’s output of the of industry groups classified It is a measure of economic Parliament of India and the
eight core sectors - also known under: activity in a country. It is Leader ofHouse in Rajya
as the infrastructural output, Broad sectors, namely, Mining, the total value of a country’s Sabha. He was earlier the
grew 9.4% in July 2021. Manufacturing, and Electricity. annual output of goods and Minister of Railways and Coal
3. b. It is compiled and Use-based sectors, namely services. It gives the economic (2017 – 19).
published monthly by Basic Goods, Capital Goods, output from the consumers’
Niti Aayog, Ministry of and Intermediate Goods. side.
Statistics and Programme Base Year for IIP is 2011-2012. GDP = Private consumption
Bhopal +91-7676564400 Indore +91-9589613810, +91-731-4987379 Prayagraj +91-8114000926, +91-8114000927 Kanpur +91-8576066660, +91-8707605589 E: support@toprankers.com
Gurugram +91-8448444207 Delhi +91-9810610466 Ranchi +91-9334969993 Lucknow +91-6390576666 Jabalpur +91-7004386936 W: www.toprankers.com
24 The CLAT Post • September 2021

Passage 24 Diarrhoeal Disease Research, Ramon Magsaysay Award annually selects the

A Bangladeshi vaccine Bangladesh, (an international in the above passage? awardees for the field of
scientist and a microfinance health research institute based in a. Japan Emergent Leadership.
pioneer from Pakistan were Dhaka) in 1988. d. All of the above
b. Philippines
among the five recipients of this Today, she is credited with
year’s Ramon Magsaysay Award developing affordable oral c. Laos 4. Ramon Magsaysay Award
— regarded as the Asian version cholera vaccine and the typhoid d. Singapore was established in which of
of the [1] — announced on conjugate vaccine for adults, the following years?
Tuesday. children, and infants. Most of 3. Which of the following
a. 1955
her work is focused in congested is TRUE about Ramon
Apart from Dr Firdausi Magsaysay Award? b. 1957
Qadri from Bangladesh and slum areas of developing
countries. a. Recognises and honours c. 1962
Muhammad Amjad Saqib from
Pakistan, the other winners are individuals and d. 1977
1. Which of the following will organisations in Asia,
Filipino fisher and community
replace [1] in the above regardless of race, creed, 5. Who among the following
environmentalist Roberto Ballon,
passage? gender, or nationality. was the First Winner of
American Steven Muncy for
a. Pulitzer Prize Ramon Magsaysay Award?
humanitarian work and refugee b. Till 2009, awards were
assistance and Indonesian b. Bookers Prize traditionally given in a. Jawaharlal Nehru
torchbearer for investigative c. Nobel Prize five categories. b. Sardar Patel
journalism, Watchdoc. Qadri, c. However, post 2009, c. Vinoba Bhave
70, who has a doctorate from d. Abel Prize
the Ramon Magsaysay d. Mahatma Gandhi
Liverpool University, UK, 2. Which of the following Award Foundation
joined International Centre For countries established the

1. c. Nobel Prize 2. b. Philippines on 31st August each year. prize and highest honour.
The Ramon Magsaysay Award Recently the Ramon Magsaysay 3. d. All of the above It is named after Ramon
was established in 1957 and is Award 2021 was announced. 4. b. 1957 Magsaysay, the third president
considered Asia’s premier prize The award is presented in a The Ramon Magsaysay Award of the Republic of the
and highest honour. formal ceremony in Manila, was established in 1957 and Philippines.
the capital of the Philippines is considered Asia’s premier 5. c. Vinoba Bhave
Bhopal +91-7676564400 Indore +91-9589613810, +91-731-4987379 Prayagraj +91-8114000926, +91-8114000927 Kanpur +91-8576066660, +91-8707605589 E: support@toprankers.com
Gurugram +91-8448444207 Delhi +91-9810610466 Ranchi +91-9334969993 Lucknow +91-6390576666 Jabalpur +91-7004386936 W: www.toprankers.com
25 The CLAT Post • September 2021

Passage 25 country?

S ri Lanka has seen long queues


to buy essential items amid
tight lockdown measures to


a. Article 360
b. Article 375
c. Article 134
control the spread of Covid-19.
Shelves at government-run d. Article 370
supermarkets have been running
low - some even empty - with 5. Which of the following
very little stock remaining statements is not true?
of imported goods like milk a. Sri Lanka does not
powder, cereal and rice. The have a universal public
government denies there are distribution system or
shortages and blames the media ration cards that can
upper middle-income country depreciated by around
for stoking fears. It follows the ensure essential goods
by the World Bank, but then 8% as of September
government declaring a state reach all consumers.
downgraded the following year. 2021.
of emergency and Sri Lanka’s b. In June 2021, Sri
Central Bank chief stepping c. The country’s debt
1. Which of the following Lankan Government’s
down amid a foreign exchange burden has also been
replaces [1] in the passage? decision to ban import
crisis. growing - from 39% of
a. Mahinda Rajapaksa of chemical fertilizers
On 30 August, President [1] Gross National Income
b. Basil Rajapaksa and adopt an “organic
announced strict controls on the (GNI) in 2010 to 69% in
only” approach.
supply of essential goods. The c. Gotabaya Rajapaksa 2019, according to the
World Bank. c. In 1947, the free
government said this was needed d. Maithripala Sirisena market economy was
to prevent traders hoarding d. All of the above
2. Which of the following is introduced to the
food items and control inflation.
true regarding Sri Lankan 3. Which of the following country, incorporating
Sri Lanka is grappling with a
Economic Crisis? replaces [2] in the passage? privatisation,
depreciating currency, inflation
deregulation, and the
and a crippling foreign debt a. The tourism industry, a. 2017
promotion of private
burden. There has also been a which represents over b. 2019
slump in foreign tourism due enterprise.
10% of the country’s c. 2018
to the pandemic. The economic Gross Domestic Product d. The emergency
slowdown is of particular d. 2020 was declared under
and brings in foreign
concern for a country that exchange, has been hit the Public Security
4. Which of the following Ordinance on the
has been one of the strongest hard by the coronavirus
performing in South Asia. empowers President supply of essential
pandemic. of India to proclaim a
In [2], it was upgraded to an goods.
b. Sri Lankan rupee has Financial Emergency in the

1. c. Gotabaya Rajapaksa in July 2021. With the supply 4. a. Article 360 welfare state established.
Mahinda Rajapaksa: Prime of foreign exchange drying Article 360 empowers the In 1977, the free market
Minister of Sri Lanka. Basil up, the amount of money that president to proclaim a economy was introduced to
Rajapaksa: Minister of Finance Sri Lankans have had to shell Financial Emergency if he the country, incorporating
Gotabaya Rajapaksa: President out to purchase the foreign is satisfied that a situation privatisation, deregulation,
of Sri Lanka Maithripala exchange necessary to import has arisen due to which the and the promotion of private
Sirisena: Former President goods has risen. Due to this, financial stability or credit enterprise. In June 2021, Sri
of Sri Lanka serving as the the value of the Sri Lankan of India or any part of its Lankan Government’s decision
seventh President from 9 rupee has depreciated by territory is threatened. to ban import of chemical
January 2015 to 18 November around 8% so far this year. 5. c. In 1947, the free market fertilizers and adopt an
2019. The country’s debt burden economy was introduced to “organic only” approach. The
2. d. All of the above has also been growing - from the country, incorporating emergency was declared under
The tourism industry, which 39% of Gross National Income privatisation, deregulation, the Public Security Ordinance
represents over 10% of the (GNI) in 2010 to 69% in 2019, and the promotion of private on the supply of essential
country’s Gross Domestic according to the World Bank. enterprise. goods. a. Sri Lanka does
Product and brings in foreign 3. b. 2019 From 1948 to 1977, socialism not have a universal public
exchange, has been hit hard In 2019, Sri Lanka was strongly influenced the distribution system or ration
by the coronavirus pandemic. upgraded to an upper middle- government’s economic cards that can ensure essential
As a result, forex reserves have income country by the World policies. Colonial plantations goods reach all consumers.
dropped from over $7.5 billion Bank, but then downgraded were dismantled, industries
in 2019 to around $2.8 billion the following year. were nationalised, and a
Bhopal +91-7676564400 Indore +91-9589613810, +91-731-4987379 Prayagraj +91-8114000926, +91-8114000927 Kanpur +91-8576066660, +91-8707605589 E: support@toprankers.com
Gurugram +91-8448444207 Delhi +91-9810610466 Ranchi +91-9334969993 Lucknow +91-6390576666 Jabalpur +91-7004386936 W: www.toprankers.com
26 The CLAT Post • September 2021

Passage 26 Biden-Harris Administration’s Indo-Pacific region. was also discussed by

I n New Delhi, the Indian priority of engaging in the c. The Quad is not the QUAD leaders.
foreign ministry early Indo-Pacific, including through structured like a d. Concerns regarding
Tuesday confirmed that Modi new multilateral configurations typical multilateral Chinese cyberattacks
would travel to the US for to meet the challenges of the organisation and lacks on the USA targets
the Quad summit and to 21st century.” a secretariat and any (Microsoft Exchange
participate in the UN General 1. Who among the following permanent decision- and SolarWinds) and
Assembly session on 25 introduced the idea of making body. also cybersecurity
September. Bilateral meetings formation of the QUAD d. In March 2021, the incidents in India, Japan
with Biden and other leaders coalition? Quad leaders met and Australia were
are also expected and the a. Manmohan Singh virtually and later discussed.
announcements could be made released a joint
soon, two people familiar with b. Shinzo Abe 5. Which of the following is
statement which
the matter said separately. c. George W. Bush outlined the group’s the joint statement released
Modi’s visit to the US is his first d. Kevin Rudd approach and objectives. by the QUAD members
visit abroad in six months—the after their first Summit?
prime minister had visited 2. Which of the following 4. Which of the following a. The Spirit of the Quad
Bangladesh in March for the countries hosted the first- issues regarding China was b. The Strategy of the
50th anniversary celebrations ever QUAD Summit? not discussed in the first Quad
of Bangladesh’s emergence as a. India QUAD Summit?
c. The Quad Cooperation
a separate country. Modi was b. Australia a. The Line of Actual Dialogue
supposed to visit Europe in c. USA Control (LAC) was
May but the trip was called discussed by the QUAD d. The Quad Security
off after India was hit by a d. Japan leaders as one of several Dialogue
particularly brutal second wave examples of Chinese 6. In which of the following
3. Which of the following places was the 25th edition
of covid-19 infections. aggression.
statements is not true? of Malabar Exercise
The US statement said that the b. Issues related to Hong
a. In 2020, all four QUAD conducted?
“Biden-Harris Administration Kong, Xinjiang, Taiwan
Countries - Japan, India, a. Vishakhapatnam, Bay of
has made elevating the Quad Strait and coercion of
Australia and the USA Bengal
a priority, as seen through Australia, harassment
took part in the Ajeya
the first-ever Quad Leaders- around the Senkaku, b. Sasebo, Omura Bay
Warrior 2020 Exercise.
level engagement in March, were also discussed. c. Guam, Pacific Ocean
which was virtual, and now b. QUAD is viewed as
a counterbalance to c. Possibility of China d. Kochi, Arabian Sea
this Summit, which will be employing bioweapons
in-person. Hosting the leaders China’s military and
economic clout in the in its military strategies
of the Quad demonstrates the

1. b. Shinzo Abe (Quadrilateral Framework). its military strategies was outlined the group’s approach
The idea of Quad was first The meeting was hosted by the also discussed by the QUAD and objectives. According to
mooted by Japanese Prime USA on a virtual platform. leaders. the ‘Spirit of the Quad’, the
Minister Shinzo Abe in 2007. 3. a. In 2020, all four QUAD Following are the factors group’s primary objectives
However, the idea couldn’t Countries - Japan, India, discussed the first QUAD include maritime security,
move ahead with Australia Australia and the USA took Summit held in March 2021 combating the Covid-19 crisis,
pulling out of it, apparently part in the Ajeya Warrior with reference to China: especially vis-à-vis vaccine
due to Chinese pressure. In 2020 Exercise. • The Line of Actual Control diplomacy, addressing the risks
December 2012, Shinzo Abe The Quad is not structured (LAC) was discussed by the of climate change, creating
again floated the concept of like a typical multilateral QUAD leaders as one of an ecosystem for investment
Asia’s “Democratic Security organisation and lacks a several examples of Chinese in the region and boosting
Diamond” involving Australia, secretariat and any permanent aggression. technological innovation.
India, Japan and the US decision-making body. QUAD • Other issues related to Hong 6. c. Guam, Pacific Ocean
to safeguard the maritime is viewed as a counterbalance Kong, Xinjiang, Taiwan Strait The Navies of the Quad
commons from the Indian to China’s military and and coercion of Australia, (Quadrilateral Framework)
Ocean to the western Pacific. economic clout in the Indo- harassment around the Nations (India, the United
In November 2017, India, the Pacific region. Senkaku, were also discussed. States, Japan, and Australia)
US, Australia and Japan gave In March 2021, the Quad • Concerns regarding Chinese have participated in the
shape to the long-pending leaders met virtually and later cyberattacks on the USA 25th edition of the Malabar
“Quad” Coalition to develop released a joint statement targets (Microsoft Exchange Exercise, which began off the
a new strategy to keep the titled ‘The Spirit of the Quad,’ and SolarWinds) and also coast of Guam in the Pacific
critical sea routes in the Indo- which outlined the group’s cybersecurity incidents in Ocean. Guam is a US island
Pacific free of any influence approach and objectives. In India, Japan and Australia territory in the North Pacific
(especially China). 2020, all four QUAD Countries were discussed. Ocean, which is much closer
2. c. USA - Japan, India, Australia and 5. a. The Spirit of the Quad to China in the East than to
In March 2021, the Prime the USA took part in the In March 2021, the Quad the US mainland in the West
Minister, Narendra Modi Malabar Exercise. leaders met virtually and later making it an ideal US military
addressed the first summit 4. c. Possibility of China released a joint statement titled strategic outpost for Indo-
of the leaders’ of the QUAD employing bioweapons in ‘The Spirit of the Quad,’ which Pacific operations.
Bhopal +91-7676564400 Indore +91-9589613810, +91-731-4987379 Prayagraj +91-8114000926, +91-8114000927 Kanpur +91-8576066660, +91-8707605589 E: support@toprankers.com
Gurugram +91-8448444207 Delhi +91-9810610466 Ranchi +91-9334969993 Lucknow +91-6390576666 Jabalpur +91-7004386936 W: www.toprankers.com
27 The CLAT Post • September 2021

Passage 27 a. Black Hydrogen substantial fraction of 1988

T he Centre on Tuesday b. Charcoal Hydrogen a nation’s energy and d. Robert Michael Gates,
constituted a task force and services. 2008
c. Coke Hydrogen
expert committee to prepare c. It is an envisioned
d. Carbonized Hydrogen 4. Which of the following
a road map for coal-based future where hydrogen
hydrogen production. This is is used as fuel for statements is not true about
2. Which of the following
aimed at contributing to Prime vehicles, energy storage National Hydrogen Energy
statements is not true
Minister Narendra Modi’s and long-distance Mission?
regarding Hydrogen
agenda of a hydrogen-based Economy? transport of energy. a. It focuses on generation
economy in a clean manner. “The of hydrogen from green
a. Hydrogen is a zero- d. The different pathways
Ministry of Coal has constituted power resources.
carbon fuel and to use hydrogen
2 Committees today, one to
is considered an economy includes b. It will help India to
oversee the program and another
alternative to fuel hydrogen production, achieve its emission
of experts to give guidance to the
goals under the Paris
Ministry,” the coal ministry said
Agreement.
in a statement.
c. India’s ambitious goal
The task force has been
constituted under the of 350 GW by 2022
chairmanship of Coal Additional got an impetus in the
Secretary Vinod Kumar Tiwari. 2021-22 budget which
Its broad terms of reference allocated Rs.1500 crore
include monitoring of activities for renewable energy
towards achieving coal-based development and NHM.
hydrogen production and usage d. All of the above
and coordination with the Coal
Gasification Mission and NITI 5. Green Hydrogen Energy
Aayog. The broad terms of is particularly important
reference of the expert committee for India to meet the
include identifying experts in and a key source of storage, transport and environmental goals of
India and co-opting as members, clean energy. It can utilization. which of the following by
desk-based review of progress be produced from 2030?
3. Who among the following
in hydrogen technology and renewable sources of a. Kyoto Protocol
coined the term ‘Hydrogen
also reviewing ongoing research energy such as coal and b. Nationally Determined
Economy’ and in which
projects in hydrogen technology. fossil fuels. Contribution (INDC)
year?
b. It is an economy that Targets (COP 21)
1. Hydrogen produced on the a. Jim O’Neil, 2007
relies on hydrogen as c. Stockholm Convention
basis of Coal is also known b. John Bockris, 1970
the commercial fuel
as which of the following? d. Rotterdam Convention
that would deliver a c. Brian Evans Conway,

1. a. Black Hydrogen Hydrogen is a zero-carbon and utilization. resources. It will help India
Coal is one of the important fuel and is considered an 3. b. John Bockris, 1970 to achieve its emission goals
sources of hydrogen making alternative to fuel and a key In 1970, the term ‘Hydrogen under the Paris Agreement.
(Black or Brown Hydrogen) source of clean energy. It can Economy’ was coined by John India’s ambitious goal of 175
apart from Natural Gas (Grey be produced from renewable Bockris. He mentioned that a GW by 2022 got an impetus
hydrogen) and renewable sources of energy such as solar hydrogen economy can replace in the 2021-22 budget which
energy (Green Hydrogen) and wind. It is an economy the current hydrocarbon-based allocated Rs.1500 crore for
through electrolysis. In case that relies on hydrogen as the economy, leading to a cleaner renewable energy development
of renewable energy (Green commercial fuel that would environment. and NHM.
Hydrogen) surplus solar power deliver a substantial fraction of 4. c. India’s ambitious goal 5. b. Nationally Determined
is used to electrolyze water a nation’s energy and services. of 350 GW by 2022 got an Contribution (INDC) Targets
into hydrogen and oxygen. It is an envisioned future impetus in the 2021-22 (COP 21)
2. a. Hydrogen is a zero- where hydrogen is used as fuel budget which allocated Green hydrogen energy is vital
carbon fuel and is considered for vehicles, energy storage Rs.1500 crore for renewable for India to meet its Nationally
an alternative to fuel and a and long-distance transport energy development and Determined Contribution
key source of clean energy. of energy. The different NHM. (INDC) Targets and ensure
It can be produced from pathways to use hydrogen National Hydrogen Energy regional and national
renewable sources of energy economy includes hydrogen Mission focuses on generation energy security, access and
such as coal and fossil fuels. production, storage, transport of hydrogen from green power availability.
Bhopal +91-7676564400 Indore +91-9589613810, +91-731-4987379 Prayagraj +91-8114000926, +91-8114000927 Kanpur +91-8576066660, +91-8707605589 E: support@toprankers.com
Gurugram +91-8448444207 Delhi +91-9810610466 Ranchi +91-9334969993 Lucknow +91-6390576666 Jabalpur +91-7004386936 W: www.toprankers.com
28 The CLAT Post • September 2021

Passage 28 sustainability of initiatives under which of the urban development?

S wachh Survekshan is an taken towards garbage free following ministry of a. Swachh Bharat Mission.
annual survey of cleanliness, and open defecation free cities, India?
b. Pradhan Mantri Awas
hygiene and sanitation in cities provide credible outcomes a. Urban and Housing Yojana.
and towns across India. It was which would be validated Development
launched as part of the Swachh by third party certification, c. AMRUT Mission.
institutionalize existing systems b. Jal Shakti Ministry
Bharat Abhiyan, which aimed d. Skill development
to make India clean and free through online processes and c. Women Empowerment Scheme for Tribals.
of open defecation by 2nd create awareness amongst d. Home Ministry
October 2019. The first survey all sections of society about 4. Which of the following can
was undertaken in 2016 and the importance of working 2. Which of the following be replaced by [1] in the
covered 73 cities; by 2020 the together towards making towns statement is correct following passage?
survey had grown to cover 4242 and cities more habitable and regarding Swachh a. Fourth
cities and was said to be the sustainable. Additionally, the Survekshan Grameen 2021?
b. Fifth
largest cleanliness survey in the survey also intends to foster a a. As part of the
spirit of healthy competition c. Sixth
world. In a bid to scale up the Survekshan, Villages,
coverage of the ranking exercise amongst towns and cities to Districts and States d. Seventh
and encourage towns and cities improve their service delivery would be ranked using
to citizens and move towards 5. Clean and accessible water
to actively implement mission key parameters.
creating cleaner cities. for all comes which of the
initiatives in a timely and b. An expert agency has following sustainable goal?
innovative manner, Ministry MoHUA & QCI will conduct been hired to conduct
of Housing and Urban Affairs intensive virtual interactions a. SDG – 4
the Survekshan 2021.
(MoHUA) is now in the process with States and ULBs to b. SDG – 6
of conducting the [1] edition familiarize them with various c. Before this, the Swachh
Survekshan Grameen c. SDG – 8
of the survey to rank all cities facets of the survey such as
under Swachh Bharat Mission- survey methodology, survey had been carried out d. SDG – 10
Urban (SBM-U) with Quality process and indicators, amongst by the Ministry in 2018
Council of India (QCI) as its others, while also clarifying and 2019.
implementation partner. their expectations from the d. all of the above
The objective of the survey survey.
3. Which of the following
is to encourage large scale
1. Swachh Survekshan initiatives takes by central
citizen participation, ensure
Grameen was launched government is not for

1. b. Jal Shakti Ministry It is carried out by an expert – Urban (PMAY-U), a flagship Tribal Affairs also implements
the Jal Shakti Ministry agency. Mission of Government of schemes applicable universally
launched Swachh Survekshan Coverage: India being implemented on pan India.
Grameen 2021 or rural As part of the 2021 rural by Ministry of Housing and 4. c. Sixth
cleanliness survey 2021 cleanliness survey, as many as Urban Affairs (MoHUA), was Ministry of Housing and
under Swachh Bharat Mission 17,475 villages spanning across launched on 25th June 2015. Urban Affairs (MoHUA)
(Grameen) Phase-II. Before 698 districts would be covered. The Government of India has is now in the process of
this, the Swachh Survekshan Before this, the Swachh launched the Atal Mission conducting the sixth edition
Grameen had been carried out Survekshan Grameen had been for Rejuvenation and Urban of the survey to rank all cities
by the Ministry in 2018 and carried out by the Ministry in Transformation (AMRUT) under Swachh Bharat Mission-
2019. 2018 and 2019. with the aim of providing Urban (SBM-U) with Quality
2. d. all of the above The Swachh Survekshan Urban basic civic amenities like Council of India (QCI) as its
Assesses the state of hygiene, 2021 is to be announced. It was water supply, sewerage, urban implementation partner.
cleanliness and sanitation in introduced by the Ministry of transport, parks as to improve 5. b. SDG – 6
rural India as a part of the Housing and Urban Affairs the quality of life for all DG 6 focuses on clean,
Centre’s initiative to award (MoHUA) in 2016 especially the poor and the accessible water for all. The
Open Defecation Free (ODF) 3. d. Skill development Scheme disadvantaged. goal aims to address water
Plus status to villages. for Tribals. Ministry of Rural scarcity, poor water quality
ODF-plus status aims to The Swachh Bharat Mission Development has initiated and inadequate sanitation
ensure management of solid - Urban (SBM-U), launched a new skill development globally. It promotes increased
as well as liquid waste and is on 2nd October 2014 aims at initiative within the Aajeevika investments in water
an upgradation of ODF status making urban India free from Skills (Placement Linked Skill management and sanitation,
in which construction of open defecation and achieving Development Scheme) called and international cooperation
adequate toilets was required 100% scientific management of “Roshni” for youth from 27 and capacity building in this
so that people did not have to municipal solid waste. most critical LWE affected respect.
defecate in the open. Pradhan Mantri Awas Yojana districts; and The Ministry of
Bhopal +91-7676564400 Indore +91-9589613810, +91-731-4987379 Prayagraj +91-8114000926, +91-8114000927 Kanpur +91-8576066660, +91-8707605589 E: support@toprankers.com
Gurugram +91-8448444207 Delhi +91-9810610466 Ranchi +91-9334969993 Lucknow +91-6390576666 Jabalpur +91-7004386936 W: www.toprankers.com
29 The CLAT Post • September 2021

Passage 29 out individual banks.

F ollowing through with one


of her key announcements
in the Budget, Finance Minister
d. It can drive a better
bargain with borrowers
and take more stringent
Nirmala Sitharaman has enforcement action
announced the formation of against them.
India’s first-ever “Bad Bank”. 3. Which of the following
She said the “National Asset replace [1] with the
Reconstruction Company
following passage?
Limited” (NARCL) has already
been incorporated under the this is where the banks earn taking deposits, but a. Rs. 3 lakh crore
Companies Act. It will acquire interest and this is money that helps commercial banks b. Rs 30600 crore
stressed assets worth about the borrower has to return to the clean up their balance c. Rs 7000 crore
Rs 2 lakh crore from various bank. sheets and resolve bad d. None of the following.
commercial banks in different The whole business model is loans.
phases. Another entity — India premised on the idea that a bank 4. Which of the following
c. NPA refers to a
Debt Resolution Company Ltd will earn more money from submitted a report for
classification for loans
(IDRCL), which has also been extending loans to borrowers setting up a bad bank to
or advances that are in
set up — will then try to sell the than what it would have to pay resolve the NPA problem?
default or in arrears.
stressed assets in the market. The back to the depositors..
NARCL-IDRCL structure is the d. all of the above a. Niti Aayog
new bad bank. To make it work, 1. Which of the following b. Reserve Bank of India
the government has okayed the 2. Which of the following is
statement is correct c. Finance Ministry
use of Rs [1] crore to be used as a not one of the benefits of
regarding Bad Bank? d. Indian Banks’
guarantee. Bad Bank?
a. A bad bank is an Association
In every country, commercial a. This helps banks or FIs
Asset Reconstruction
banks accept deposits and clear-off their balance 5. Which of the following
Company (ARC) or
extend loans. The deposits are a sheets by transferring could be the reason for
an Asset Management
bank’s “liability” because that is the bad loans and focus NPA to occur?
Company (AMC) that
the money it has taken from a on its core business
takes over the bad a. Bad lending practice
common man, and it will have lending activities.
loans of commercial b. When incremental
to return that money when the banks, manages them b. It will help the
components like credit
depositor asks for it. Moreover, and finally recovers the government to settle
in the interim, it has to pay the card is affected.
money over a period of down the loan of
depositor an interest rate on farmers and wokers. c. When bank witness a
time. lower than average rate
those deposits. c. It can affect speedier
b. The bad bank is not of revenues and profit.
In contrast, the loans that banks settlements with
involved in lending and d. All of the above
give out are their “assets” because borrowers by cutting

1. d. all of the above of the loan and the bad bank settlements with borrowers by be paid from the Rs 30,600
The bad bank is unable to sell tries to recover as much as cutting out individual banks. crore that has been provided
the bad loan, or has to sell it possible subsequently. • It can drive a better bargain by the government.
at a loss, then the government 2. b. It will help the with borrowers and take more 4. d. Indian Banks’
guarantee will be invoked and government to settle down stringent enforcement action Association
the difference between what the loan of farmers and against them. In May 2020 the banking
the commercial bank was wokers. • It can raise money from sector, led by the Indian Banks’
supposed to get and what the Benefits: institutional investors rather Association, had submitted
bad bank was able to raise will • This helps banks or FIs clear- than looking only to the a proposal for setting up
be paid from the Rs 30,600 off their balance sheets by Government. a bad bank to resolve the
crore that has been provided transferring the bad loans 3. b. Rs 30600 crore NPA problem, proposing
by the government. and focus on its core business The bad bank is unable to sell equity contribution from the
The bad bank is not involved lending activities. the bad loan, or has to sell it government and banks.
in lending and taking deposits, • Large debtors have many at a loss, then the government 5. d. All of the above
but helps commercial banks creditors. Hence bad bank guarantee will be invoked and All of the reasons are correct
clean up their balance sheets could solve the coordination the difference between what regarding NPA to occur.
and resolve bad loans. problem, since debts would be the commercial bank was Other than this are More
The takeover of bad loans is centralised in one agency. supposed to get and what the competition in the market and
normally below the book value • It can effect speedier bad bank was able to raise will Environmental factors.
Bhopal +91-7676564400 Indore +91-9589613810, +91-731-4987379 Prayagraj +91-8114000926, +91-8114000927 Kanpur +91-8576066660, +91-8707605589 E: support@toprankers.com
Gurugram +91-8448444207 Delhi +91-9810610466 Ranchi +91-9334969993 Lucknow +91-6390576666 Jabalpur +91-7004386936 W: www.toprankers.com
30 The CLAT Post • September 2021

Passage 30 ceremony), baptizing

F ormer Uttarakhand chief Guru Gobind Singh and


minister Harish Rawat has about 80,000 others on
apologised for comparing Punjab the festival of Baisakhi.
Congress chief Navjot Sidhu c. All five Panj Piare
and his four advisers to the Panj fought beside Guru
Piare. On Wednesday, he not Gobind Singh and the
only rendered his apology but Khalsa in the siege
also promised to do sewa at a of Anand Purin and
gurdwara in his state as a form of helped the guru to
penance. escape from the battle of
Any baptised Sikh can become Chamkaur in December
a Panj Piare. The unanimous the Shiromani Akali Dal-Badal, founded? 1704.
decision taken by Panj Piare has and added to the long list of d. All of the above
a. 1665
to be followed by everyone in the allegations, which the party
community. Akal Takht Jathedar b. 1667 4. The First Rendition of the
was already facing for allegedly
also can’t take any decision demeaning Sikh institutions, c. 1697 Guru Granth Sahib was
unilaterally and every diktat from traditions and concepts. SAD d. 1699 completed under which of
the Akal Takht has to be signed could secure only 15 seats in the the following Guru?
by all the five Jathedars of the five following elections in 2017. 3. Which of the following a. Guru Arjan
Takhts (Temporal seats) or their Source: : Extracted with edits and Statement is TRUE about
representatives. the contribution of Panj b. Guru Nanak
revisions from news-article of
There was huge controversy Piare? c. Guru Angad
Hindustan Times.
when the SGPC-appointed Panj a. These spiritual warriors d. Guru Gobind
Piare had summoned then Akal 1. the men who were initiated vowed not only to fight
Takht Jathedar Giani Gurbachan into the Khalsa under the adversaries on the 5. Who among the following
Singh over the issue of pardon leadership of which of the battlefield but to combat is not one of the original
granted to Dera Sacha Sauda following Sikh Gurus? the inner enemy, five panj piare in the Sikh
head Gurmeet Ram Rahim in egoism, with humility history?
a. Guru Angad
2015. through service to a. Bhai Himmat Singh
b. Guru Ram Das
The SGPC was in a fix and it humanity and efforts to b. Bhai Muhkam Singh
used its powers to sack the Panj c. Guru Har Krishan
abolish caste. c. Bhai Sahib Singh
Piaras from the job. However, d. Guru Gobind Singh
b. They performed d. Bhai Bahadur Singh
mishandling of this crisis by the original Amrit
the SGPC reflected poorly on 2. In which of the following
years was the Panj Piare Sanchar (Sikh initiation

1. d. Guru Gobind Singh These spiritual warriors vowed followed by everyone in the “bani” of the ninth guru, Guru
‘Panj Piare’ is not just a group not only to fight adversaries on community. Teg Bahadur, as well into
of five baptised people but a the battlefield but to combat Akal Takht Jathedar also can’t the Adi Granth and formally
concept and tradition founded the inner enemy, egoism, take any decision unilaterally installed it as a “guru” at the
by 10th Sikh Guru Gobind with humility through service and every diktat from the Akal Takht Damdama Sahib in
Singh. Guru Gobind Singh to humanity and efforts to Takht has to be signed by all 1708.
established the institution of abolish caste. the five Jathedars of the five 5. d. Bhai Bahadur Singh
Panj Piare while founding the They performed the original Takhts (Temporal seats) or The original five beloved Panj
Khalsa on the day of Baisakhi Amrit Sanchar (Sikh initiation their representatives. Piare who played a vital role
in 1699. Addressing a large ceremony), baptizing Guru 4. a. Guru Arjan in shaping Sikh history and
gathering, he asked for five Gobind Singh and about The Adi Granth, its first defining Sikhism are:
heads for sacrifice. Five men 80,000 others on the festival of rendition, was compiled by Bhai Daya Singh (1661 - 1708
responded to his call and the Baisakhi in 1699. the fifth guru, Guru Arjan CE) from Lahore
Guru baptised them and called All five Panj Piare fought (1564–1606). Its compilation Bhai Dharam Singh (1699 -
them Panj Piare. beside Guru Gobind Singh was completed on 29 August 1708 CE) from Hastinapur
2. d. 1699 and the Khalsa in the siege of 1604 and first installed inside Bhai Himmat Singh (1661 -
Guru Gobind Singh Anand Purin and helped the Golden Temple in Amritsar on 1705 CE) from Jagannath Puri
established the institution of guru to escape from the battle 1 September 1604. The Guru Bhai Muhkam Singh (1663 -
Panj Piare while founding the of Chamkaur in December Granth Sahib, in its present 1705 CE) from Dwarka
Khalsa on the day of Baisakhi 1704. form, was compiled by Guru Bhai Sahib Singh (1662 - 1705
in 1699. The unanimous decision Gobind Singh, the last Sikh CE) from Bidar
3. d. All of the above taken by Panj Piare has to be guru, who incorporated the
Bhopal +91-7676564400 Indore +91-9589613810, +91-731-4987379 Prayagraj +91-8114000926, +91-8114000927 Kanpur +91-8576066660, +91-8707605589 E: support@toprankers.com
Gurugram +91-8448444207 Delhi +91-9810610466 Ranchi +91-9334969993 Lucknow +91-6390576666 Jabalpur +91-7004386936 W: www.toprankers.com
31 The CLAT Post • September 2021

Bhopal +91-7676564400 Indore +91-9589613810, +91-731-4987379 Prayagraj +91-8114000926, +91-8114000927 Kanpur +91-8576066660, +91-8707605589 E: support@toprankers.com
Gurugram +91-8448444207 Delhi +91-9810610466 Ranchi +91-9334969993 Lucknow +91-6390576666 Jabalpur +91-7004386936 W: www.toprankers.com
32 The CLAT Post • September 2021

Bhopal +91-7676564400 Indore +91-9589613810, +91-731-4987379 Prayagraj +91-8114000926, +91-8114000927 Kanpur +91-8576066660, +91-8707605589 E: support@toprankers.com
Gurugram +91-8448444207 Delhi +91-9810610466 Ranchi +91-9334969993 Lucknow +91-6390576666 Jabalpur +91-7004386936 W: www.toprankers.com
33 The CLAT Post • September 2021

Bhopal +91-7676564400 Indore +91-9589613810, +91-731-4987379 Prayagraj +91-8114000926, +91-8114000927 Kanpur +91-8576066660, +91-8707605589 E: support@toprankers.com
Gurugram +91-8448444207 Delhi +91-9810610466 Ranchi +91-9334969993 Lucknow +91-6390576666 Jabalpur +91-7004386936 W: www.toprankers.com
34 The CLAT Post • September 2021

Bhopal +91-7676564400 Indore +91-9589613810, +91-731-4987379 Prayagraj +91-8114000926, +91-8114000927 Kanpur +91-8576066660, +91-8707605589 E: support@toprankers.com
Gurugram +91-8448444207 Delhi +91-9810610466 Ranchi +91-9334969993 Lucknow +91-6390576666 Jabalpur +91-7004386936 W: www.toprankers.com
35 The CLAT Post • September 2021

Bhopal +91-7676564400 Indore +91-9589613810, +91-731-4987379 Prayagraj +91-8114000926, +91-8114000927 Kanpur +91-8576066660, +91-8707605589 E: support@toprankers.com
Gurugram +91-8448444207 Delhi +91-9810610466 Ranchi +91-9334969993 Lucknow +91-6390576666 Jabalpur +91-7004386936 W: www.toprankers.com
36 The CLAT Post • September 2021

ONE LINERS
1) Nine new judges, including The Ramon Magsaysay announced. The University was launched as a part of
three women, have been Award is Asia’s highest of Oxford is at the top of the the week-long activities
administered the oath honor and is widely regarded list. The Indian Institute of and campaigns organised
as the region’s equivalent of Science (IISc), Bengaluru, by the Ministry of Ayush
the Nobel Prize. It celebrates is placed between the 301- to commemorate Azadi
the memory and leadership 350 position. Incidentally, it ka Amrit Mahotsav from
example of the third is also the only university in August 30 to September 5,
Philippine president after the top 350 rankings. 2021.
whom the award is named
Three Indian educational 7) The Maharashtra
3) Ministry of Road Transport institutes have found a place Government has decided to
of office as judges of the
& Highways has introduced in the top 400 list: set up a world-class science
Supreme Court by Chief
Justice of India (CJI) NV 1. IISc Bangalore – 301-350
Ramana. The strength of the bracket
Supreme Court would go up
2.
Indian Institute of
to 33, including the CJI, out
Technology (IIT) Ropar –
of the sanctioned strength of
351-400 bracket.
34, after the swearing-in of
the nine new judges. Three 3. IIT Indore – 401-500
city at Pimpri-Chinchwad
out of these nine new judges bracket.
a new registration mark for near Pune to develop a
— Justice Vikram Nath and 5)
Raksha Mantri, Rajnath
new vehicles i.e. “Bharat scientific outlook among
Justice BV Nagarathna and Singh visited the Army
series (BH-series)”. The the students and prepare
Justice PS Narasimha — are
vehicles bearing the BH- them to become students
in line to become the chief
series mark will not have in future. To be named as
justice of India.
to require the assignment ‘Bharat Ratna Rajiv Gandhi
2) Ramon Magsaysay Award of a new registration mark Science Innovation City’,
2021 awardees have been when the owner of the a science Centre will be
announced, the winners vehicle shifts from one developed on one acre area
will be formally awarded the State to another. Earlier, in an eight-acre complex in
Magsaysay Award during under section 47 of the the PCMC area. The Centre
Motor Vehicles Act, 1988, Sports Institute (ASI), has granted an assistance
if a person moves from Pune and named the Army of Rs 191 crore to build the
one state to another, a Sports Institute stadium as Science city in the PCMC
person is allowed to keep “Neeraj Chopra Stadium”. area.
the vehicle for not more The focus of the Indian
8) The International Monetary
than 12 months in any state Army (in the field of sports)
Fund (IMF) has made
other than the state where is to identify and train
the November 28 event the vehicle is registered, promising sportspersons in
at the Ramon Magsaysay but a new registration with 11 disciplines
Centre in Manila. Among the new stateregistering 6) The Union AYUSH Minister,
the awardees is Dr Firdousi authority has to be made Sarbananda Sonowal has
Qadri of Bangladesh within the stipulated time of launched ‘Y Break’ Yoga
and Muhammad Amjad 12 months. protocol mobile application
Saqib of Pakistan,
4) The Times Higher Education
as well as Philippine
(THE) World University
fisheries and community
an allocation of Special
environmentalist Roberto
Drawing Rights (SDR) 12.57
Ballon, American citizen
billion to India. (approx
Steven Munsi, who works
USD 17.86 billion). With
for humanitarian work in New Delhi. The app has this, the total SDR holding
and help refugees, and been developed by Morarji of India has gone upto SDR
Indonesian WatchDoc for Desai National Institute of 13.66 billion (equivalent to
investigative journalism. Yoga (MDNIY). The app
Rankings 2022 has been around USD 19.41 billion).
Bhopal +91-7676564400 Indore +91-9589613810, +91-731-4987379 Prayagraj +91-8114000926, +91-8114000927 Kanpur +91-8576066660, +91-8707605589 E: support@toprankers.com
Gurugram +91-8448444207 Delhi +91-9810610466 Ranchi +91-9334969993 Lucknow +91-6390576666 Jabalpur +91-7004386936 W: www.toprankers.com
37 The CLAT Post • September 2021
9) The Assam cabinet has initiated the negotiation year allowed at 12. percent. In India, economic
decided to remove the to expand its membership. activity is picking up
14)
A Book Titled Know
name of former Prime As a result UAE, Uruguay
Your Rights: and Claim
Minister Rajiv Gandhi and Bangladesh have been
Them Jointly written
admitted as the first new
by Angelina Jolie and
member countries of the
Amnesty International with
NDB.
Geraldine Van Bueren QC.
12) National Mission for Clean The book is a guide for every
Ganga (NMCG) signed child and young person
an MoU with South Asian who believes in liberty,
alongside a gradual easing
Institute for Advanced equality. The Book deals
from the Orang National of restrictions that were
Research and Development with from gender and racial
Park. Since the name Orang implemented in response to
(SAIARD) to create an equality, to the rights to free
is associated with the the second wave. And there
expression, health, a clean
sentiments of the Adivasi is further upside to growth
climate and a sustainable
and teatribe community, as economies around the
environment.
the cabinet has decided to world progressively reopen.
rename the Rajiv Gandhi 15) Information and
18)
Dorje Angchuk, an
Orang National Park as Broadcasting Minister
Engineer in charge at
Orang National Park. Anurag Thakur presented
the Indian Astronomical
10) Sri Lanka has declared a Observatory of the Indian
state of emergency as the
food crisis worsened after academic platform to
private banks ran out of engage young students as
foreign exchange to finance well as develop a research
and capacity building
hub to support regional
development in the eastern
region. Both NMCG and Institute of Astrophysics
SAIARD will focus on (IIA) at Hanle in the Ladakh
the first copy of the
geospatial technology in region, has been admitted as
book titled Reflecting,
riverine ecosystems. an Honorary Member of the
Recollecting, Reconnecting
imports. With the country International Astronomical
13) The Reserve Bank of India to the Vice President M
suffering a hard-hitting Union (IAU). Angchuk is
has increased the limit of Venkaiah Naidu. The book
economic crisis, President the first and only Indian
fund transfer under the chronicles Venkaiah Naidu’s
Gotabaya Rajapaksa said to bag the position in the
Indo-Nepal Remittance fourth year in office as the
he ordered emergency prestigious body.
Vice President of India.
regulations to counter the
19)
Vartika Shukla took
hoarding of sugar, rice and 16) America based Investment
charge as the first woman
other essential foods. The Bank, Morgan Stanley has
Chairperson and Managing
Sri Lankan rupee has fallen maintained the GDP growth
Director of public sector
by 7.5% against the US forecast of India at 10.5 per
enterprise Engineers India
dollar this year. cent for the financial year
Ltd. She spearheaded the
2021-22 (FY2022). GDP
11) The Shanghai-based New company’s innovative
growth is expected to move
Development Bank (NDB) Facility Scheme from Rs energy programmes,
into positive territory on a
has approved the United 50,000 per transaction to including biofuels, coal
two-year CAGR basis from
Arab Emirates, Uruguay Rs 2 lakh per transaction. gasification, waste to fuel
QE September.
Earlier there was a ceiling
limit of 12 transactions in a 17)
Moody’s Investors Service
year. Now, this limit has also has retained India’s growth
been removed. However, for forecast for the calendar
cash-based transfers under year (CY) 2021 at 9.6 per
the Indo-Nepal Remittance cent, in its August update
Facility, the per transaction to ‘Global Macro Outlook
and Bangladesh as its new limit of Rs 50,000 will still 2021-22’ report. The GDP
member countries. In 2020, be present with a maximum growth forecast for calendar and hydrogen energy,
the NDB Board of Governors number of transfers in a year 2022 is retained at 7 among others.
Bhopal +91-7676564400 Indore +91-9589613810, +91-731-4987379 Prayagraj +91-8114000926, +91-8114000927 Kanpur +91-8576066660, +91-8707605589 E: support@toprankers.com
Gurugram +91-8448444207 Delhi +91-9810610466 Ranchi +91-9334969993 Lucknow +91-6390576666 Jabalpur +91-7004386936 W: www.toprankers.com
38 The CLAT Post • September 2021
20)
IRS officer JB Mohapatra companies are participating India Future Unicorn List
was appointed as the in the EEF Summit 2021 in 2021, as per which India is
chairman of the Central Russia. the third-largest Unicorn/
Board of Direct Taxes. His startup ecosystem in the
26) Amul, Gujarat Cooperative
appointment was approved
Milk Marketing Federation
(GCMMF) has dropped
two places to rank 18th in
was represented by frigate Rabobank’s 2021 Global
“Trikand” while the German Top 20 Dairy Companies
Navy was represented by list. Amul was placed at
frigate “Bayern”. 16th spot in 2020. Amul
24)
Raksha Mantri Rajnath
today by the Appointments Singh has dedicated to the world. The USA is on
Committee of the Cabinet nation, the indigenously top followed by China
(ACC). He has already built Coast Guard Ship on second. India has 51
served as the acting ‘Vigraha’ in Chennai, unicorns according to the
chairman of the CBDT. Tamil Nadu. According list. The USA has 396 and
to the defence ministry, China has 277 unicorns.
21)
Pankaj Kumar Singh, a
Zilingo is the top unicorn
1988-batch IPS officer from has achieved an annual
in the ranking in India with
the Rajasthan cadre, has turnover of $5.3 billion.
funding of USD 310. The
taken charge as the new Frenchbased dairy company
headquarters of Zilingo is
Director General (DG) of Lactalis has topped the list
in Singapore. Bengaluru is
the Border Security Force as the world’s largest dairy
the top Indian city in terms
(BSF). Prior to this, he was company with a turnover
of the number of Unicorns
the 98-meter vessel will be of US $23.0 billion. It has
headquartered in a city.
based in Andhra Pradesh’s unseated Switzerland-based
Visakhapatnam (Vizag) global behemoth Nestle, 29) Max Verstappen (Red Bull
and will be operated by a which dominated the list for – Netherlands) has been
company of 11 officers and decades. declared the winner of the
110 sailors. The ship has 27)
Indian Institutes of
been designed and built Technology (IIT), Ropar
indigenously by Larsen and Kanpur and Faculty
serving as the special DG & Toubro Ship Building of Management Studies
at the BSF headquarters in Limited. With the joining
Delhi. of this ship, the Indian
22) Sanjay Arora, a 1988-batch Coast Guard now have 157
Ships and 66 Aircraft in its Belgian Grand Prix 2021.
IPS officers from Tamil
inventory. The Belgian Grand Prix
was stopped due to rain
25)
Prime Minister Narendra and only two laps were
Modi virtually addressed completed. The winner was
the plenary session of the decided based on progress
made in these two laps.
of Delhi University have
George Russell Williams
jointly launched a living-
came second.
plant based air purifier
Nadu cadre, has taken named “Ubreathe Life”. 30)
Shooter Avani Lekhara
over as the new DG of the This air purifier will boost has scripted history as she
Indo-Tibetan Border Police the air purification process
(ITBP). 6th Eastern Economic in indoor spaces, such as
Forum (EEF), organised hospitals, schools, offices
23)
The Indian Navy and the
in Vladivostok, Russia. and Homes. It is the world’s
German Navy carried out
An Indian delegation led first, state-of-the-art ‘Smart
a joint exercise in the Gulf
by Minister of Petroleum Bio-Filter’ that can make
of Aden near Yemen, in
and Natural Gas, Hardeep breathing fresh.
the Indian Ocean leg of
Indo-Pacific Deployment Singh Puri and comprising 28)
Hurun Research Institute
leading Indian oil and gas became the first Indian
2021. The Indian Navy has released the Hurun
Bhopal +91-7676564400 Indore +91-9589613810, +91-731-4987379 Prayagraj +91-8114000926, +91-8114000927 Kanpur +91-8576066660, +91-8707605589 E: support@toprankers.com
Gurugram +91-8448444207 Delhi +91-9810610466 Ranchi +91-9334969993 Lucknow +91-6390576666 Jabalpur +91-7004386936 W: www.toprankers.com
39 The CLAT Post • September 2021
woman to win a gold medal diaspora around the world International Society for governor of Uttarakhand, a
at the Paralympics, firing Krishna Consciousness post which fell vacant after
her way to the top of the Baby Rani Maurya tendered
podium in the R-2 women’s her resignation. According
10m Air Rifle Standing SH1 to a Rashtrapati Bhavan
event. communique, President
Ram Nath Kovind accepted
31)
India’s Nishad Kumar has
the resignation of Maurya
clinched a silver medal
and appointed Singh as the
in the men’s high jump
and to eliminate all forms (ISKCON) commonly governor of the state.
of discrimination against known as the ‘Hare Krishna
39)
Prime Minister Narendra
people of African descent. movement’. The spiritual
Modi has inaugurated the
leader was born as Abhay
34)
The world’s highest movie “Shikshak Parv-2021” and
Charan De on September
theatre has recently been addressed the inaugural
1, 1896, in Calcutta and
inaugurated in Ladakh, conclave through a video
later came to be known
which got its first-ever conference. The theme of
by the honorific A.C.
T47 event at the Tokyo mobile digital movie theatre
Bhaktivedanta Swami
Paralympics 2020. This in the Paldan area of Leh,
Prabhupada.
is the second medal for at an altitude of 11,562
India at the Tokyo 2020 feet. The inflatable theatre 37)
The Union Minister of
Paralympics. The 23-year- can operate in -28 degrees Education, Dharmendra
old Nishad made a jump Celcius. The initiative aims Pradhan released the
of 2.06m, and in doing so to bring cinema watching NIRF India Rankings
created an Asian record. He experience to most remotest 2021, on September 09, ‘Shikshak Parv-2021’ is
also equalled his jump with areas of India. It must be 2021, through video “Quality and Sustainable
USA’s Dallas Wise, who also noted that four such theatres conferencing. The NIRF Schools: Learnings from
took home silver. will be established in Leh in Schools in India”. The
the coming period. event was organised by the
32)
Every year, 29th August
Ministry of Education, will
has been observed as 35)
The Union Territory of
continue to be celebrated
National Sports Day in Ladakh has declared snow
from September 07 to 17,
leopard (Panther unica) as
2021.
the new state animal and
black-necked crane (Grus 40) India has become the first
nicricollis) as the new state country in Asia to launch a
India Rankings 2021 is the
Plastics Pact, a new platform
sixth edition of the annual
to promote a circular system
list which ranks higher
educational institutions
India. The first National
in the country based on
Sports Day was celebrated
objective criteria to promote
on 29th August 2012, on
competitive excellence. In
the birth anniversary of
Overall Winner: The Indian
Major Dhyan Chand who
Institute of Technology
was the star of hockey team bird. The notification in this
(IIT) Madras has retained
of India. Recently Prime regard was released by Shri
the top spot in the overall
Minister Narendra Modi Radha Krishna Mathur, for plastics. The India
category
has announced that Rajiv Lieutenant Governor of Plastic Pact platform was
Gandhi Khel Ratna Award Union Territory of Ladakh 38) President Ram Nath Kovind launched on September 03,
will be renamed as Major on August 31, 2021. has appointed Lt Gen 2021, by the British High
Dhyan Chand Khel Ratna Commissioner to India,
36)
Prime Minister Narendra
Award. Alexander Ellis, at the 16th
Modi virtually unveiled a
Sustainability Summit
33)
The International Day for special commemorative
hosted by the Confederation
People of African Descent coin of Rs 125 to mark the
of Indian Industry (CII).
is celebrated for the first 125th birth anniversary
time on 31 August 2021. of ISKCON founder Srila 41)
Tamil Nadu state
The United Nations aims to Bhaktivedanta Swami government has announced
Gurmit Singh, who retired
promote the extraordinary Prabhupada. In July 1966, to set up India’s first dugong
as the deputy chief of Army,
contributions of the African Prabhupada founded the conservation reserve at
Bhopal +91-7676564400 Indore +91-9589613810, +91-731-4987379 Prayagraj +91-8114000926, +91-8114000927 Kanpur +91-8576066660, +91-8707605589 E: support@toprankers.com
Gurugram +91-8448444207 Delhi +91-9810610466 Ranchi +91-9334969993 Lucknow +91-6390576666 Jabalpur +91-7004386936 W: www.toprankers.com
40 The CLAT Post • September 2021
the northern part of the 44) The Ministry of Defence, the 46) Union Minister of Culture RBI. Financial Stability
Palk Bay. Dugong are Government of India and Shri Kishan Reddy and Development
US Department of Defence Gangapuram has launched Council (FSDC) is the
have signed a Project a book titled “Gita Govinda: apex level forum set up
Agreement (PA) for Air- Jaydeva’s Divine Odyssey” by the Government of
Launched Unmanned Aerial India for strengthening
Vehicle (ALUAV). This and institutionalizing
agreement is signed under the mechanism for
maintaining financial
stability, enhancing inter-
commonly known as sea regulatory coordination and
cows. According to Wildlife authored by Dr Utpal K. promoting financial sector
Institute of India (WII) Banerjee. The book is the development.
estimates, only 200-250 first-ever fullyrhymed
49)
Asian Development Bank
Dugongs are left in the wild, translation of the book
(ADB) and the Government
of which 150 are found in the Joint Working Group GeetaGovindam by great
of India have signed a USD
the Palk Bay and Gulf of Air Systems in the Defence 12th-century poet Jayadeva.
112 million loan to develop
Mannar in Tamil Nadu. Technology and Trade
47)
S&P Global Ratings has
Initiative (DTTI). Under
42) China successfully launched revised India’s growth
the collaboration, the two
a new Earth observation forecast and now expects
countries will work towards
satellite, Gaofen-5 02, the economy to grow 9.5
the design, development,
into space, aboard a Long
demonstration, testing
and evaluation of systems
to co-develop an ALUAV
Prototype.
45)
Japanese Prime Minister water supply infrastructure
Yoshihide Suga would step and strengthen capacities of
down, setting the stage urban local bodies (ULBs)
for a new premier after a for improved service
March-4C rocket, from the oneyear tenure marred by per cent in 2021-22 (FY22) delivery in four towns in the
Taiyuan Satellite Launch an unpopular COVID-19 and 7.0 per cent in 2022- state of Jharkhand. This is
Centre in north China’s response and rapidly 23 (FY23). India’s external the first project of ADB in
Shanxi Province. The position is very strong and the state of Jharkhand.
Gaofen-5 02 satellite is the this is quite supportive of
50)
Former IPS officer Iqbal
24th in the series of Gaofen India’s sovereign rating
Singh Lalpura has been
Earthobservation satellites despite the fact that we have
of China, to monitor had this deterioration in
environmental protection fiscal position concurrently.
efforts and boost its natural
48) Union Minister for Finance
resources surveillance. dwindling public support.
& Corporate Affairs Smt.
Suga, who took over after
43) El Salvador has become the Nirmala Sitharaman has
Shinzo Abe resigned last
first country in the world chaired the 24th meeting
September, citing ill health,
of the Financial Stability appointed chairman of the
has seen his support ratings
and Development Council National Commission for
sink to below 30% as the
Minorities. He hails from
nation struggles with its
Punjab and has authored
worst wave of COVID-19
several books on Sikh
infections ahead of a
philosophy. He has also won
general election this year.
several awards such as the
Suga’s decision to not run in
to accept Bitcoin as legal president’s police medal,
a ruling Liberal Democratic
tender. The government of police medal for meritorious
Party (LDP) election in
El Salvador claimed that (FSDC). Finance Minister services, Shiromani Sikh
September means the party
the move will give many is the chairperson of FSDC. Sahitkar Award and Sikh
will choose a new leader,
citizens of the country It must be noted that scholar award.
who will become prime
access to bank services for FSDC Sub-Committee is
minister. 51)
The 28th edition of
the first time. chaired by the Governor,
the Singapore-India
Bhopal +91-7676564400 Indore +91-9589613810, +91-731-4987379 Prayagraj +91-8114000926, +91-8114000927 Kanpur +91-8576066660, +91-8707605589 E: support@toprankers.com
Gurugram +91-8448444207 Delhi +91-9810610466 Ranchi +91-9334969993 Lucknow +91-6390576666 Jabalpur +91-7004386936 W: www.toprankers.com
41 The CLAT Post • September 2021
Maritime Bilateral Exercise adopted and implemented Clean Air for blue skies societies. The theme of
various energy conservation is observed globally on 55th International Literacy
measures. GOC Workshop September 07 to promote Day is Literacy for a
is the only workshop of human-centred recovery:
Indian Railways to have Narrowing the digital
received the award this year. divide.
54)
Prime Minister of India, 59)
The National Mission for
Narendra Modi has Clean Ganga organized
(SIMBEX) took place from
chaired the 13th BRICS Himalayan Diwas on
September 02 to 04, 2021.
Summit, through video September 09, 2021, in
The SIMBEX-2021 annual and facilitate actions
conferencing. The theme association with Naula
bilateral maritime exercise to improve air quality.
Foundation. This year’s
was hosted by the Republic The 2021 theme for the
theme is ‘Contribution
of Singapore Navy (RSN) in International Day of Clean
of Himalayas and our
the southern fringes of the Air for blue skies is “Healthy
responsibilities’. The event
South China Sea. Air, Healthy Planet” which
was part of the ongoing
emphasizes the health
52)
The 4th edition of celebration of ‘Azadi ka
aspects of air pollution,
AUSINDEX, a bilateral Amrit Mahotsav’.
especially considering the
navy exercise between
COVID-19 pandemic. 60)
The Chandigarh Railway
the Indian Navy and the of the India-led Summit
Station(CRS) was awarded
was “BRICS@15: Intra- 57) To commemorate 75 years
a 5 star ‘Eat Right Station’
BRICS Cooperation for of India’s independence,
certification for providing
Continuity, Consolidation the Government of India
and Consensus.” The theme
chosen by India reflects
the fifteenth anniversary
of BRICS, being observed
Royal Australian Navy has in 2021. PM Modi called
begun from September 06, for enhanced BRICS
2021, and will continue cooperation under the
high-quality, nutritious
up to September 10, 2021. motto of ‘Build-back
food to passengers. The
Indian Navy Task Group Resiliently, Innovatively, is celebrating the ‘Azadi
certification was awarded
comprises INS Shivalik and Credibly and Sustainably’. Ka Amrit Mahotsav.’ As a
by the Food Safety and
INS Kadmatt. This edition part of the celebration, the
55) Portuguese captain Standards Authority of
of AUSINDEX includes Ministry of Food Processing
Cristiano Ronaldo has India (FSSAI) upon the
complex surface, sub- Industries is observing
broken the world record conclusion of the FSSAI-
surface and air operations ‘Food Processing Week’
for most goals scored in empanelled third-party
between ships, submarines, from 6th to 12th September
men’s international football. audit agency. CRS was
helicopters and Long Range 2021, under which, the
Ronaldo surpassed Iranian selected to upgrade on the
Maritime Patrol Aircraft of Ministry is organizing
striker Ali Daei’s long-held pattern of the World Class
the participating Navies. various programs.
Railway Station.
53) The Golden Rock Railway 58)
International Literacy Day
61) India’s tallest air purification
Workshop (GOC), is observed globally on
tower has been inaugurated
Tiruchchirappalli has 8th September every year.
in the Union Territory of
bagged 22nd National The day spread awareness
about the importance of
literacy for individuals,
record of 109 international
goals by scoring a brace
against Ireland in a World
Cup Qualifier. Ronaldo
at 36 years of age is now
Chandigarh. The tower
a Guinness world record
has been installed at the
Award for Excellence in holder of most international
initiative of the Chandigarh
Energy Management from goals of all time with 111 communities, and societies
Pollution Control
the Confederation of Indian goals. and the need for intensified
Committee (CPCC) at
Industry (CII) for having efforts towards more literate
56)
The International Day of Transport Chowk, Sector
Bhopal +91-7676564400 Indore +91-9589613810, +91-731-4987379 Prayagraj +91-8114000926, +91-8114000927 Kanpur +91-8576066660, +91-8707605589 E: support@toprankers.com
Gurugram +91-8448444207 Delhi +91-9810610466 Ranchi +91-9334969993 Lucknow +91-6390576666 Jabalpur +91-7004386936 W: www.toprankers.com
42 The CLAT Post • September 2021
26, by Pious Air Pvt Limited. magazine. The list is divided ecological role and promote other essential products to
into six categories – Icons, further research. remote areas using drones.
62)
India’s first indigenously
Pioneers, Titans, Artists, The Medicine from the Sky
Designed High Ash 69) Niti Aayog in collaboration
Leaders and Innovators. project will be taken up on
Coal Gasification Based with US-based Rocky
a pilot basis in 16 green
Methanol Production Plant 65) In the men’s category, Daniil Mountain Institute (RMI)
zones in Telangana and later
at Bharat Heavy Electricals and RMI India, has
scaled up on the national
launched a campaign named
level based on data.
Shoonya, to promote zero-
72) Nagaland’s first and India’s
61st Software Technology

Medvedev has lifted his first


Limited (BHEL) Research Grand Slam trophy after
and Development Centre, defeating Novak Djokovic pollution delivery vehicles
Hyderabad was inaugurated. 6-4, 6- 4, 6-4 in the US by working with consumers
The project was funded by Open men’s singles final at and industry. The campaign
the Department of Science the Arthur Ashe Stadium in will promote the adoption Park of India (STPI) centre
and Technology, which New York. of electric vehicles (EVs) was inaugurated at Kohima.
provided a Rs 10 crore in the urban deliveries The inauguration of the
66) Hindi Diwas or Hindi Day
grant, at the initiative of segment and create STPI centre in Kohima is a
is observed every year on
NITI Aayog, PMO-India consumer awareness about fulfilment of Prime Minister
14 September to mark
and the Ministry of Coal. the health, environmental Narendra Modi’s vision
the popularity of Hindi
and economic benefits of of creating a technology
63)
The world’s largest plant as an official language of
electric vehicles. ecosystem in the northeast
designed to capture carbon India. The language was
to create opportunities for
adopted under Article 343 70) Haryana has topped among
future generations in the
of the Indian Constitution. all other states of the country
region.
The first Hindi day in terms of installation
was celebrated on 14th 73) Aziz Akhannouch has been
September 1953. appointed as the new Prime
Minister of Morocco by the
67)
Bhupendra Patel has been
dioxide out of the air began country’s King Mohammed
chosen as the new Chief
its operations in Iceland. VI. Akhannouch’s National
The plant is named Orca, Rally of Independents
which means ‘Energy’ in (RNI) party secured 102
Icelandic word. It will suck of off-grid solar pumps out of the 395 seats in the
out up to 4,000 tons of CO2 under Pradhan Mantri parliament election, held
per year. Kisan Urja Suraksha Evam on September 10, 2021.
Utthan Mahaabhiyaan Prior to this appointment,
64)
TIME magazine has
(PM-KUSUM), as per the the 60-year-old was the
unveiled its annual list of
data by the Union Ministry Minister of Agriculture
‘The 100 Most Influential Minister of Gujarat at the
of New and Renewable from 2007 to 2021.
People of 2021’. Prime BJP legislature meeting.
Energy. Haryana has
He is a BJP MLA from the 74)
United States, United
installed 14,418 pumps
Ghatlodia assembly seat in Kingdom and Australia
against 15,000 sanctioned
Ahmedabad. This comes have announced the new
pumps for the year 2020-21.
after the resignation of Vijay
Haryana was given a target
Rupani from the post of CM
of 15,000 pumps for the year
of Gujarat.
2020-21 with a total cost of
68)
India’s largest open- Rs 520 crore.
Minister Narendra Modi,
air fernery has been
West Bengal Chief Minister 71)
Civil aviation minister
inaugurated in Ranikhet
Mamata Banerjee and Jyotiradtiya Scindia has
of Uttarakhand. The new
Serum Institute of India launched a first of its kind
centre will serve the dual
CEO Adar Poonawalla have “Medicine from the Sky” partnership “AUKUS”
objective of ‘conservation of
been named among the project in Telangana. a new trilateral security
fern species as well as ‘create
world’s 100 most influential The project has aimed at partnership for the Indo-
awareness about their
people of 2021 by TIME transporting vaccines and Pacific region. The trilateral
Bhopal +91-7676564400 Indore +91-9589613810, +91-731-4987379 Prayagraj +91-8114000926, +91-8114000927 Kanpur +91-8576066660, +91-8707605589 E: support@toprankers.com
Gurugram +91-8448444207 Delhi +91-9810610466 Ranchi +91-9334969993 Lucknow +91-6390576666 Jabalpur +91-7004386936 W: www.toprankers.com
43 The CLAT Post • September 2021
grouping was formally PEACEFUL MISSION at the Hyderabad House Council of Heads of State
announced by British Prime -2021, a Joint Counter in New Delhi. The high- was held in hybrid format
Minister Boris Johnson, Terrorism Exercise between level foreign and defence on September 17, 2021, in
his Australian counterpart Shanghai Cooperation ministerial-level dialogue Dushanbe, Tajikistan. The
Scott Morrison and US Organisation (SCO) will further ramp up meeting was held under
President Joe Biden with a member states. the overall defence and the Chairmanship of the
vision for a safer and more strategic cooperation President of Tajikistan,
78) The 15th edition of Indo–
secure Indo-Pacific. between the two countries, Emomali Rahmon. This was
Nepal Joint Military
including in the Indo- the first SCO Summit held
75) A book has titled ‘Human Training Exercise Surya
Pacific cooperation amid in a hybrid format and the
Rights and Terrorism in Kiran will be held from
China’s increasing military fourth Summit in which
India’ authored by BJP September 20, 2021, at
assertiveness in the region. India participated as a full-
MP Subramanian Swamy. Pithoragarh in Uttarakhand.
fledged member of SCO.
He has come out with a The previous edition of the 82)
India and the United
book “Human Rights and Exercise Surya Kiran was 84)
The Indian Institute
Terrorism in India” which conducted in Nepal in 2019. of Technology (IIT)
traces how the combating The exercise was called off
of terrorism can be in 2020, due to the Covid-19
harmonised with human and pandemic.
fundamental rights within
79) Mathematician Anand
reasonable restrictions
Kumar was conferred with
that are permitted by the States of America (USA)
the Swami Brahmanand
Constitution and upheld by have launched the
Award 2021 for his
the Supreme Court. “Climate Action and Bombay has launched
contribution in the field
Finance Mobilization ‘Project Udaan’, a language
76)
India’s first nuclear- of education through his
Dialogue (CAFMD)”. It translator, to break the
missile tracking ship, ‘Super 30’ initiative, which
will strengthen India-US language barrier in
named INS Dhruv, has prepares underprivileged
education, which hampers
students for the IIT entrance
the flow of messages. The
exam. He received the
Project Udaan, a donation-
award from professor Roop
based project, is an end-
Kishore Shastri, the vice-
to-end ecosystem, which
chancellor of Haridwar’s
can translate scientific and
Gurukula Kangri deemed
technical content from
university, at a function in
English to Hindi and all
the Rath area of Hamirpur
other Indian languages.
been commissioned from district of Uttar Pradesh.
bilateral cooperation on
Visakhapatnam in Andhra 85)
Union Minister for
80)
The Florence Nightingale climate and environment.
Pradesh. The 10,00 tonnes Panchayati Raj and Rural
Award will be given The dialogue was launched
satellite and ballistic missile Development Giriraj Singh
to a nurse, Bhanumati by Union Minister of
tracking ship has been built
Gheewala, from Sir Environment, Forest and
by the Hindustan Shipyard
Sayajirao General Hospital, Climate Bhupender Yadav
Limited in collaboration
Gujarat. She has been and Mr John Kerry, U.S.
with the DRDO and
in charge of COVID-19 Special Presidential Envoy
National Technical Research
positive pregnant women’s for Climate (SPEC) in New
Organisation (NTRO).
deliveries as well as infant Delhi.
77)
The Indian military care. She worked in the
contingent comprising gynaecology department
inaugurated a national
of an all arms combined as well in the paediatric
meet on SVAMITVA
force of 200 personnel is ward. In 2019, when the
Scheme: A Stepping-Stone
wards of the hospital were
Towards Upliftment of
waterlogged due to flood.
Rural Economy in New
She carried her duty in the
Delhi. SVAMITA is aimed
gynaecology department
to provide an integrated
and the pediatric ward.
property validation solution
81)
India and Australia are 83)
The 21st Meeting of the for rural India leveraging
undertaking their first-ever Shanghai Cooperation property as a financial asset
2+2 ministerial dialogue Organisation (SCO) by the citizens with accurate
participating in the Exercise
Bhopal +91-7676564400 Indore +91-9589613810, +91-731-4987379 Prayagraj +91-8114000926, +91-8114000927 Kanpur +91-8576066660, +91-8707605589 E: support@toprankers.com
Gurugram +91-8448444207 Delhi +91-9810610466 Ranchi +91-9334969993 Lucknow +91-6390576666 Jabalpur +91-7004386936 W: www.toprankers.com
44 The CLAT Post • September 2021
land records for rural States across five parameters and business sophistication. Tamenglong mandarin
planning. States to complete of food safety. The Minister orange have been granted
90)
Assam is setting up a tea
the implementation of felicitated nine leading the Geographical Indication
Park at Chayygaon in
SVAMITVA scheme well States/UTs based on the (GI) tag. This is a historic
before the deadline 2024. ranking for the year 2020- milestone in the history of
21 for their impressive Manipur and it will increase
86)
International Day of
performance. Minister also the income of farmers in
Democracy is observed
flagged off 19 Mobile Food Manipur.
globally on 15th September
Testing Vans (Food Safety
every year. It was established 94)
The Government of
on Wheels) to supplement
through a resolution passed Sikkim has declared
the food safety ecosystem
by the UN General Assembly
across the country taking
in 2007, for promoting and
the total number of such
upholding the principles
mobile testing vans to
of democracy and provide Kamrup District. This tea
109. Here is the list of nine
an opportunity to review garden will have rail and
leading States/UTs in the
the state of democracy port connectivity, cargo
Index:
and warehouse amenities,
Larger States: • Gujarat • processing facilities like ‘Cooper Mahseer’ locally
Kerala • Tamil Nadu tea grinding, blending,
packaging and other utility
Smaller States: • Goa •
Meghalaya • Manipur
Among UTs: • Jammu and
in the world. The theme Kashmir, • Andaman and
for 2021 International Nicobar Islands • New Delhi
Day of Democracy is
89)
India has been ranked
“Strengthening democratic named as ‘Katley’ as the
at 46th place in the
resilience in the face of services under one roof state fish. Neolissochilus
Global Innovation Index
future crises”. hexagonolepis is the
2021 released by World 91) The world’s highest electric
scientific name of the
87)
Double Olympic hammer Intellectual Property vehicle charging station
Cooper Mahseer.
throw gold medallist Organization (WIPO). has been inaugurated in
Yuriy Sedykh, a Ukrainian India has jumped 2 Kaza village of Lahaul and 95) Niti Aayog’s Atal Innovation
track and field athlete spots from last year’s Spiti district in Himachal Mission in collaboration
who represented the ranking. Under the Lower Pradesh. The electric vehicle with ISRO and CBSE has
Soviet Union until 1991, middleincome category charging station is set up at launched ‘Space Challenge’
has passed away. He set group, India has been a height of 500 ft. for school students across
the world record for the ranked at the second spot India. This challenge has
92) Iran was officially admitted
hammer throw with a after Vietnam. The Global been designed for all the
as a full member of the
throw of 86.74 metres at the Innovation Index 2021 school students, mentors
Shanghai Cooperation
European Championships captures the innovation and teachers across the
Organization (SCO). The
in Stuttgart in 1986 which is ecosystem performance of
decision to admit Iran as a
still not broken. 132 economies and tracks
full member was declared
the most recent global
88) Union Minister for Health
innovation trends.
and Family Welfare,
About the Global
Innovation Index (GII):
The Global Innovation
country who not only are
Index (GII) provides
associated with schools
detailed metrics about the in the 21st summit of the
having Atal Tinkering Labs
innovation performance SCO leaders in Dushanbe,
(ATL) labs but for all the
of 132 countries and Tajikistan.
non-ATL schools as well.
Mansukh Mandaviya has economies around the
93)
Two famous products of
released the Food Safety world. Its 80 indicators 96) Vice Chief of the Indian Air
Manipur, Hathei chilli,
and Standards Authority explore a broad vision Force Air Marshal, Vivek
which is found in Manipur’s
of India (FSSAI)’s 3rd State of innovation, including Ram Chaudhari has been
Ukrul district and is known
Food Safety Index (SFSI) to political environment, appointed the next Chief
for its unique flavour, and
measure the performance of education, infrastructure of Air Staff. The current
Bhopal +91-7676564400 Indore +91-9589613810, +91-731-4987379 Prayagraj +91-8114000926, +91-8114000927 Kanpur +91-8576066660, +91-8707605589 E: support@toprankers.com
Gurugram +91-8448444207 Delhi +91-9810610466 Ranchi +91-9334969993 Lucknow +91-6390576666 Jabalpur +91-7004386936 W: www.toprankers.com
45 The CLAT Post • September 2021
ever contingent of 54 (Mixed 50m Pistol) be headed by National
paraathletes at Tokyo Education Policy-2020
• Table Tennis: Bhavina
Paralympics to compete (NEP-2020) drafting
Patel (Women’s Singles)
across as many as 9 sporting committee chairperson
disciplines at the Games. Bronze K Kasturirangan. The
• Archery: Harvinder Singh committee will discuss
• Prior to this, India
(Men’s Individual Recurve) “position papers” finalised
had won a total of 12
Chief of Air Staff Air Chief by national focus groups
Paralympics medals, since
• Athletics: Sharad Kumar on different aspects of the
Marshal RKS Bhadauria is making its first appearance
(Men’s High Jump) four areas drawing inputs
set to retire on September at the Paralympics in 1968,
30. Chaudhari had taken • Athletics: Sundar Singh from state curriculum
till the 2016 Rio.
over as the vice chief on July Gurjar (Men’s Javelin frameworks. The NCFs
1 this year, after Air Marshal • The Indian theme song of serve as a guideline for
HS Arora retired. He is Paralympics 2020 “Kar de syllabus, textbook and
a recipient of the Param kamaal tu”. The composer teaching practices for
Vishisht Seva Medal, Ati and singer of the song is schools in India.
Vishisht Seva Medal and the Sanjeev Singh, a Divyang
99) CISF official Geeta Samota
Vayu Sena Medals. cricket player from
became the “fastest Indian”
Lucknow.
97)
India finished their to summit two peaks located
campaign at the Tokyo List of Indian medallists in Africa and Russia. Earlier
winners of Tokyo this month, Sub Inspector
Paralympics 2020: Geeta Samota had scaled
Gold Mount Elbrus in Russia,
the highest peak in Europe.
• Athletics: Sumit Antil While Mt Elbrus (5,642 mts)
(Men’s Javelin Throw)
• Badminton: Pramod
Paralympics 2020 with an Bhagat (Men’s Singles)
all-time high of 19 medals
• Badminton: Krishna Throw)
which includes five gold,
Nagar (Men’s Singles)
eight silver, and six bronze. • Badminton: Manoj Sarkar
This is the best medal tally • Shooting: Manish Narwal (Men’s Singles)
for India a single edition (Mixed 50m Pistol) is in Russia, the Kilimanjaro
• Shooting: Singhraj Adhana peak (5,895 mts) is located
of the Paralympic Games.
• Shooting: Avani Lekhara (Men’s 10m Air Pistol) in Tanzania and is Africa’s
India is placed at 24th
(Women’s 10m Air Rifle highest peak.
position in the overall • Shooting: Avani Lekhara
Standing)
medal tally out of a total of (Women’s 50m Rifle 3 100) Two more beaches in
162 nations. Silver Positions) India have been awarded
Indian Flag Bearers: • Athletics: Yogesh 98)
The Union education “Blue Flag” certification, an
Kathuniya (Men’s Discus ministry has formed a
• Javelin thrower Tek Chand
Throw) 12- member committee to
• Athletics: Nishad Kumar
(Men’s High Jump)
• Athletics: Mariyappan
Thangavelu (Men’s High
Jump)
was the flag bearer at the • Athletics: Praveen Kumar international eco-level tag,
opening ceremony of the (Men’s High Jump) taking the total number of
Tokyo Paralympics. • Athletics: Devendra such beaches in the country
develop new curriculums
• Shooter Avani Lekhara Jhajharia (Men’s Javelin for school, early childhood, to 10. The two beaches to
was India’s flag-bearer at the Throw) teacher and adult receive the certification
closing ceremony. education. The panel this year are Kovalam in
• Badminton: Suhas Yathiraj Tamil Nadu and Eden in
(Men’s Singles) tasked with developing
India at Paralympics 2020: Puducherry.
four national curriculum
• India sent its biggest • Shooting: Singhraj Adhana frameworks (NCFs) will
Bhopal +91-7676564400 Indore +91-9589613810, +91-731-4987379 Prayagraj +91-8114000926, +91-8114000927 Kanpur +91-8576066660, +91-8707605589 E: support@toprankers.com
Gurugram +91-8448444207 Delhi +91-9810610466 Ranchi +91-9334969993 Lucknow +91-6390576666 Jabalpur +91-7004386936 W: www.toprankers.com
46 The CLAT
ThePost
CLAT• September
Post • July 2021

Legal Current Affairs


1. A suit was filed by a Bengali for creamy layer, held Khan, after he received an advocate-general/
actor in the Delhi High while quashing a Haryana externment order passed articleshow/86354481.cms
Court, pleading to remove government notification by the Maharashtra police. )
the objectionable videos of August 17, 2016, that According to the SC, the
that were uploaded on the the economic criterion drastic action of externment
online platforms, without cannot be the sole basis for should only be taken 5. The Jammu and Kashmir
her consent and infringes deciding the creamy layer in exceptional cases, to government has decided to
from among backward maintain law and order in implement the Forest Rights
classes for the purpose a locality and/or prevent a Act (FRA), 2006, which will
of excluding it from the breach of public tranquility elevate the socio-economic
purview of providing and peace. status of a sizable section
reservation. The court held of the 14-lakh-strong
(Source: https://www.
that the given notifications population of tribals and
thehindu.com/news/
which specified the criteria nomadic communities. FRA
cities/Delhi/externment-
her privacy. Justice Asha for exclusion of creamy enacted in 2006 recognises
orders-must-be-used-only-
Menon recognised that the layer within the backward the rights of forest-dwelling
in-exceptional-cases-sc/
right to privacy includes classes are in violation of the tribal communities and
article36188413.ece)
the right to be forgotten and Indira Sawhney I case.
the right to be left alone.
(Source: https://
The court held that the
indianexpress.com/article/ 4. The Advocate General of
plaintiff is entitled to be left
india/sc-quashes-2016- Punjab Atul Nanda resigned
alone and forgotten and is
h a r y a n a - n o t i f i c at i o n - from the post, a day after
entitled to protection from
economic-criterion- Punjab’s CM Amarinder
invasion of her privacy, that
cannot-be-sole-basis- Singh resigned, highlighting
is exhibited against her will.
to-decide-creamy- that his appointment was
(Source: https:// layer-7469188/) co-terminus with the chief
timesofindia.indiatimes. minister’s appointment.
com/city/delhi/right- Further, Senior Advocate
to-privacy-includes- 3. A division bench of Supreme D.S. Patwalia has been
other traditional forest
r i g ht - to - b e - forgotten - Court consisting of Justices appointed as the new
dwellers to forest resources
s ays-del hi-hig h-cour t/ Indira Banerjee and V.
on which these communities
articleshow/85674714.cms) Ramasubramanian, while
were dependent for a
variety of needs, including
livelihood, habitation
2.
A division bench of
and other sociocultural
Supreme Court consisting
needs. It recognizes and
of Justices L Nageswara
vest the forest rights and
Rao and Aniruddha
occupation in Forest land in
Bose while hearing a Advocate General of
Forest Dwelling Scheduled
petition challenging two the state. Patwalia has
Tribes (FDST) and Other
been practised on the
Traditional Forest Dwellers
Constitutional side and has
(OTFD) who have been
also dealt with service and
residing in such forests for
criminal matters.
generations.
setting aside an externment
(Source: https://livelaw.
order against a journalist, (Source: https://www.
in/news-updates/senior-
the court ruled that a thehindu.com/news/
a d v o c ate - d s - p at w a l i a -
person’s fundamental right national/other-states/jk-to-
appointed-new-advocate-
to reside and to move about implement-forest-rights-
general-punjab-182284
notifications issued by the freely anywhere in India act-2006/article36446427.
Haryana government sub- cannot be refused on “flimsy https://timesofindia. ece)
classifying backward classes grounds”. The judgment indiatimes.com/city/
solely on economic basis came after an appeal was chandigarh/nanda-
while fixing the criteria filed by Journalist Rahmat resigns-as-punjab- 6. A division bench of Supreme
Bhopal +91-7676564400 Indore +91-9589613810, +91-731-4987379 Prayagraj +91-8114000926, +91-8114000927 Kanpur +91-8576066660, +91-8707605589 E: support@toprankers.com
Gurugram +91-8448444207 Delhi +91-9810610466 Ranchi +91-9334969993 Lucknow +91-6390576666 Jabalpur +91-7004386936 W: www.toprankers.com
47 The CLAT Post • September 2021
Court consisting of Justices against the orders of Undergraduate Medical Pradesh State Legal Services
Ajay Rastogi and Abhay Punjab and Haryana HC, Degree Courses Bill, 2021”, Authority. Over 94297
S. Oka observed that the upholding the appointment which seeks to ‘dispense’ Cases have been settled
sentence awarded in a non- of Dinkar Gupta as the DGP with the requirement for mutually involving an
compoundable criminal of Punjab. The petitions had candidates to qualify in amount of 3,59,44,56,371
matter cannot be reduced been filed by IPS officers the National Eligibility- in the 2nd National Lok
solely on the basis of the fact who were not considered cum-Entrance Test Adalat 2021. This Lok
that there is a compromise for appointment as the top (NEET) for admission to Adalat was organized across
between the victim and the police officer of the state of undergraduate medical the State including Principal
Punjab. courses in the State. The bill Seat Jabalpur & Benches at
was introduced by CM M.K. Gwalior & Indore of the
(Source: https://www.
Madhya Pradesh High
l i vel aw. i n / top - stor i e s /
Court and all district,
supreme-court-dinkar-
family, labour and other
gupta-punjab-dgp-181694)
courts and tribunals across
the state. Further, Justice
Prakash Shrivastava is the
8. Senior Advocate Gopal
Executive Chairman of the
Mukherjee is appointed as
MPSLSA- Jabalpur.
the Advocate General of
the state of West Bengal. (Source: https://www.
The appointment came livelaw.in/news-updates/
accused. The court held that
after senior advocate madhya-pradesh-state-
in considered view, such a Stalin, who contended that
Kishore Datta tendered his legal-services-authority-
brutality cannot be ignored the admission to medical
resignation as the Advocate organises-2nd-national-
which is not against the courses was traceable to
General citing ‘personal lok-adalat-181492)
individual but the crime is entry 25 of List III, Schedule
reasons’. The appointment
against the society which VII of the Constitution, and
took place under the Article
has to be dealt with sternly. the State was “competent to
11. The Central Government
regulate”.
(Source: https://www. has appointed G.S. Pannu
l i vel aw. i n / top - stor i e s / (Source: https://www. as Officiating President of
supreme-court-non- livelaw.in/news-updates/ the Income Tax Appellate
comp oundable- no-neet-for-govt-medical- Tribunal (ITAT), till the
compromise-not-solitary- seats-admissions-tamil- appointment of regular
basis-182050 nadu-passes-law-181516 ) President or further orders.
G.S. Pannu was serving as
Case: Bhagwan Narayan
Gaikwad vs State Of
10. The Madhya Pradesh Legal
Maharashtra)
Service Authority organised
165(1) of the Constitution 2nd National Lok Adalat
by the Governor of the state of 2021. It was inaugurated
7. A Constitutional bench of
Jagdeep Dhankar. Further, by Justice Mohammad
Supreme Court consisting
Datta is the fourth advocate
of Justices LN Rao, Sanjiv
general of the state to
Khanna and BR Gavai,
resign since 2011 when the
reserved judgment on batch
Trinamool Congress first
came to power. Vice-President of Income
Tax Appellate Tribunal,
(Source: https://www.
New Delhi.
livelaw.in/news-updates/
calcutta-high-court- (Source: https://www.
advocate-general-gopal- livelaw.in/news-updates/
mukerjee-181584) itat-president-vice-
president-gs-pannu-
centre-regular-
of special leave petitions 9. The legislative assembly president-181048)
filed by IPS Officers to Tamil Nadu passed a
Mohammad Mustafa and bill named “The Tamil
Rafiq, the Chief Justice &
Siddarth Chattopadhyay Nadu Admission to
Patron-in-Chief, Madhya
Bhopal +91-7676564400 Indore +91-9589613810, +91-731-4987379 Prayagraj +91-8114000926, +91-8114000927 Kanpur +91-8576066660, +91-8707605589 E: support@toprankers.com
Gurugram +91-8448444207 Delhi +91-9810610466 Ranchi +91-9334969993 Lucknow +91-6390576666 Jabalpur +91-7004386936 W: www.toprankers.com
48 The CLAT Post • September 2021

ARTICLES YOU CANNOT AFFORD TO MISS


PEACE MISSION-2021: INDIA TAKES PART IN
SCO JOINT MILITARY EXERCISE IN RUSSIA
India took part in the sixth edition of military exercise organised by the Shanghai Cooperation
Organisation (SCO) in Russia, the Army said in a statement.
India participated in the The sixth edition of the between the Armed Forces of landmark event in military
sixth edition of multinational military exercise ‘Peaceful SCO nations. The exercise will interactions and global
military exercise organised Mission-2021’ is being hosted also provide an opportunity cooperation to counter-
by the Shanghai Cooperation by Russia in the Orenburg to the Armed Forces of SCO terrorism,” the Army said in
Organisation (SCO) in Russia’s region. The exercise began on Nations to train in Counter- the statement.
Oregon, the Army said in a September 13 and will end on Terrorism Operations in
statement on Wednesday. September 25. an urban scenario in a In the past, Pakistan, China
multinational and joint and India have taken part
“The Indian military “The joint counter-terrorism environment,” the Army said. in multinational military
contingent comprising an exercise ‘Peaceful Mission’ is a exercises. However, last year,
all arms combined force of India did not send troops for
200 personnel including 38 the SCO military exercise
personnel from the Indian where both China and
Air Force is participating in Pakistan were present.
the Exercise Peaceful mission
-2021. The Indian contingent The SCO is an economic and
was inducted to the exercise security bloc in which India
area by two IL-76 aircrafts,” the and Pakistan were admitted
Army said in a press statement. as full members in 2017. Its
founding members included
“Prior to their departure, China, Russia, Kazakhstan,
the contingent underwent Kyrgyzstan, Tajikistan and
training and preparation under Uzbekistan.
the aegis of South Western multilateral exercise, which is “The scope of the exercise
Command,” the Army said. (Source: https://www.
conducted biennially as part includes professional indiatoday.in/india/
of military diplomacy between interaction, mutual story/peace-mission-
The aim of the exercise is to
Shanghai Cooperation understanding of drills and
2021-indian-army-sco-
foster close relations between
Organisation (SCO) member procedures, establishment
joint-military-exercise-
SCO member states and
states,” the Army said. of joint command, control
russia-1853099-2021-09-15)
to enhance abilities of the
structures and elimination
military leaders to command “The exercise will enable of terrorist threats. Exercise
multi-national military sharing of best practices Peaceful Mission 2021 is a
contingents, the Army said.

QUOTA FOR SCS, STS IN JOB PROMOTION


| DON’T WANT TO REOPEN ORDER, SAYS
SUPREME COURT
Bench says its up to State to reservation in promotion in to find ways and means of judgment that required the
find ways to implement the government jobs to Scheduled implementing the court’s June State to show quantifiable data
rule. Castes and Scheduled Tribes. 2018 judgment on the issue. to prove the “backwardness” of
a Scheduled Caste/Scheduled
The Supreme Court on A Bench led by Justice L. In a 58-page judgment in 2018, Tribe community in order to
Tuesday said it was not Nageswara Rao reiterated a Constitution Bench, led by provide quota in promotion in
ready to reopen a debate the court’s stand that it was then Chief Justice of India public employment.
on its judgments granting up to the State governments Dipak Misra, modified a 2006
Bhopal +91-7676564400 Indore +91-9589613810, +91-731-4987379 Prayagraj +91-8114000926, +91-8114000927 Kanpur +91-8576066660, +91-8707605589 E: support@toprankers.com
Gurugram +91-8448444207 Delhi +91-9810610466 Ranchi +91-9334969993 Lucknow +91-6390576666 Jabalpur +91-7004386936 W: www.toprankers.com
49 The CLAT Post • September 2021
The five-judge Bench judgment expression ‘backward class of determine. Senior advocate Indira
in the Jarnail Singh case in citizens’”. Jaising appearing for one
2018 had given a huge fillip On Tuesday, Justice Rao said of the petitioners, said the
to the government’s efforts The Constitution Bench had the court had no intention question was how the States
to provide “accelerated refused to refer the Nagaraj to reopen either Nagaraj were supposed to identify
promotion with consequential judgment to a seven-judge or Jarnail Singh. The court “backwardness”. “In some
seniority” for Scheduled Bench. It had also dismissed expects the States to comply cases, High Courts have struck
Castes/ Scheduled Tribes (SC/ the government’s view that with the judgments and not down on the ground that
ST) members in government comparing the strength of try expand or deflect the backwardness has not been
services. an SC/ST community to the scope of the case, the Bench, shown,” she submitted.

Indra Sawhney case Justice Rao shot back, “We


are not here to advise the
Writing the verdict for the government what they should
Constitution Bench three do. It is not for us to tell the
years ago, Justice Rohinton government how to implement
Nariman (now retired) had policy. It has been specifically
held that the requirement to held as to how the States
have quantifiable data before have to implement it and
granting reservation to SCs consider backwardness and
and STs was contrary to an representation. The States have
earlier nine-judge Bench to decide what to do subject to
verdict in the Indra Sawhney population of India cannot comprising Justices Sanjiv judicial review”.
case. be the test to determine Khanna and B.R. Gavai,
whether they are adequately clarified. (Source: https://www.
In the Indira Sawhney case, thehindu.com/news/national/
represented in government
the court had held that the It made these remarks while quota-for-scs-sts-in-job-
services. It had held that
“test or requirement of social hearing over 130 petitions promotion-dont-want-to-
the question of adequate
and educational backwardness variously seeking the reopen-order-says-supreme-
representation of an SC/ST
cannot be applied to Scheduled implementation of judgments court/article36458962.ece)
community ought to be left
Castes and Scheduled Tribes, for reservation in promotion.
to the respective States to
who indubitably fall within the

EXPLAINED: HOW WAS THE QUAD FORMED,


AND WHAT ARE ITS OBJECTIVES
Following the Indian Ocean tsunami, India, Japan, Australia, and the US created an informal
alliance to collaborate on disaster relief efforts.
On Monday night, the White Formation of the Quad The early iteration of the Quad, Union or United Nations,
House announced that US largely based around maritime the Quad has focused on
President Joe Biden will be Following the Indian Ocean security, eventually dissipated. expanding existing agreements
hosting the first in-person tsunami, India, Japan, between member countries
meeting of the Quad countries Australia, and the US In 2017, faced again with the and highlighting their shared
on September 24. Prime created an informal alliance rising threat of China, the four values. Additionally, unlike
Minister Narendra Modi, to collaborate on disaster countries revived the Quad, NATO, the Quad does
Australian PM Scott Morrison relief efforts. In 2007, then broadening its objectives and not include provisions for
and Japanese PM Yoshihide PM of Japan, Shinzo Abe, creating a mechanism that collective defence, instead
Suga will be present at the formalised the alliance, as aimed to slowly establish a choosing to conduct joint
meeting. the Quadrilateral Security rules-based international military exercises as a show of
Dialogue or the Quad. The order. However, despite its unity and diplomatic cohesion.
According to the White House Quad was supposed to lofty ambitions, the Quad is
statement, at the meeting, the establish an Asian Arc of not structured like a typical In 2020, the trilateral India-
Quad leaders will focus on Democracy but was hampered multilateral organisation and US-Japan Malabar naval
issues related to the Covid-19 by a lack of cohesion amongst lacks a secretariat and any exercises expanded to include
crisis, climate change, its members and accusations permanent decision-making Australia, marking the first
cyberspace and security in the that the group was nothing body. Instead of creating policy official grouping of the Quad
Indo-Pacific. more than an anti-China bloc. along the lines of the European since its resurgence in 2017
Bhopal +91-7676564400 Indore +91-9589613810, +91-731-4987379 Prayagraj +91-8114000926, +91-8114000927 Kanpur +91-8576066660, +91-8707605589 E: support@toprankers.com
Gurugram +91-8448444207 Delhi +91-9810610466 Ranchi +91-9334969993 Lucknow +91-6390576666 Jabalpur +91-7004386936 W: www.toprankers.com
50 The CLAT Post • September 2021
and the first joint military we are recommitting to a China initially opposed the the international rules-based
exercises among the four shared vision for an Indo- formation of the Quad and order. Japan and Australia
countries in over a decade. In Pacific region that is free, in the 13 years since, Beijing’s are likewise both concerned
March 2021, the Quad leaders open, resilient and inclusive. position has not changed. In about China’s expanding
met virtually and later released We are striving to ensure that 2018, the Chinese Foreign presence in the South and
a joint statement titled ‘The the Indo-Pacific is accessible Minister referred to the Quad East China Seas. For Australia
Spirit of the Quad,’ which and dynamic, governed by as a “headline-grabbing idea” in particular, relations with
outlined the group’s approach international law and bedrock and after the joint statement Beijing are at a considerable
and objectives. principles such as freedom was issued earlier this year, low after Australia passed
of navigation and peaceful the Chinese foreign ministry foreign interference laws in
Objectives of the Quad resolution of disputes, and that accused the group of openly 2018 which China responded
all countries are able to make inciting discord among to by restricting trade to
According to the Spirit of the
their own political choices, free regional powers in Asia. Canberra. As the only Quad
Quad, the group’s primary
from coercion. In recent years, Beijing sees the existence of country to share a land border
objectives include maritime
that vision has increasingly the Quad as part of a larger with China, India is also
security, combating the
been tested. Those trials have strategy to encircle China suitably wary of Beijing but
Covid-19 crisis, especially
only strengthened our resolve and has pressured countries also reluctant to allow tensions
vis-à-vis vaccine diplomacy,
to spill over.
addressing the risks of climate
change, creating an ecosystem However, although the Quad
for investment in the region is perceived to be anti-China,
and boosting technological there is no direct reference to
innovation. Quad members China or military security in
have also indicated a either the joint statement or
willingness to expand the the Washington Post op-ed.
partnership through a so- This in turn has led experts
called Quad Plus that would to speculate that the Quad
include South Korea, New will refrain from addressing
Zealand, and Vietnam amongst the military threat posed by
others. China and instead focus on its
to reckon with the most urgent like Bangladesh to avoid economic and technological
In a March 2021 opinion piece
of global challenges together.” cooperating with the group. influence. The Quad’s decision
in the Washington Post, the
to establish working groups
leaders of all four member
However, despite the Quad’s Each of the Quad members on vaccine development and
nations described the need for
seeming commitment to a are threatened by China’s critical technologies can then
the alliance and its intentions
broad range of issues, its raison actions in the South China Sea be viewed as an attempt to
for the future. They wrote:
d’etre is still considered to and its attempts to extend its constrain China but more
“Since the tsunami, climate be the threat of China. Each sphere of influence through importantly, to create a
change has grown more of the Quad’s member states initiatives such as the One democratic, inclusive blueprint
perilous, new technologies have their own reasons to fear Belt One Road Project. The that will encourage other states
have revolutionized our daily the rise of China and curbing US has long been concerned to work with the Quad.
lives, geopolitics have become Beijing’s regional advances is about the global competition
in all of their national interests. with China and successive US (Source: https://indianexpress.
ever more complex, and a
presidents have maintained com/article/explained/
pandemic has devastated the
China that China aims to subvert quad-nations-meeting-us-
world. Against this backdrop,
china-7508605/)

A TIME TO INTROSPECT: ON CENTRE-STATE


DIALOGUE AND GST
More Centre-State deliberations needed with GST regime at a critical turning point
At its first physical meeting such as oxygen concentrators of vaccination, there are no view on pandemic essentials.
during the pandemic, the GST will lapse on September 30, signs the virus and its variants Tax rate tweaks were okayed
Council approved a flurry of while the lower rates on would be extinct on New Year’s for an eclectic range of sectors
changes. Concessional tax rates medicines were extended till Day, so the Council could with long-pending course
on vital COVID-19 equipment December. Whatever the pace have taken a more considerate correction on inverted duty
Bhopal +91-7676564400 Indore +91-9589613810, +91-731-4987379 Prayagraj +91-8114000926, +91-8114000927 Kanpur +91-8576066660, +91-8707605589 E: support@toprankers.com
Gurugram +91-8448444207 Delhi +91-9810610466 Ranchi +91-9334969993 Lucknow +91-6390576666 Jabalpur +91-7004386936 W: www.toprankers.com
51 The CLAT Post • September 2021
structures plaguing several a personal care item at 18% for discussed petro products only up a little revenue to spur
items, including footwear and pack sizes below one litre and briefly to comply with a Kerala spending. A similar dialogue
textiles. The semblance of retain the 5% rate on edible High Court order, consumers is needed for an honest
clarity brought in on a much- oils for larger packs, has been who need some relief on review of the GST regime’s
disputed issue — the definition held back for study, and will fuel prices — irrespective of progress and the way ahead.
of an intermediary — is hopefully be shelved for good. who cuts taxes — may have With just nine more months
welcome, for it was hurting held misplaced hopes. If the of assured compensation
several sectors, including These pluses and minuses Government really wants for States, they are worried
IT services exports. Double aside, two things stand out a consumption rebound about revenue streams falling
taxation on the import of for Indian consumers — the that may reignite private off the cliff thereafter. Their
pleas for an extension in the
compensation period have met
with stern diffidence and the
argument that GST revenues
are below expectations. Two
ministerial groups have been
tasked to augment revenues
using technology and rate
rationalisations. The Centre
need not wait for their reports
to hold a special Council
meeting to discuss States’
compensation concerns, as
had been promised. At this
juncture, the Council should
be a forum for empathetic
contemplation, not fractious
leased aircraft goes. Food Council’s firm dismissal of any investments, the Centre and friction.
delivery services players shall shift of petroleum products to States must begin talks on
be made liable to collect and GST to lower the tax burden rationalising fuel taxes. The (Source: https://www.
remit taxes instead of the and the fact that GST cess Finance Minister has often thehindu.com/opinion/
restaurants. One awaits the fine on automobiles, tobacco and expressed the worry: ‘What editorial/a-time-to-introspect-
print to assess the impact on aerated drinks will now be if we cut taxes and States do the-hindu-editorial-on-
consumers and smaller outlets. levied till April 2026, not June not’. Perhaps, a compact could centre-state-dialogue-and-gst/
The plan to tax coconut oil as 2022 as originally envisaged. be arrived at, so both give article36619857.ece)
While the Council may have

JUVENILE JUSTICE LAWS NEED TO UPHOLD


THE TWIN OBJECTIVES OF JUSTICE AND
DETERRENCE
The Juvenile Justice Amendment Act, 2021 was one of the crucial legislations that were bull-
dozed through in the monsoon session.
Among the several other serious deliberation. Although progress, it was not possible juvenile prove to be dangerous
important legislations that the Bill, which relates to justice for me to speak, considering towards the normal course
were bulldozed during the and protection to (or from) the situation. Hence, I would of the functioning of society,
monsoon session 2021, the juveniles, needed elaborate like to deliberate on the issue that person may be referred
Juvenile Justice Amendment discussion, it became the law of juvenile delinquency in to as a juvenile delinquent.
Act, 2021 was one. While of the land without that. India to exercise my most Factors like personality,
the intervention did make fundamental right here. family, substance abuse, peer
some bold improvements Although I did possess an relationships etc. can induce
with respect to assigning opinion regarding the subject As the term suggests, it delinquency in any minor,
responsibilities, it cannot and wanted to speak on it refers to the involvement of given his/her vulnerable
be assumed that it does not in parliament, when the minors in illegal activities and nature.
demand due diligence and discussion on the Bill was in crimes. When the actions of a
Bhopal +91-7676564400 Indore +91-9589613810, +91-731-4987379 Prayagraj +91-8114000926, +91-8114000927 Kanpur +91-8576066660, +91-8707605589 E: support@toprankers.com
Gurugram +91-8448444207 Delhi +91-9810610466 Ranchi +91-9334969993 Lucknow +91-6390576666 Jabalpur +91-7004386936 W: www.toprankers.com
52 The CLAT Post • September 2021
The most intriguing aspect of that 91% of these juveniles entertainment in transforming also be another contribution
this phenomenon is that it has had received at least primary children into adults. And we towards evidence-based policy
been regarded as a problem level education. This indicator often run on the assumption intervention. Parents, teachers,
integral to contemporary is clearly reflective of the that just because the child has society and media, all have an
society, while the reality systematic failure of the state a good family background, important role in moulding
is that just like any other to fulfil the objective of quality we can be assured of the children into adults and it is
criminal behaviour, it has been education. These statistics base values given to him. necessary that proper care and
prevalent in some form or surely reflect the need for To me, it feels like the state attention are given to children
other in every phase of human strong preventive measures in shrugging off from its most while they grow.
history. order to reduce the nature of basic responsibility of ensuring
these crimes. fundamental values of welfare. It has been observed by many
As per the 2018 ‘Crime in schools, during the course of
India’ report, about 85% of Though, I strongly appreciate Our education policy, which online classes, that very young
the apprehended juveniles the intent and provisions of is currently on the stove of children are sharing extremely
lived with their parents, which this Act, which of course pave debate these days, needs to objectionable material online.
clearly reflects an inherent way for a better ecosystem also discuss this aspect of child Many of their chats show
failure of the system in terms of the child protection welfare. Being mindful of the dangerous levels of aggression
of nurturing the future system in India. But what’s dangerous trends of increasing and expression of sexual

generation. Moreover, 92% important here is to study the aggression levels among violence against the opposite
of juvenile cases were IPC- nuances behind these data children, our curriculum needs gender. I strongly feel that the
related crimes. Almost 40% points closely to formulate to heavily emphasise on the current nature of education in
of the offences were related legislative interventions to values of kindness, respect India focuses on academics in
to affecting the human body, solve the policy problem. As and empathy. Moreover, isolation.
which included hurt and it is and it always has been, community involvement needs
grievous hurt, rape and assault implementation has been a to become a crucial facet of Very few schools essentially
on women to outrage their fundamental issue. No matter our curriculum. target on the holistic
modesty among many others. how procedural it looks on development of a child.
Apart from this, property- paper, the reality deviates The ministry of education A nuanced study must be
related offences amounted far away from the optimal must give directives to urgently conducted to dig
to about 38% of all crimes solution. all schools to prepare a deeper onto the functioning of
involving juveniles. Theft comprehensive report of schools. As we are preparing
alone constituted 70% of these As a strong proponent problems related to the ourselves with opening up of
offences. of quality education, I mindset and attitude prevalent schools, we must also keep in
bolster the role played by among students. Studying mind the big lag in physical
What’s interesting to note is parents, teachers, books and the nature of conflicts could engagement among students.
Bhopal +91-7676564400 Indore +91-9589613810, +91-731-4987379 Prayagraj +91-8114000926, +91-8114000927 Kanpur +91-8576066660, +91-8707605589 E: support@toprankers.com
Gurugram +91-8448444207 Delhi +91-9810610466 Ranchi +91-9334969993 Lucknow +91-6390576666 Jabalpur +91-7004386936 W: www.toprankers.com
53 The CLAT Post • September 2021
Their patience levels have and diet. the gruesomeness of the crime, is 16 years and above and is
been adversely affected and if the criminal was below 18 involved in a heinous crime,
excessive use of the internet, Not even a single CCI in years of age. The entire nation then on the assessment report
and probably inadvertent the country was found to be waited and watched, as the of the Juvenile Board, the case
exposure to forbidden content, 100% compliant to the Act’s family of the girl broke down, can be trialed in the children’s
have contributed to increasing provisions. As per the latest time and again on television court and the minors would
aggression levels. social audits done by the channels, in disappointment be treated as adults and would
National Commission for every time the journey towards be subjected to criminal
By studying the history of Protection of Child Rights justice took them only to the procedure.
the juvenile justice system in (NCPCR) of CCIs, significant beginning of another road
India, our focus has always variations exist between states towards it. Our judicial system The potential solutions
been on strengthening it by in terms of complying with needs to overhaul itself from
facilitating rehabilitation and the Act’s provisions. Hence, it I believe that law must serve
this pathetic image. And our its purpose. It must act as
better treatment of young is essential to strengthen the laws need to be framed, not in
people in conflict with law compliance and regulatory deterrent and provide justice
unnecessary haste, but after to the victim, as well as prevent
and by improving protection framework of even the existing minute thinking and deserving
of child victims. As put Act. further crime. This ought to
deliberation. get the primary focus of the
forth by the Supreme Court,
“Regrettably our juvenile There have been multiple The statistics law. The Act must protect the
justice system still thinks in instances where the Supreme right to life of the victim. And
terms of terror, not cure, of Court has recommended Here, I wish to put forth some the right to life also implies the
wounding, not healing, and a improvement in the data of the year 2012 National right to justice!
sort of blind man’s bluff is the functioning of CCIs. A good Bureau of Crime Records,
start could be to introduce just as an example. In the year Law should be more inclined
result. This negative approach towards the victims because
converts even the culture of legal interventions setting 2012, 27,936 juveniles were
minimum standards for allegedly involved in serious victims are also juveniles and
juvenile homes into juvenile sometimes even two-year-olds,
jails. From the reformatory infrastructure of the CCIs crimes including banditry,
with their regular review to murder, rape and rioting. three-year-olds. I would have
angle, the detainees are liked to suggest an amendment
left to drift, there were no ensure the adherence. Our From among those who faced
judicial system requires laws Juvenile Justice Boards in 2012, that another category be made
constructive programme for for juveniles along with petty
the detainees nor correctional that are free from ambiguity 66.6% were between 16 and 18
and loopholes, for enabling the years, 30.9% were between 12 offences, serious offences and
orientation and training for heinous offences, it should be
the institutional staff.. juvenile meting out of speedy justice. and 16 years and 2.5 % were
even those from seven to 12 categorised as gruesome sexual
detainees need a new focus The “Nirbhaya gang-rape” offences, where irrespective
and a new rationale.” years. There was an increase
case is an example in itself. In by 143% in rapes by juveniles, of the legal age, the biological
This integral remark never spite of the public fury and 87% in murders, 500% in age of the accused may be
actually got reflected in the the unending candle marches kidnappings of women and considered, and any juvenile,
recent amendment. and the establishment of girls by minors. These statistics who attains puberty and is
committees and commissions, can certainly not be ignored. capable of performing and
Compliance and regulatory the road to justice was commits a “gruesome” sexual
framework of the existing Act exhaustingly long and Moreover, the statistical data crime, that destroys the life of
rough. The juvenile, who of the NCRB reveals and is of a minor, and that of a family,
Privatisation, a tenet of was a partner in the crime, graver concern that there has must be tried as adult.
the neoliberal era, has also had allegedly been the most been a steady increase in the
impacted the juvenile justice heinous and gruesome of all gravity of heinousness and The punishment for a
system, which has also resulted others in the performance of gruesomeness of the crimes “gruesome” sexual crime
in a shift in the accountability the disgusting act of the rape committed by juveniles, of late, should be focused on as a
of the functionaries of and murder. and almost 30% of them are preventive measure for further
privately-run child care from the age group of 12 to 16 crimes against potential
institutions (CCIs). As of ABVP members protest against years. victims. Punishment for one
March 2020, there are around the release of the juvenile can save a thousand! So in our
2,000 CCIs across India. The convict in the Nirbhaya case, I believe that law should go by efforts to bring about measures
committee on review exercise at Jantar Mantar, in New Delhi the gravity of these heinous for strengthening the child
of CCIs in 2018 noted that on Monday. Credit: PTI Photo crimes. “Punishment should protection ecosystem, we are
many CCIs fail to provide by Kamal Kishore befit the gravity of the crime.” protecting the child!
even the basic services to the
children including individual Our laws, however, did not The Juvenile Justice Act, 2015 If international customary law
bedding, and proper nutrition provide for penal action for lays down that if the minor is taken into consideration,
Bhopal +91-7676564400 Indore +91-9589613810, +91-731-4987379 Prayagraj +91-8114000926, +91-8114000927 Kanpur +91-8576066660, +91-8707605589 E: support@toprankers.com
Gurugram +91-8448444207 Delhi +91-9810610466 Ranchi +91-9334969993 Lucknow +91-6390576666 Jabalpur +91-7004386936 W: www.toprankers.com
54 The CLAT Post • September 2021
then the position in the US, by them. I feel that the society’s soul than the way it (Source: https://thewire.in/
the UK and Canada concludes entire Act must be revisited. treats its children.” And with law/juvenile-justice-laws-
that minors can be prosecuted The Noble Laureate Nelson expression of a firm intent by need-to-uphold-the-twin-
under criminal law for grave Mandela stated: “There can this government, it’s crucial objectives-of-justice-and-
and heinous crimes committed be no keener revelation of a that we do it right! deterrence)

‘WOMEN OF THE WORLD, UNITE’: CJI


SUPPORTS 50% RESERVATION FOR WOMEN
IN JUDICIARY
In a speech, the CJI detailed the extent of the gender imbalance in the judiciary, the various re-
al-world problems women lawyers face and the need for inclusive, infrastructural change.
New Delhi: Chief Justice of women in the judiciary face. of the 6,000 trial courts in the The CJI also expressed support
India N.V. Ramana on Sunday, The gender roles which are country, only 22% of them for the reservation of seats for
September 26, called for the enforced on women when they have washrooms for women. women in law schools across
“urgent correction” of the choose to become lawyers, “We need to create a more the country, saying, “We hope
gender imbalance in the Indian the dual-responsibility they welcoming environment,” more women will join the
judiciary, stating that women have to contend with as family he noted, pointing to the profession and achieve the goal
are entitled to 50% reservation responsibilities inevitably proposed National Judicial of 50% shortly.”
in the judiciary and should fall on them and a general Infrastructure Corporation
demand it, the Indian Express preference of clients for male (NJIC) project which the CJI Justice B.V. Nagarathna, who
reported. advocates all pose significant himself has envisaged. was appointed to the Supreme
challenges to women in the Court in August and will serve
The CJI’s remarks came at a as India’s first woman Chief
felicitation ceremony which Justice in 2027, also spoke at
was organised by the Lady the event. She noted that the
Advocates of the Supreme visibility of women as judicial
Court of India, where he officers can “pave the way
remarked that, after thousands for greater representation of
of years of suppression, equal women in other decision-
representation is a right of making positions such as
women. “It is not a matter of legislative and executive
charity,” he said. branches of the government,”
the Hindustan Times reported.
During his speech, CJI
Ramana also detailed the Justice Nagarathna will only
measly representation of enjoy a term of one month as
women in the judiciary in the CJI.
numbers, stating that women
make up only 30% of the lower The CJI also expressed hope
judiciary, 11.5% of the high for the physical resumption
courts and 12% of the Supreme of courts after the Dussehra
Court. Moreover, only 15% vacation, which is scheduled
of the country’s 1.7 million from October 11-16.
advocates are women. The Supreme Court has
judicial system. CJI Ramana, while speaking been hearing cases virtually
The picture is worse when it
about the NJIC on September ever since the coronavirus
comes to bar councils: only
The environment in the 11, said that the project will lockdown was imposed last
2% of women are elected
courtrooms are also not follow the design principles year, only recently moving to a
representatives in state bar
favourable for women, as the of “socially responsible and “hybrid” mode of functioning.
councils and the Bar Council
CJI pointed out. Crowded inclusive architecture” and
of India has no female (Source: https://thewire.in/
courtrooms and a lack of that it would work towards
members whatsoever. law/cji-calls-for-reservation-
infrastructure affect women creating “national assets” in the
The CJI went on to detail disproportionately. CJI country. for-women-in-judiciary)
the real-world problems Ramana pointed out that, out
Bhopal +91-7676564400 Indore +91-9589613810, +91-731-4987379 Prayagraj +91-8114000926, +91-8114000927 Kanpur +91-8576066660, +91-8707605589 E: support@toprankers.com
Gurugram +91-8448444207 Delhi +91-9810610466 Ranchi +91-9334969993 Lucknow +91-6390576666 Jabalpur +91-7004386936 W: www.toprankers.com
55 The CLAT Post • September 2021

‘HINDI CANNOT BE USED FOR OFFICIAL


CORRESPONDENCE WITH TAMIL NADU’:
MADRAS HC
The bench was hearing a PIL filed by Lok Sabha MP from Madurai, Su Venkatesan, who ap-
proached the HC after he received a reply from the Centre in Hindi to a representation made by
him in English.
New Delhi: The Madras high give an impression that one Union Government to give a filed by Lok Sabha MP from
court recently told the Union language alone is superior reply in English only which Madurai, Su Venkatesan, who
government that it is duty- and being imposed upon the will also be in consonance with approached the court after
bound to communicate in people speaking different the statute, viz., the Official he received a reply from the
English with those states that languages,” the bench of Languages Act.” Union government in Hindi to
have not adopted Hindi as Justices N. Kirubakaran (now a representation made by him
their official language, news retired) and M Duraiswamy “To put it in other words, in English.
reports said. observed. the official language of India
Venkatesan told the court that
he had written back to the
Union government to send the
reply in English. However, after
not receiving any response on
the matter, he moved Madras
high court.

Assistant Solicitor General L.


Victoria Gowri, representing
the government, submitted
that the reply to Venkatesan
had been inadvertently written
in Hindi, the report said.

“There was no intention on the


part of the Central government
to violate any provision of the
Official Languages Act, 1963 or
“Any kind of fanaticism is Referring to the Official (Hindi) cannot be used for Rules,” the court was told.
not good for any society. Languages Act 1963 and official correspondence with
Fanaticism, in any form, the Official Language Rules the State of Tamil Nadu,” the (Source: https://thewire.in/
is to be condemned, if it is 1976, the court noted, “Once court said. law/hindi-cannot-be-used-
exhibited. Linguistic fanaticism a representation is given in for-official-correspondence-
is more dangerous as it would English, it is the duty of the The bench was hearing a with-tamil-nadu-madras-hc)
Public Interest Litigation (PIL)

HABEAS CORPUS CANNOT LIE AGAINST


MOTHER WHO HAS CUSTODY OF CHILD:
SUPREME COURT REJECTS FATHER’S PLEA
The Supreme Court on with the mother (Amit detainer?” Singapore back,” asked Justice
Monday pulled up a Gulrajani v. State of Haryana). Surya Kant.
child’s father involved in a “How can you file a Habeas
transnational custody battle A Bench of Chief Justice Corpus petition? She is with The Court proceeded to hold
with the mother, questioning of India NV Ramana and her mother. How can the that the family court is the
the rationale behind filing a Justices Surya Kant and Hima mother become an illegal appropriate authority to decide
Habeas Corpus to get custody Kohli questioned as to how detainer? Is the girl a table or the issue of custody, even in
of the child currently staying a “mother can be an illegal chair that she can be moved to cross-border custody disputes.
Bhopal +91-7676564400 Indore +91-9589613810, +91-731-4987379 Prayagraj +91-8114000926, +91-8114000927 Kanpur +91-8576066660, +91-8707605589 E: support@toprankers.com
Gurugram +91-8448444207 Delhi +91-9810610466 Ranchi +91-9334969993 Lucknow +91-6390576666 Jabalpur +91-7004386936 W: www.toprankers.com
56 The CLAT Post • September 2021
The parties to the case had The husband then moved the she was threatened by her ego you want to kill the child,”
gotten married in Gurugram, Supreme Court by way of a husband and that proceedings said CJI.
India in 2010 and soon Special Leave Petition assailing for guardianship are already
after the marriage, shifted the order of the High Court. pending. It is only the family “This is most unfortunate,”
to Singapore. In July 2011, The chief question before court which can adjudicate added Justice Kohli.
the couple was blessed with the Supreme Court were on custody after an elaborate The top court, therefore,
a daughter. However, the whether there was any illegal enquiry in the facts and rejected the plea but directed
relationship between them detention or not and whether a circumstances of the case, it that the proceedings in
soured and the wife left Habeas Corpus plea would be was contended. the subordinate court be
Singapore for India with the maintainable in such a case.
child in 2019 alleging domestic
abuse.

The husband initiated custody


proceedings in Singapore after
the wife had left while she
filed a guardianship petition
in Gurugram. Singapore court
passed an order directing the
minor child be returned to
Singapore and granted joint
custody to the parties.

Since the wife was afraid to


return, she continued staying
in India. The husband then
filed a Habeas Corpus petition
before the Punjab & Haryana
High Court.

The High Court while completed within six months.


Senior Advocate Geeta The Court agreed stating said
dismissing the same held that
Luthra appeared along with that such a petition would not (Source: https://www.
since the child is in the custody
Advocates Virender Tarun, be maintainable. barandbench.com/news/
of her natural guardian, it
Chirag Mahalwal, Kamakshi litigation/habeas-corpus-
cannot be termed as illegal CJI Ramana at one point
Gupta and Advocate Apoorva cannot-lie-against-mother-
detention and directed the observed that parents ego
Maheshwari for the wife. who-has-custody-of-child-
husband to approach the would kill the child.
competent authority under It was submitted that the wife supreme-court-rejects-
Guardians & Wards Act. had to leave Singapore after “It feels like because of your fathers-plea)

GENDERED CHILDCARE IN INDIA: TIME


FOR ‘HE’ FOR ‘SHE’
While childbearing is a role specific to women, the law fails to account for child rearing as an
equal responsibility of both partners.
As a lawyer, and also a with an obvious outcome. The In her strong and empowering gender equality, India’s role
woman who may give birth law on maternity leave and speech for the initiative ‘He in enabling equal treatment
to a child one day, the law the absence of the same on for She’, UN Woman Goodwill on the basis of sex and gender
tells me: I must contribute paternity leave in India creates Ambassador Emma Watson has also gained momentum.
disproportionately to a disparate burden on potential reflected on the importance Many reforms on abortion law
childcare. If a choice is to be mothers to act as primary of involvement of men and and permanent commission
made between childcare and caregivers. The law tells me, I people of all genders to achieve to women in the armed forces
work, I will be placed in a am unequal at home. gender equality. With such have been welcome. However,
position to make that choice, global mainstreaming of the real question is: has the

Bhopal +91-7676564400 Indore +91-9589613810, +91-731-4987379 Prayagraj +91-8114000926, +91-8114000927 Kanpur +91-8576066660, +91-8707605589 E: support@toprankers.com
Gurugram +91-8448444207 Delhi +91-9810610466 Ranchi +91-9334969993 Lucknow +91-6390576666 Jabalpur +91-7004386936 W: www.toprankers.com
57 The CLAT Post • September 2021
State been able to ensure through the Central Civil fructified. Pertinently, even by women and to empower
greater participation of men to Service (Leave Rules), 1972 this proposal only seeks to them. Any legislation drafted
ensure equality of the sexes? and similar rules for state ensure greater efficiency of hereunder cannot be subject
Laws that have taken the shape government employees. government employees and to a constitutional challenge
of protective (or beneficial) The Rules provide for 15 ease of living and is not aimed on the ground of unequal
legislation have in fact often days of paid paternity leave to further gender equality treatment alone. The Maternity
ignored the social context that before childbirth or up to 6 at home by dispelling the Benefit Act and other childcare
encircle gendered issues. This months from the date of the notion of women as primary leave are enacted under this
has made a special case for delivery of the child. Very caregivers. provision, coupled with the
childcare leave. few private organisations State’s duty under the Directive
provide for a similar 14-15 Protective legislation under Principles to ensure maternal
Legislated as a beneficial day period of paid paternity Article 15(3) may perpetuate health. Drafted with this good
provision for women, the leave as a matter of their own inequality intention of empowering
Maternity Benefit Act has initiative. Needless to state, the mothers, gender-specific
no parallel statute providing parenting rights may, however,
for paternity leave in India. do more harm than good.
Childcare leave that is available
to government employees till Protective legislation, however
the child turns eighteen is also well-intentioned, must be
exclusively available to female understood in the social
employees. In this context, it is context under which benefit is
argued that while childbearing sought to be provided. Notable
is a role specific to women, academician Professor Sandra
the law fails to account for Fredman in her book on
child rearing as an equal Discrimination Law (2011) has
responsibility of both partners. cautioned against protective
legislation which might appear
The law at present to remedy disadvantage, but in
fact reinforces sex stereotypes
At the outset, it is important
for women. In a similar tone,
to visit the current law on
the Supreme Court has also
maternity, paternity, and
warned against the dangers
childcare leave in India.
of protective legislation and
Under the State’s duty to held that it must not create
provide maternal care as per classifications that perpetuate
Article 42 of the Constitution, legal, social or economic
the Maternity Benefit Act, inferiority of women.
1961 (amended in 2017)
Reason being that such
mandatorily provides for 26
classifications may perpetuate
weeks of paid maternity leave
sex stereotypes and therefore
by every establishment, public
contribute to discrimination
or private, with 50 or more
rather than overcome it. To
employees. It also provides
achieve this, the Court has also
for other benefits such as a
consistently rejected the State’s
creche facility in or around
attitude towards pregnancy,
the establishment. The state unorganised sector remains
Article 14 of the Constitution motherhood and domestic
governments of Haryana, Bihar entirely precluded from any
provides that every person is obligations towards children
and other states have extended provision for paternity leave.
equal before the law and enjoys and family as roles only being
the benefit of this leave to a
The Central Service Rules also an equal protection of law. Any attributed to women. These
period of 52 weeks.
provide for an additional paid discrimination on the grounds stereotypes, premised on
On the contrary, there is childcare leave up to two years of sex/gender is prohibited assumptions about socially
no central legislation that for government employees under Article 15. This, ascribed roles of gender which
mandates paternity leave that can be availed by women however, does not prevent discriminate against women,
in establishments in India. employees only, till the child the State from making special are against the spirit of Article
The only legislative scheme reaches the age of 18. The provisions for the benefit of 14 of the Constitution.
for paternity leave is for proposal to provide this leave women under Article 15(3).
The purpose is to remedy the A notable distinction between
government employees to both partners has not yet
historic disadvantage faced pregnancy and parenting
Bhopal +91-7676564400 Indore +91-9589613810, +91-731-4987379 Prayagraj +91-8114000926, +91-8114000927 Kanpur +91-8576066660, +91-8707605589 E: support@toprankers.com
Gurugram +91-8448444207 Delhi +91-9810610466 Ranchi +91-9334969993 Lucknow +91-6390576666 Jabalpur +91-7004386936 W: www.toprankers.com
58 The CLAT Post • September 2021
rights made by Professor Guardianship Act, where the ensuring equal parenting more progressive equality law
Fredman nips the issue in the mother is recognised as a rights. jurisprudence in India.
bud. She argues that while guardian only secondary to the
the former requires benefits father of the child. The decisions of courts in The legislature must step up
solely granted to women, the Canada and of the European
The State stands in the witness Court of Human Rights To address gendered childcare
latter must be equally available in India, the legislature surely
to both mothers and fathers. box, compelled to revisit (ECHR) have also made
the law and answer: Does concrete observations in has a greater role to play than
Unless this is achieved, it is the judiciary. Moreover, it
difficult to alter the sexual the Maternity Benefit Act, dispelling the role of women as
legislated as a special provision primary child-carers. In Fraser is difficult to contemplate
division of labour at home, intervention by the courts to
which also subsequently under Article 15(3), truly v. Canada (2020), the Canadian
advance the cause of women? Supreme Court set aside a strike down the present law
affects the division of labour at since it would leave a complete
workplace. Special measures Does the disproportionate policy on job sharing positions
burden of childcare on women offered by the Canadian vacuum in the area of childcare
that empower (only) women leave. It is also certainly
with parenting rights, “run have deleterious implications Police, which permitted
for workplace equality and either parent to take up more difficult to raise a challenge
the risk of reinforcing their to the law on maternity leave
primary role as childcarers, thus gets caught in the net responsibilities at home, but
prohibited by Article 14 of the subsequently disentitled them itself, which is otherwise
and therefore perpetuating constitutionally valid under
their disadvantage” rather than Constitution? from pension benefits. The
Court found the policy to Article 15(3). Thus, it is only
transcend it. Taking a cue from other the legislature that is capable
have a disparate impact on
Thus, it is argued that jurisdictions female police officers who of bridging this disparity in
protective legislation such as predominantly availed the childcare leave and reducing
India is not alone in its the disparate impact caused by
maternal benefits, without leave and were made ineligible
struggle to achieve equality the present Central and state
an equivalent paternity leave, for pension. This was in
at home. While many legislation.
fails to understand the social violation of equal protection
jurisdictions fail to ensure even
context of childbearing in of laws under Article 15 of the Gender equality is an issue
maternity leave and severely
India. In the absence of Canadian Charter, and thus for fathers as much as it is for
violate the socio-economic
law enabling any assistance impermissible. mothers. It is the need of the
rights of women, I only focus
from their partners, it causes on jurisdictions to learn from. hour for sex discrimination
women to disproportionately Dealing with a different issue,
For this, Germany provides an the ECHR in Gruba & Ors v. legislation to be studied with
contribute to childcare. excellent example of legislative this lens, a focus which the
It suggests that besides Russia (2021) struck down a
policy on childcare. weak paternity leave policy legislature has missed for
childbirth, even childbearing several decades since the
is a role specific to women and for policemen in Russia which
Germany offers paid maternity Maternity Benefit Act was
perpetuates the sex stereotype disentitled fathers from taking
leave of 6 weeks prior and enacted. While social change
ascribed to the female gender. leave if maternal leave could
8 weeks after childbirth. It can only come with time,
Pressed with a law which be availed by their partners.
further provides for unpaid the legislature must at least
recognises them as the sole The State’s justification of
parental leave that can be endeavour to enable equal
benefactors of childbirth, paternity leave in undermining
availed by either or both responsibilities in childcare
women find themselves at the operational effectiveness
parents for up to three years. by granting equal parenting
home being the sole caretakers of the police force was
If both parents are working, rights. It is only when this
of their children. rejected by the Court as
the leave may be availed equality is achieved by the
unreasonable justification for
simultaneously or alternately. State that gender relations at
This even has implications sex discrimination. Unequal
During this period, the home will begin to change
for workplace equality where treatment was in violation
government offers parental and gender equality will be
prolonged childcare leave of Article 14 (right to non-
allowance to raise the child achieved in its true sense,
makes it difficult for women discrimination) and Article
(Elterngeldstellen). It is as contemplated by the
to resume employment, who 8 (right to family) of the
significant that the employer Constitution.
nevertheless face a gap in their European Convention on
cannot terminate the contract
professional life, something Human Rights and was thus (Source: https://thewire.in/
of either parent for the period
their partners seldom do. An impermissible. law/hindi-cannot-be-used-
of parental leave and entitles
unequal childcare leave policy both parents to resume for-official-correspondence-
thus contributes to gender With the social implication
employment at the same of questioning the role of with-tamil-nadu-madras-hc)
inequality by reiterating position (and working hours)
gender roles in childcare. On women as primary child
at which they left. The German carers, these decisions pave the
a separate note, this also bears policy on childcare leave is
a stark contrast to the Hindu way and offer guidance for a
evidently a case in point for

Bhopal +91-7676564400 Indore +91-9589613810, +91-731-4987379 Prayagraj +91-8114000926, +91-8114000927 Kanpur +91-8576066660, +91-8707605589 E: support@toprankers.com
Gurugram +91-8448444207 Delhi +91-9810610466 Ranchi +91-9334969993 Lucknow +91-6390576666 Jabalpur +91-7004386936 W: www.toprankers.com
59 The CLAT Post • September 2021

THIS JUDICIAL SELECTION NEEDS MORE


THAN A TWEAK
The collegium system and the mysteries underlining its decision-making dilute the importance
of the High Courts
In recent weeks, the Supreme positions with others, and a independent scrutiny of its press was that this logjam
Court of India’s collegium has slew of puisne judges will be decisions. These misgivings are owed to a reluctance amongst
been busy. New judges have moved to new courts. usually seen in the context of some of its members to elevate
a battle between the executive Justice Akil Kureshi to the
been appointed to the Court A need for transparency and the judiciary. Less evident Court. Indeed, it was only after
on its advice and long overdue is the effect that the failings a change in its composition
vacancies have been filled up. These recommendations are
have on the status of the that the panel recommended
seen as reflective of a new and
High Courts. Today, even on August 17 a list of names
Now, after a meeting held on proactive collegium. A resolve
without express constitutional for elevation. This list did not
September 16, the body has for swiftness is fine as far as
sanction, the collegium contain Justice Kureshi’s name.
made proposals to alter the it goes; clearing up vacancies

effectively exercises a power The perfunctory nature of


existing composition of various is a minimal requirement
of supervision over each of the collegium’s resolutions
High Courts. When these of a functioning system.
the High Courts. For nearly means that we do not know
recommendations are notified, What ought to concern us,
two years, despite vacancies the reasons for his exclusion.
though, is that long-standing We also do not know why five
new Chief Justices will be on the Bench, the collegium
apprehensions about the Chief Justices, including Justice
appointed to as many as made no recommendations for
collegium’s operation remain Kureshi, and several other
eight different courts, five appointments to the Supreme
unaddressed: specifically, puisne judges are now being
existingChief Justices will swap Court. The conjecture in the
its opacity and a lack of
Bhopal +91-7676564400 Indore +91-9589613810, +91-731-4987379 Prayagraj +91-8114000926, +91-8114000927 Kanpur +91-8576066660, +91-8707605589 E: support@toprankers.com
Gurugram +91-8448444207 Delhi +91-9810610466 Ranchi +91-9334969993 Lucknow +91-6390576666 Jabalpur +91-7004386936 W: www.toprankers.com
60 The CLAT Post • September 2021
transferred to different courts. Case, the word consultation appointments and transfers scheme envisages no
This is not to suggest that these has been interpreted to mean was an essential feature of power of administrative
decisions are unfounded. It “concurrence”. What is more, the Constitution. In other superintendence in the
is possible that each of the that concurrence, the Court words, the Court held that a Supreme Court over the High
choices made is predicated held there, ought to be secured body that found no mention Courts. But when transfers
on administrative needs. But not from the CJI alone, but in the actual text of the are made routine, when the
whatever the rationale, surely from a body of judges that Constitution had assumed a process of appointing Chief
the public has a right to know. the judgment described as a position so sacrosanct that it Justices to High Courts is
“collegium”. Thus, the Court could not be touched even by a shrouded in secrecy, a de
The middle course wound up creating a whole constitutional amendment. facto system of oversight is put
new process for making in place.
Separation of powers is a To be sure, the NJAC was
appointments and transfers
bedrock principle of Indian far from perfect. There Getting back the shine
and carved out a system where
constitutionalism. Inherent were legitimate fears that
notional primacy came to It is clear that we have come
in that idea is the guarantee the commission might have
rest in the top echelons of the a long way from a time when
of an autonomous judiciary. resulted in the appointment of
judiciary. Chief Justices of High Courts
To that end, the process of malleable judges. Therefore, it
appointing and transferring This procedure has since been is plausible to argue that until declined invitations to the
judges assumes salience. clarified. The collegium for a proper alternative is framed, Supreme Court, because they
But the question of how to appointments to the Supreme the collegium represents the valued the work that they
strike a balance between the Court and for transfers best solution; that allowing were already entrusted with.
sovereign function of making between High Courts now senior judges of the Supreme Restoring High Courts to
appointments and the need comprises the CJI and his four Court primacy in matters of that position of prestige must
to ensure an independent senior- most colleagues, and appointments and transfers be seen as essential to the
judiciary has long plagued the for appointments to the High is the only practical way to process of building trust in
republic. Courts comprises the CJI and guarantee the independence of our Constitution. Achieving
his two senior-most colleagues. the judiciary. this will no doubt require
The Constitution’s framers more than just a tweak in the
When appointing judges to the
wrestled over the question But when the Court struck process of appointments. But
High Courts, the collegium
for many days. Ultimately, down the NJAC, it also what is clear is that the present
must also consult other
they adopted what Dr. B.R. promised to reform the system and the mysteries
senior judges on the Supreme
Ambedkar described as a existing system. Six years underlining the decision-
Court who had previously
“middle course”. That path down the line those promises making only further dilute
served as judges of the High
stipulates the following: Judges have been all but forgotten. the High Courts’ prominence.
Court under consideration.
to the Supreme Court are to A new MoP, for instance, When Chief Justices are
All of this is contained in a
be appointed by the President is nowhere in sight. The moved around with alacrity,
“Memorandum of Procedure”
of India in consultation with considerations that must go and when they are accorded
(MoP). But there is, in fact, no
the Chief Justice of India (CJI) into the procedure for selecting tenures lasting a matter of
actual guidance on how judges
and such other judges that he judges is left unexplained. The months, at best, it is impossible
are to be selected.
deems fit. words “merit” and “diversity” for them to make any lasting
The NJAC and after are thrown around without changes. At some point we
Judges to the High Courts
any corresponding debates must take seriously the task
are to be appointed by the In 2015, Parliament sought on what they, in fact, mean. of reforming the existing
President in consultation to undo the labyrinthine Somehow, amidst all of this, scheme, because the status quo
with the CJI, the Governor procedures put in place by we have arrived at a consensus is ultimately corrosive of the
of the State and the Chief the Court through the 99th that enveloping a veil over the very institutions that it seeks to
Justice of that court. In the Constitutional Amendment. process of selection is essential protect.
case of transfers, the President The National Judicial to judicial autonomy, and that
may move a judge from one Appointments (Source: https://www.
there is no legitimate reason
High Court to another, after thehindu.com/opinion/
why the public ought to know
consulting the CJI. Commission (NJAC), that lead/this-judicial-selection-
how judges are chosen and
the law created, comprised needs-more-than-a-tweak/
Where primacy rests transferred.
members from the judiciary, article36619908.ece)
the executive, and the In the case of the latest
In this design, there is no
lay-public. But the Court set of recommendations,
mention of a “collegium”.
scuppered the efforts to replace five Chief Justices of
But since 1993, when the
the collegium and it held in High Courts have been
Supreme Court rendered a
the Fourth Judges Case that reshuffled. Our constitutional
ruling in the Second Judges
judicial primacy in making

Bhopal +91-7676564400 Indore +91-9589613810, +91-731-4987379 Prayagraj +91-8114000926, +91-8114000927 Kanpur +91-8576066660, +91-8707605589 E: support@toprankers.com
Gurugram +91-8448444207 Delhi +91-9810610466 Ranchi +91-9334969993 Lucknow +91-6390576666 Jabalpur +91-7004386936 W: www.toprankers.com
61 The CLAT Post • September 2021

Bhopal +91-7676564400 Indore +91-9589613810, +91-731-4987379 Prayagraj +91-8114000926, +91-8114000927 Kanpur +91-8576066660, +91-8707605589 E: support@toprankers.com
Gurugram +91-8448444207 Delhi +91-9810610466 Ranchi +91-9334969993 Lucknow +91-6390576666 Jabalpur +91-7004386936 W: www.toprankers.com

You might also like